22.07.2024

Формула расчета сопротивления при параллельном соединении: Онлайн-калькулятор расчета последовательного и параллельного соединения резисторов

Содержание

формула и примеры расчета сопротивления, напряжения, тока и мощности

При проектировании электрических схем возникает необходимость использования последовательного и параллельного соединений резисторов. Соединения применяются также и при ремонтах электрооборудования, поскольку в некоторых ситуациях невозможно найти эквивалентный номинал резистора. Выполнить расчет просто, и справиться с этой операцией может каждый.

Типы проводников

Проводимость веществом электрического тока связана с наличием в нем свободных носителей заряда. Их количество определяется по электронной конфигурации. Для этого необходима химическая формула вещества, при помощи которой можно вычислить их общее число. Значение для каждого элемента берется из периодической системы Дмитрия Ивановича Менделеева.

Электрический ток — упорядоченное движение свободных носителей заряда, на которые воздействует электромагнитное поле. При протекании тока по веществу происходит взаимодействие потока заряженных частиц с узлами кристаллической решетки, при этом часть кинетической энергии частицы превращается в тепловую энергию. Иными словами, частица «ударяется» об атом, а затем снова продолжает движение, набирая скорость под действием электромагнитного поля.

Процесс взаимодействия частиц с узлами кристаллической решетки называется электрической проводимостью или сопротивлением материала. Единицей измерения является Ом, а определить его можно при помощи омметра или расчитать. Согласно свойству проводимости, вещества можно разделить на 3 группы:

  1. Проводники (все металлы, ионизированный газ и электролитические растворы).
  2. Полупроводники (Si, Ge, GaAs, InP и InSb).
  3. Непроводники (диэлектрики или изоляторы).

Проводники всегда проводят электрический ток, поскольку содержат в своем атомарном строении свободные электроны, анионы, катионы и ионы. Полупроводники проводят электричество только при определенных условиях, которые влияют на наличие или отсутствие свободных электронов и дырок. К факторам, влияющим на проводимость, относятся следующие: температура, освещенность и т. д. Диэлектрики вообще не проводят электричество, поскольку в их структуре вообще отсутствуют свободные носители заряда. При выполнении расчетов каждый радиолюбитель должен знать зависимость сопротивления от некоторых физических величин.

Зависимость сопротивления

Значение электропроводимости зависит от нескольких факторов, которые необходимо учитывать при расчетах, изготовлении элементов резистивной нагрузки (резисторов), ремонте и проектировании устройств. К этим факторам необходимо отнести следующие:

  1. Температура окружающей среды и материала.
  2. Электрические величины.
  3. Геометрические свойства вещества.
  4. Тип материала, из которого изготовлен проводник (полупроводник).

К электрическим величинам можно отнести разность потенциалов (напряжение), электродвижущую силу (ЭДС) и силу тока. Геометрией проводника является его длина и площадь поперечного сечения.

Электрические величины

Зависимость величины электропроводимости от параметров электричества определяется законом Ома. Существует две формулировки: одна — для участка, а другая — для полной цепи. В первом случае соотношение определяются, исходя из значений силы тока (I) и напряжения (U) простой формулой: I = U / R. Из соотношения видна прямо пропорциональная зависимость тока от величины напряжения, а также обратно пропорциональная от сопротивления. Можно выразить R: R = U / I.

Для расчета электропроводимости всего участка следует воспользоваться соотношением между ЭДС (e), силой тока (i), а также внутренним сопротивлением источника питания (Rвн): i = e / (R+Rвн). В этом случае величина R вычисляется по формуле: R = (e / i) — Rвн. Однако при выполнении расчетов необходимо учитывать также геометрические параметры и тип проводника, поскольку они могут существенно повлиять на вычисления.

Тип и геометрические параметры

Свойство вещества к проводимости электричества определяется структурой кристаллической решетки, а также количеством свободных носителей. Исходя из этого, тип вещества является ключевым фактором, который определяет величину электропроводимости. В науке коэффициент, определяющий тип вещества, обозначается литерой «р» и называется удельным сопротивлением. Его значение для различных материалов (при температуре +20 градусов по Цельсию) можно найти в специальных таблицах.

Иногда для удобства расчетов используется обратная величина, которая называется удельной проводимостью (σ). Она связана с удельным сопротивлением следующим соотношением: p = 1 / σ. Площадь поперечного сечения (S) влияет на электрическое сопротивление. С физической точки зрения, зависимость можно понять следующим образом: при малом сечении происходят более частые взаимодействия частиц электрического тока с узлами кристаллической решетки. Поперечное сечение можно вычислить по специальному алгоритму:

  1. Измерение геометрических параметров проводника (диаметр или длину сторон) при помощи штангенциркуля.
  2. Визуально определить форму материала.
  3. Вычислить площадь поперечного сечения по формуле, найденной в справочнике или интернете.

В случае когда проводник имеет сложную структуру, необходимо вычислить величину S одного элемента, а затем умножить результат на количество элементов, входящих в его состав. Например, если провод является многожильным, то следует вычислить S для одной жилы. После этого нужно умножить, полученную величину S, на количество жил. Зависимость R от вышеперечисленных величин можно записать в виде соотношения: R = p * L / S. Литера «L» является длиной проводника. Однако для получения точных расчетов необходимо учитывать температурные показатели внешней среды и проводника.

Температурные показатели

Существует доказательство зависимости удельного сопротивления материала от температуры, основанное на физическом эксперименте. Для проведения опыта нужно собрать электрическую цепь, состоящую из следующих элементов: источника питания, нихромовой спирали, соединительных проводов амперметра и вольтметра. Приборы нужны для измерения значений силы тока и напряжения соответственно. При протекании электричества происходит нагревание нихромовой пружины. По мере ее нагревания, показания амперметра уменьшаются. При этом происходит существенное падение напряжения на участке цепи, о котором свидетельствуют показания вольтметра.

В радиотехнике уменьшение величины напряжение называется просадкой или падением. Формула зависимости р от температуры имеет следующий вид: p = p0 * [1 + a * (t — 20)]. Значение p0 — удельное сопротивление материала, взятого из таблицы, а литера «t» — температура проводника.

Температурный коэффициент «а» принимает следующие значения: для металлов — a>0, а для электролитических растворов — a<0. Для получения формулы, определяющей все зависимости, необходимо подставить все соотношения в общую формулу зависимости R от типа материала, температуры, длины и сечения: R = p0 * [1 + a * (t — 20)] * L / S. Формулы используются только для расчетов и изготовления резисторов. Для быстрого измерения величины сопротивления применяется омметр.

Объединение резистивных радиокомпонентов

Для получения необходимого номинала сопротивления применяются два типа соединения резисторов: параллельное и последовательное. Если их соединить параллельно, то нужно два вывода одного резистора подключить к двум выводам другого. Если соединение является последовательным, то один вывод резистора соединяется с одним выводом другого резистора. Соединения используются для получения необходимых номиналов сопротивлений, а также для увеличения рассеивания мощности тока, протекающего по цепи.

Каждое из соединений обладает определенными характеристиками. Кроме того, последовательно или параллельно могут объединяться несколько резисторов. Соединения также могут быть смешанными, т. е. применяться оба типа объединения радиокомпонентов.

Параллельное соединение

При параллельном подключении значение напряжения на всех резисторах одинаковое, а сила тока — обратно пропорциональна их общему сопротивлению. В интернете web-разработчики создали для расчета величины общего сопротивления параллельного соединения резисторов онлайн-калькулятор.

Рассчитывается общее сопротивление при параллельном соединении по формуле: 1 / Rобщ = (1 / R1) + (1 / R2) + …+ (1 / Rn). Если выполнить математические преобразования и привести к общему знаменателю, то получится удобная формула параллельного соединения для расчета Rобщ. Она имеет следующий вид: Rобщ = (R1 * R2 * … * Rn) / (R1 + R2 + … + Rn). Если необходимо рассчитать величину Rобщ только для двух радиокомпонентов, то формула параллельного сопротивления имеет следующий вид: Rобщ = (R1 * R2) / (R1 + R2).

При ремонте или проектировании схемы устройства возникает задача объединения нескольких резистивных элементов для получения конкретной величины сопротивления. Например, значение Rобщ для определенной цепочки элементов равно 8 Ом, которое получено при расчетах. Перед радиолюбителем стоит задача, какие нужно подобрать номиналы для получения нужного значения (в стандартном ряду резисторов отсутствует радиокомпонент с номиналом в 8 Ом, а только 7,5 и 8,2). В этом случае нужно найти сопротивление при параллельном соединении резистивных элементов. Посчитать значение Rобщ для двух элементов можно следующим образом:

  1. Номинал резистора в 16 Ом подойдет.
  2. Подставить в формулу: R = (16 * 16) / (16 + 16) = 256 / 32 = 8 (Ом).

В некоторых случаях следует потратить больше времени на подбор необходимых номиналов. Можно применять не только два, но и три элемента. Сила тока вычисляется с использованием первого закона Кирхгофа. Формулировка закона следующая: общее значение тока, входящего и протекающего по цепи, равен выходному его значению. Величина силы тока для цепи, состоящей из двух резисторов (параллельное соединение) рассчитывается по такому алгоритму:

  1. Ток, протекающий через R1 и R2: I1 = U / R1 и I2 = U / R2 соответственно.
  2. Общий ток — сложение токов на резисторах: Iобщ = I1 + I2.

Например, если цепь состоит из 2 резисторов, соединенных параллельно, с номиналами в 16 и 7,5 Ом. Они запитаны от источника питания напряжением в 12 В. Значение силы тока на первом резисторе вычисляется следующим способом: I1 = 12 / 16 = 0,75 (А). На втором резисторе ток будет равен: I2 = 12 / 7,5 = 1,6 (А). Общий ток определяется по закону Кирхгофа: I = I1 + I2 = 1,6 + 0,75 = 2,35 (А).

Последовательное подключение

Последовательное включение резисторов также применяется в радиотехнике. Методы нахождения общего сопротивления, напряжения и тока отличаются от параллельного подключения. Основные правила соединения следующие:

  1. Ток не изменяется на участке цепи.
  2. Общее напряжение равно сумме падений напряжений на каждом резисторе.
  3. Rобщ = R1 + R2 + … + Rn.

Пример задачи следующий: цепочка, состоящая из 2 резисторов (16 и 7,5 Ом), питается от источника напряжением 12 В и током в 0,5 А. Необходимо рассчитать электрические параметры для каждого элемента. Порядок расчета следующий:

  1. I = I1 = I2 = 0,5 (А).
  2. Rобщ = R1 + R2 = 16 + 7,5 = 23,5 (Ом).
  3. Падения напряжения: U1 = I * R1 = 0,5 * 16 = 8 (В) и U2 = I * R2 = 0,5 * 7,5 = 3,75 (В).

Не всегда выполняется равенство напряжений (12 В не равно 8 + 3,75 = 11,75 В), поскольку при этом расчете не учитывается сопротивление соединительных проводов. Если схема является сложной, и в ней встречается два типа соединений, то нужно выполнять расчеты по участкам. В первую очередь, рассчитать для параллельного соединения, а затем для последовательного.

Таким образом, параллельное и последовательное соединения резисторов применяются для получения более точных значений сопротивлений, а также при отсутствии необходимого номинала радиокомпонента при проектировании или ремонте устройств.

Сопротивление при параллельном соединении, формула для расчета сопротивления при параллельном соединении

В этой статье мы разберем, как посчитать общее сопротивление при параллельном соединении сопротивлений. Параллельным соединением сопротивлений называется соединение (рисунок ниже), при котором один зажим каждого из сопротивлений присоединяется к одной точке (узлу) электрической цепи, а другой зажим каждого из тех же сопротивлений присоединяется к другой точке электрической цепи. Таким образом, между двумя точками (узлами) электрической цепи включается несколько сопротивлений, образующих параллельные ветви.

При этом напряжение между концами всех ветвей будет одним и тем же, а токи в отдельных ветвях определяются по закону Ома:
I1 = U / r1 ; I2 = U / r2 ; I3 = U / r3.

Напряжение U между узлами (А и Б):
U = I1r1 = I2r2 = I3r3,
откуда
I1 / I2 = R2 / R1  и  I2 / I3 = R3 / R2,
т. е.

Токи в параллельных ветвях распределяются обратно пропорционально их сопротивлениям.

Согласно первому правилу Кирхгофа,
I = I1 + I2 + I3
или
U / Rсум = U / R1 + U / R2 + U / R2 = U (1 / R1 + 1 / R2 + 1 / R3).
Произведя сокращение на U, получим:
1 / Rсум = 1 / R1 + 1 / R2 + 1 / R3
или
g = g1 + g2 + g3 ,

где R и g—сопротивление и проводимость разветвленной цепи или, как их часто называют, общие сопротивление и проводимость цепи.
Из полученной формулы следует, что

Общая проводимость разветвленной цепи равна сумме проводимостей отдельных ветвей.

Формула
1 / Rсум = 1 / R1 + 1 / R2 + 1 / R3
дает возможность определить общее сопротивление цепи. Например, для трех параллельно соединенных сопротивлений, приведя правую часть уравнения к общему знаменателю, получим:
1 / Rсум = R2R3 + R1R3 + R1R2 / R1R2R3
откуда
Rсум = R1R2R3 / R2R3 + R1R3 + R1R2
Если сопротивления R1 = R2 = R3, то общее сопротивление цепи:
Rсум = R1 / 3,
а в общем случае при n параллельных ветвях с равными сопротивлениями R1 :
Rсум = R1 / n
В случае двух параллельных ветвей:
1 / Rсум = 1 / R1 + 1 / R2
откуда
Rсум = R1R2 / R2R3 + R1R3

При параллельном соединении приемников энергии все они находятся под одним и тем же напряжением, и режим работы каждого из них не зависит от остальных. Совершенно иначе обстоит дело при последовательном соединении приемников, при котором изменение сопротивления одного из них тотчас же приводит к изменению напряжения на других, последовательно соединенных с ним. Поэтому электрические лампы и двигатели, предназначенные для работы при определенном (номинальном) напряжении, включаются параллельно. Одинаковые электрические лампы иногда соединяются последовательно. Пусть, например, напряжение сети U, а напряжение лампы U0 < U Тогда n ламп соединяются цепочкой друг за другом, причем n > U / U0
Такое соединение ламп можно встретить, например, в трамваях, метро и других случаях.

Пример 1:
К сети с напряжением 220 в параллельно подключены двигатель мощностью 1,1 квт и 11 ламп, каждая мощностью 40 вт. Определить ток в главных (подводящих) проводах

Ток двигателя
I1 = P1 / U = 1100 / 220 = 5a.
Ток ламп
I2 = P2 / U = 11 x 40 / 220 = 2a.
Ток в подводящих проводах
I = I1 + I2 = 5 + 2 = 7a.

Пример 2:
Определить общее сопротивление десяти параллельно включенных ламп накаливания, если каждая из них 240 ом:
R = Rл / n = 240 / 10 = 24ом.

Формула расчета сопротивления при параллельном соединении резистора

Параллельное соединение резисторов — одно из двух видов электрических соединений, когда оба вывода одного резистора соединены с соответствующими выводами другого резистора или резисторов. Зачастую резисторы соединяют последовательно или параллельно для того, чтобы создать более сложные электронные схемы.

Схема параллельного соединения резисторов показан на рисунке ниже. При параллельном соединении резисторов, напряжение на всех резисторах будет одинаковым, а протекающий через них ток будет пропорционален их сопротивлению:

Формула параллельного соединения резисторов

Общее сопротивление нескольких резисторов соединенных параллельно определяется по следующей формуле:

Ток, протекающий через отдельно взятый резистор, согласно закону Ома, можно найти по формуле:

При разработке устройства, возникла необходимость установить резистор с сопротивлением 8 Ом. Если мы просмотрим весь номинальный ряд стандартных значений резисторов, то мы увидим, что резистора с сопротивлением в 8 Ом в нем нет.

Выходом из данной ситуации будет использование двух параллельно соединенных резисторов. Эквивалентное значение сопротивления для двух резисторов соединенных параллельно рассчитывается следующим образом:

Данное уравнение показывает, что если R1 равен R2, то сопротивление R составляет половину сопротивления одного из двух резисторов. При R = 8 Ом, R1 и R2 должны, следовательно, иметь значение 2 × 8 = 16 Ом.
Теперь проведем проверку, рассчитав общее сопротивление двух резисторов:

Таким образом, мы получили необходимое сопротивление 8 Ом, соединив параллельно два резистора по 16 Ом.

Пример расчета №2

Найти общее сопротивление  R из трех параллельно соединенных резисторов:

Общее сопротивление R рассчитывается по формуле:

Этот метод расчета может быть использованы для расчета любого количества отдельных сопротивлений соединенных параллельно.

Один важный момент, который необходимо запомнить при расчете параллельно соединенных резисторов – это то, что общее сопротивление всегда будет меньше, чем значение наименьшего сопротивления в этой комбинации.

Как рассчитать сложные схемы соединения резисторов

Более сложные соединения резисторов могут быть рассчитаны путем систематической группировки резисторов. На рисунке ниже необходимо посчитать общее сопротивление цепи, состоящей из трех резисторов:

Резисторы R2 и R3 соединены последовательно (группа 2). Они в свою очередь соединены параллельно с резистором R1 (группа 1).

Последовательное соединение резисторов группы 2 вычисляется как сумма сопротивлений R2 и R3:

В результате мы упрощаем схему в виде двух параллельных резисторов. Теперь общее сопротивление всей схемы можно посчитать следующим образом:

Расчет более сложных соединений резисторов можно выполнить используя законы Кирхгофа.

Ток, протекающий в цепи параллельно соединенных резисторах

Общий ток I протекающий в цепи параллельных резисторов равняется сумме отдельных токов, протекающих во всех параллельных ветвях, причем ток в отдельно взятой ветви не обязательно должен быть равен току в соседних ветвях.

Несмотря на параллельное соединение, к каждому резистору приложено одно и то же напряжение. А поскольку величина сопротивлений в параллельной цепи может быть разной, то и величина протекающего тока через каждый резистор тоже будет отличаться (по определению закона Ома).

Рассмотрим это на примере двух параллельно соединенных резисторов. Ток, который течет через каждый из резисторов ( I1 и I2 ) будет отличаться друг от друга поскольку сопротивления резисторов R1 и R2 не равны.
Однако мы знаем, что ток, который поступает в цепь в точке «А» должен выйти из цепи в точке «B» .

Первое правило Кирхгофа гласит: «Общий ток, выходящий из цепи равен току входящий в цепь».

  • Таким образом, протекающий общий ток в цепи  можно определить как:
  • I = I1 + I2
  • Затем с помощью закона Ома можно вычислить ток, который протекает через каждый резистор:
  • Ток, протекающий в R1 = U ÷ R1 = 12 ÷ 22 кОм = 0,545 мА
  • Ток, протекающий в R 2 = U ÷ R2 = 12 ÷ 47 кОм = 0,255 мА
  • Таким образом, общий ток будет равен:
  • I = 0,545 мА + 0,255 мА = 0,8 мА
  • Это также можно проверить, используя закон Ома:
  • I = U ÷ R = 12 В ÷ 15 кОм = 0,8 мА (то же самое)
  • где 15кОм — это общее сопротивление двух параллельно соединенных резисторов (22 кОм и 47 кОм)
  • И в завершении хочется отметить, что большинство современных резисторов маркируются цветными полосками и назначение ее можно узнать здесь.

Параллельное соединение резисторов — онлайн калькулятор

Чтобы быстро вычислить общее сопротивление двух и более резисторов, соединенных параллельно, вы можете воспользоваться следующим онлайн калькулятором:

Подведем итог

Когда два или более резистора соединены так, что оба вывода одного резистора соединены с соответствующими выводами другого резистора или резисторов, то говорят, что они соединены между собой параллельно. Напряжение на каждом резисторе внутри параллельной комбинации одинаковое, но токи, протекающие через них, могут отличаться друг от друга, в зависимости от величины сопротивлений каждого резистора.

Эквивалентное или полное сопротивление параллельной комбинации всегда будет меньше минимального сопротивления резистора, входящего в параллельное соединение.

Источник: http://www.joyta.ru/7362-parallelnoe-soedinenie-rezistorov/

Последовательное и параллельное соединение резисторов

Последовательное соединение – это соединение двух или более резисторов в форме цепи, в которой каждый отдельный резистор соединяется с другим отдельным резистором только в одной точке.

Общее сопротивление Rобщ

При таком соединении, через все резисторы проходит один и тот же электрический ток. Чем больше элементов на данном участке электрической цепи, тем «труднее» току протекать через него. Следовательно, при последовательном соединении резисторов их общее сопротивление увеличивается, и оно равно сумме всех сопротивлений.

Напряжение при последовательном соединении

Напряжение при последовательном соединении распределяется на каждый резистор согласно закону Ома:

Т.е чем большее сопротивление резистора, тем большее напряжение на него падает.

Параллельное соединение резисторов

Параллельное соединение – это соединение, при котором резисторы соединяются между собой обоими контактами. В результате к одной точке (электрическому узлу) может быть присоединено несколько резисторов.

Общее сопротивление Rобщ

При таком соединении, через каждый резистор потечет отдельный ток. Сила данного тока будет обратно пропорциональна сопротивлению резистора. В результате общая проводимость такого участка электрической цепи увеличивается, а общее сопротивление в свою очередь уменьшается.

Таким образом, при параллельном подсоединении резисторов с разным сопротивлением, общее сопротивление будет всегда меньше значения самого маленького отдельного резистора.

Формула общей проводимости при параллельном соединении резисторов:

Формула эквивалентного общего сопротивления при параллельном соединении резисторов:

Для двух одинаковых резисторов общее сопротивление будет равно половине одного отдельного резистора:

Соответственно, для n одинаковых резисторов общее сопротивление будет равно значению одного резистора, разделенного на n.

Напряжение при параллельном соединении

Напряжение между точками A и B является как общим напряжением для всего участка цепи, так и напряжением, падающим на каждый резистор в отдельности. Поэтому при параллельном соединении на все резисторы упадет одинаковое напряжение.

Электрический ток при параллельном соединении

Через каждый резистор течет ток, сила которого обратно пропорциональна сопротивлению резистора. Для того чтобы узнать какой ток течет через определенный резистор, можно воспользоваться законом Ома:

Смешанное соединение резисторов

Смешанным соединением называют участок цепи, где часть резисторов соединяются между собой последовательно, а часть параллельно. В свою очередь, смешанное соединение бывает последовательного и параллельного типов.

Общее сопротивление Rобщ

Для того чтобы посчитать общее сопротивление смешанного соединения:

  • Цепь разбивают на участки с только пареллельным или только последовательным соединением.
  • Вычисляют общее сопротивление для каждого отдельного участка.
  • Вычисляют общее сопротивление для всей цепи смешанного соединения.

Так это будет выглядеть для схемы 1:

Также существует более быстрый способ расчета общего сопротивления для смешанного соединения. Можно, в соответствии схеме, сразу записывать формулу следующим образом:

  • Если резисторы соединяются последоватеьно — складывать.
  • Если резисторы соединяются параллельно — использовать условное обозначение «||».
  • Подставлять формулу для параллельного соединения где стоит символ «||».

Так это будет выглядеть для схемы 1:

После подстановки формулы параллельного соединения вместо «||»:

Источник: http://hightolow.ru/resistor3.php

Параллельное соединение сопротивлений в электрической цепи. Параллельное соединение конденсаторов и катушек

Параллельное соединение электрических элементов (проводников, сопротивлений, емкостей, индуктивностей) — это такое соединение, при котором подключенные элементы цепи имеют два общих узла подключения.

Другое определение: сопротивления подключены параллельно, если они подключены одно и той же паре узлов. 

Графическое обозначение схемы параллельного соеднинения

На приведенном рисунке показана схема параллельное подключения сопротивлений R1, R2, R3, R4. Из схемы видно, что все эти четыре сопротивления имеют две общие точки (узла подключения). 

В электротехнике принято, но не строго требуется, рисовать провода горизонтально и вертикально. Поэтому эту же схему можно изобразить, как на рисунке ниже. Это тоже параллельное соединение тех же самых сопротивлений.

Формула для расчета параллельного соединения сопротивлений

При параллельном соединении обратная величина от эквивалентного сопротивления равна сумме обратных величин всех параллельно подключенных сопротивлений. Эквивалентная проводимость равна сумме всех параллельно подключенных проводимостей электрической схемы.

Для приведенной выше схемы эквивалентное сопротивление можно рассчитать по формуле:

В частном случае при подключении параллельно двух сопротивлений:

Эквивалентное сопротивление цепи определяется по формуле:

 В случае подключения «n» одинаковых сопротивлений, эквивалентное сопротивление можно рассчитать по частной формуле:

Формулы для частного рассчета вытекают из основной формулы. 

Формула для расчета параллельного соединения емкостей (конденсаторов)

При параллельном подключении емкостей (конденсаторов) эквивалентная емкость равна сумме параллельно подключенных емкостей:

 

Формула для расчета параллельного соединения индуктивностей

  • При параллельном подключении индуктивностей, эквивалентная индуктивность рассчитывается так же, как и эквивалентное сопротивление при параллельном соединении: 
  •  
  • Необходимо обратить внимание, что в формуле не учтены взаимные индуктивности.

Пример свертывания параллельного сопротивления  

Для участка электрической цепи необходимо найти параллельное соединение сопротивлений выполнить их преобразование до одного.

Из схемы видно, что параллельно подключены только R2 и R4. R3 не параллельно, т.к. одним концом оно подключено к источнику ЭДС E1. R1 — одним концом подключено к R5, а не к узлу. R5 — одним концом подключено к R1, а не к узлу. Можно так же говорить, что последовательное соединение сопротивлений R1 и R5 подключено параллельно с R2 и R4.

Рассчитать эквивалентное сопротивлений R14 можно по формуле для двух сопротивлений.

Ток при параллельном соединении

При параллельном соединении сопротивлений ток через каждое сопротивление в общем случае разный. Величина тока обратно пропорциональна величине сопротивления.

Напряжение при параллельном соединении 

При параллельном соединении разность потенциалов между узлами, объединяющими элементы цепи, одинакова для всех элементов.

Применение параллельного соединения

1. В промышленности изготавливаются сопротивления определенных величин. Иногда необходимо получить значение сопротивления вне данных рядов. Для этого можно подключить несколько сопротивлений параллельно. Эквивалентное сопротивление всегда будет меньше самого большого номинала сопротивления.

2. Делитель токов.

Источник: https://kurstoe.ru/osnovnie-svedeniya/preobrazovanie-tcepej/parallelnoe-soedinenie.html

Последовательное и параллельное соединение резисторов

Последовательное и параллельное соединение резисторов в схемах являются самыми распространенными, также — это база для расчета более сложных схем.

Последовательное подключение

Начнем с последовательного соединения. По этой схеме каждый резистор подключается с другим только в одной точке, их может быть в цепи 2, 3 и больше.

Рис. Последовательное подключение.

Обозначение:

Обозначим сопротивления: R1, R2, R3 и напряжение источника в цепи Uц. При подключении источника питания в ней начнет протекать ток Iц. В цепи с последовательным соединением ток протекает по всем резисторам один за другим.

Поскольку ток течет через все резисторы их сопротивления и ток суммируется, Iц = I1+I2+I3, Rц = R1 +R2 + R3, чем больше отдельно взятое сопротивление, тем тяжелее электронам преодолевать участок цепи. Мощность резисторов при последовательном и параллельном соединении рассчитывается по разным формулам.

В последовательных цепях — складываем, в параллельных — это обратно пропорциональная величина.

Параллельное соединение

Рис. Параллельное подключение.

Данный вид подключения характерен тем, что все элементы цепи соединяется выводами в одной точке друг другу, т.е. точка входа и выхода всех нагрузок сходятся в одну точку (или еще одно обозначение на схемах — //). Электроток, двигаясь по проводнику, дойдя до общего соединения делится на количество имеющихся веток. Если представить движение воды в трубе, то можно сказать, что вода двигающиеся по одной трубе, равномерно перетекает в несколько отводов, подсоединенных к ней. В нашем случае заряженные электроны, двигающиеся по проводнику, также растекаются на количества предложенных веток в узле. Более наглядно это можно представить в виде формул: 1. Каждый вид соединения находится под одинаковым напряжением: U = U1 = U2; 2. Суммарная сила тока равняется суммарному значению тока каждого участка I = I1 + I2; 3. Сопротивление цепи равно сумме величина обратных сопротивлению участка: 1/R = 1/R1 + 17R2 + . . . + 1/Rn; 4. Сила тока пропорциональна сопротивлению каждого участка I1/I2=R2/R1.

Далее рассмотрим схему как работает не только последовательное параллельное, но и смешанное соединение резисторов.

Смешанное подключение

Рис. Смешанное подключение резисторов

В электрических схемах используются не только типовые схемы, но и смешанное, созданное из критерий определенных требований. Чаще всего в схемах встречается третий вариант, представляющий набор из элементарных типов схем. В смешанных участках учитываются не только элементы, но и направления движения тока. При вычислении мощности резисторов смешанного подключения используются формулы для параллельного и последовательного соединения резисторов, формула также является составной.

Основные законы электротехники, наиболее часто используемые для расчетов

Рассмотрим основные законы электротехники и свойства последовательного и параллельного соединения резисторов для участка цепи

Закон Ома

Напряжение находится по закону Ома по формуле I=U/R — чем больше сопротивление, тем меньше ток. Напряжение можно найти из этой же формулы. U=R*I, ток умножается на сопротивление. Запишем эту формулу для каждого участка U1=R1· I1, Un=Rn · In.

Законы Кирхгофа

Первый закон

Ещё один очень важный закон — это закон Кирхгофа. Для участка цепи постоянного тока их два.

Рис. иллюстрация к пояснению действия первого закона Кирхгофа.

Первый закон имеет формулировку: Сумма всех токов, входящих в узел и выходящих из него равна нулю. Если посмотреть на схему, I1 — это ток, который заходит в узел, I2 и I3 — это электроны, которые вытекают из него. Применяя формулировку первого закона можно записать формулу по-другому:

I1-I2+I3=0. В этой формуле знаки плюс имеют значения, которые прибывают в узел, минус, который отходит от него.

Второй закон Кирхгофа

Рис. иллюстрация к пояснению действия второго закона Кирхгофа.

Если к цепи с включенными сопротивлениями подключен один источник ЭДС (батарея питания) тогда всё понятно, можно обойтись законом Ома. А, если, источников несколько и схема с различным схемным расположением элементов, тогда вступает в силу второй закон, который гласит: сумма токов всех источников питания для замкнутого контура, равна сумме падений напряжения на всех сопротивлениях участка в этом контуре. E1- Е2 = — UR1 — UR2 или E1 = Е2 — UR1 — UR2.

Параллельное и последовательное соединение резисторов,  решение задач

Алгоритм расчёта смешанных подключений находится в тех же правилах, что и в элементарных схемах расчета последовательного и параллельного соединения резисторов. Ничего нового нет: нужно правильно разбить предложенную схему на пригодные для расчета участки. Участки, с элементами, подключены поочередно либо параллельно.

Рис. Порядок замещения при расчете сложных позиций более простыми.

Для решения задачи на последовательное и параллельное соединение резисторов необходимо правильно оценить цепи элементов. Рассмотрим схему №1 на рис.

На схеме присутствует параллельная и последовательная часть соединения элементов. Для расчета очень важно аккуратно, шаг за шагом упрощать цепи и не брать сразу всю схему (рис.1).

Как же правильно определить параллельное и последовательное соединение резисторов?

Для примера расчета возьмем резисторы R3, R4, которые подключены параллельно. Эквивалентный резистор этих элементов, будет равенRэ. = 1/R34 =1/R3 + 1/R4, после преобразования формулы и приведения к одному знаменателю получим R34 = R3 · R4 / (R3 + R4). Э. = 1/3+1/4 /(3+4) =1,7 Ом.

Далее видно, что приведённая эквивалентное R эк и R6 соединены последовательно, чтобы узнать сопротивление их необходимо сложить, тогда общее сопротивление будет равно R346 = R34 + R6, тогда Rэк346 = 1,7 + 6 = 7, 7 Ом. Заменяем на схеме одним общим элементом, теперь, позиция упрощается еще больше (рис 3).

Теперь образовалась ситуация — включение трех элементов в //. Как вычисляется такое соединение нам уже известно, 1/ R23465 = 1/ R2 +1/R346 + 1/R5 после вычисления правой части получаем 0,82 Ом. После окончательного вычисления получаем R23465 = 2,1 Ом. Здесь следует обратить внимание, что общее сопротивление получилось меньше самого меньшего из трех.

Заменяем эти сопротивление одним эквивалентным R23465. В конечном итоге все выглядит уже намного проще. Rц = Rэк + R1+ R2. R об. = R ц = 1,21 +7+1 =9,21 Ом. Из приведенного алгоритма расчёта видно, как из сложной схемы путем простого математического вычисления и применения правил сокращения резисторов участок становится простой и понятной.

Схема с подключением сопротивлений «треугольником»

Рис. Расчетная схема соединения резисторов в треугольник.

Иногда некоторые затруднения возникают при разборе схемы соединения в треугольник.

Рассмотрим на примере рисунка расчет резисторов по этому подключению. Из схемы видно, что R1 и R2 соединены последовательно Rэ12 будет соединяться R3 последовательно.

Затем Rэ123 соединяется с сопротивлением R4, R5 в последовательную цепь. Затем все это объединяется с Rэ в //.

Проведем несложные вычисления учитывая, что R1, R2, R4, R5 равняется 1 Ом. R3, R7 — 2 Ом.

RЭ1,2 = R1+R2 = 1+1=2 Ом.

Вычисляем параллельное подключение: Rэ 12 с R3. Rэ1,3 = (Rэ12*R3) /(Rэ12+R3) = (2*2) /(2+2) = 1Ом.

Далее мы видим последовательное: RЭ123 + R4 + R5 = 1+1+1 = 3 Ом. И последнее — Rэ123 4 5 с R6 — параллельное.

Общее сопротивление цепи Rц = Rоб = (RЭ1,2,3,4,5 *R6) /(RЭ1,2,3,4,5+R6) = (3 * 2) / (3+2) = 1,2 Ом. Как видно, что расчет подобного варианта также не сложный.

Расчет последовательного и параллельного подключения резисторов онлайн

Подсчитать значение мощность и сопротивлений подставляя их в формулы можно только в учебных целях, или, когда объемы не очень большие.

Наиболее практичный вариант расчета является онлайн калькуляторы, которые расположены на многочисленных интернет ресурсах.

Для расчёта любой сложности нужно правильно определить тип соединения резисторов последовательное или параллельное и внести данные для расчета в поля калькулятора.

Также такая форма расчета подойдет и для проверки результатов решения учебных задач.

Последовательное и параллельное соединение резисторов и конденсаторов

Электрические цепи состоят не только из резисторов, в них применяется большое количество различных деталей, например, конденсатор, которые подключаются в последовательное, // и смешанное соединение.

Рис. Замещения последовательно включенных элементов.

Определение этому элементу можно дать следующее: Конденсатор — это совокупность проводящих тел служащий для накопления электрического заряда. Элементарный конденсатор имеет две пластины, форма этих пластин может быть различной: сферической, круглой, цилиндрической, прямоугольной — по форме пластин разделяется и тип конденсатора.

Важное свойство. Одно из важных свойств конденсатора: если заряжается одна пластина конденсатора, то благодаря явлению электростатической индукции заряжается и вторая половина, но с противоположным знаком.

Устройство конденсатора

Плоский конденсатор состоит из двух плоских пластин отстоящих друг от друга на маленькое расстояние. У конденсатора к двум пластинам припаивается вывод всего их получается два.

Типовые схемы подключения конденсаторов

Рассмотрим различные виды подключения конденсатора.

Последовательное

Первый вид — это последовательное соединение. Предположим, что емкость этих конденсаторов будут равны.

Тогда заряды также будут равны: q1=q2=q3, как и в примере с резисторами,  сложный тип позиций с конденсатором можно упростить, заменив несколько элементов одним.

У элементов соединенных друг за другом, общая емкость будет обратно пропорциональная всем имеющимся элементам. То есть: Rэк будет равняться 1/С1 + 1/С2 +…. 1/Сn/

Напряжение складывается,  U эк = U1 + U2+ … Un.

Параллельное

Второй тип подключения конденсаторов — это соединение в паралель

Рис. Схема замещения элементов, включенных в параллель.

  • Соответственно эти конденсаторов обозначены C1, C2, … Cn заряды: Q1, Q2, … Qn и напряжение: U1, U2, … Un.
  • У элементов в // емкость складывается Сэ = C1 + C2 + … C n. Напряжение Un на каждом конденсаторе будет равно напряжению на эквивалентном
  • Uэ = U1 = U2 =… = Un — это особенность параллельного подсоединения всех элементов цепи.
  • Емкость будет складываться из суммы отдельных элементов Сэ =С1 + С2 + … Сп.

Рис. Расчетные позиции элементов при различном включении.

Простая позиция, которая не требует преобразования №1 — последовательное подключение. По известной формуле для этих поз. запишем 1/Сэ = 1/С1 +1/С2 +1/С3,  подставив формулу значения, которые даны в условии задачи, получим 1/Сэ = 1/С1 +1/С2 +1/С3 = 59 мФ.

Не требует преобразования и 2 схема: емкость общего конденсатора будет равняться сумме конденсаторов которые включены в параллельной цепи: Сэ =С1 +С2 +С3 Сэ = 100 + 200 + 500 = 800 мФ.

Рассмотрев рис. №3 видно, что пара конденсаторов включена параллельно и один последовательно. Алгоритм преобразования таких цепей мы уже рассматривали, поэтому: сразу же находим емкость конденсатора Сэ соединения: Сэ = С1+С2 = 200+500 = 700 мФ.

Теперь находим общие эквивалентную емкость элементов с последовательным подключением 1/Сэ = 1/С2,3 +1/ С1 = 89 мф. Практическая задача решена.

Источник: http://themechanic.ru/posledovatelnoe-i-parallelnoe-soedinenie-rezistorov/

Соединение резисторов

Радиоэлектроника для начинающих

О том, как соединять конденсаторы и рассчитывать их общую ёмкость уже рассказывалось на страницах сайта. А как соединять резисторы и посчитать их общее сопротивление? Именно об этом и будет рассказано в этой статье.

Резисторы есть в любой электронной схеме, причём их номинальное сопротивление может отличаться не в 2 – 3 раза, а в десятки и сотни раз. Так в схеме можно найти резистор на 1 Ом, и тут же неподалёку на 1000 Ом (1 кОм)!

Поэтому при сборке схемы либо ремонте электронного прибора может потребоваться резистор с определённым номинальным сопротивлением, а под рукой такого нет. В результате быстро найти подходящий резистор с нужным номиналом не всегда удаётся. Это обстоятельство тормозит процесс сборки схемы или ремонта. Выходом из такой ситуации может быть применение составного резистора.

Для того чтобы собрать составной резистор нужно соединить несколько резисторов параллельно или последовательно и тем самым получить нужное нам номинальное сопротивление. На практике это пригождается постоянно.

Знания о правильном соединении резисторов и расчёте их общего сопротивления выручают и ремонтников, восстанавливающих неисправную электронику, и радиолюбителей, занятых сборкой своего электронного устройства.

Последовательное соединение резисторов

  • В жизни последовательное соединение резисторов имеет вид:
  • Последовательно соединённые резисторы серии МЛТ
  • Принципиальная схема последовательного соединения выглядит так:

На схеме видно, что мы заменяем один резистор на несколько, общее сопротивление которых равно тому, который нам необходим.

Подсчитать общее сопротивление при последовательном соединении очень просто. Нужно сложить все номинальные сопротивления резисторов входящих в эту цепь. Взгляните на формулу.

  1. Общее номинальное сопротивление составного резистора обозначено как Rобщ.
  2. Номинальные сопротивления резисторов включённых в цепь обозначаются как R1, R2, R3,…RN.
  3. Применяя последовательное соединение, стоит помнить одно простое правило:

Из всех резисторов, соединённых последовательно главную роль играет тот, у которого самое большое сопротивление. Именно он в значительной степени влияет на общее сопротивление.

Что это значит?

Так, например, если мы соединяем три резистора, номинал которых равен 1, 10 и 100 Ом, то в результате мы получим составной на 111 Ом.

Если убрать резистор на 100 Ом, то общее сопротивление цепочки резко уменьшиться до 11 Ом! А если убрать, к примеру, резистор на 10 Ом, то сопротивление будет уже 101 Ом.

Как видим, резисторы с малыми сопротивлениями в последовательной цепи практически не влияют на общее сопротивление.

Параллельное соединение резисторов

  • Можно соединять резисторы и параллельно:
  • Два резистора МЛТ-2, соединённых параллельно
  • Принципиальная схема параллельного соединения выглядит следующим образом:

Для того чтобы подсчитать общее сопротивление нескольких параллельно соединённых резисторов понадобиться знание формулы. Выглядит она вот так:

Эту формулу можно существенно упростить, если применять только два резистора. В таком случае формула примет вид:

Есть несколько простых правил, позволяющих без предварительного расчёта узнать, каково должно быть сопротивление двух резисторов, чтобы при их параллельном соединении получить то, которое требуется.

Если параллельно соединены два резистора с одинаковым сопротивлением, то общее сопротивление этих резисторов будет ровно в два раза меньше, чем сопротивление каждого из резисторов, входящих в эту цепочку.

Это правило исходит из простой формулы для расчёта общего сопротивления параллельной цепи, состоящей из резисторов одного номинала. Она очень проста. Нужно разделить номинальное сопротивление одного из резисторов на общее их количество:

Здесь R1 – номинальное сопротивление резистора. N – количество резисторов с одинаковым номинальным сопротивлением.

Ознакомившись с приведёнными формулами, вы скажите, что все они справедливы для расчёта ёмкости параллельно и последовательно соединённых конденсаторов. Да, только в отношении конденсаторов всё действует с точностью до «наоборот”. Узнать подробнее о соединении конденсаторов можно здесь.

Проверим справедливость показанных здесь формул на простом эксперименте.

Возьмём два резистора МЛТ-2 на 3 и 47 Ом и соединим их последовательно. Затем измерим общее сопротивление получившейся цепи цифровым мультиметром. Как видим оно равно сумме сопротивлений резисторов, входящих в эту цепочку.

  1. Замер общего сопротивления при последовательном соединении
  2. Теперь соединим наши резисторы параллельно и замерим их общее сопротивление.
  3. Измерение сопротивления при параллельном соединении
  4. Как видим, результирующее сопротивление (2,9 Ом) меньше самого меньшего (3 Ом), входящего в цепочку. Отсюда вытекает ещё одно известное правило, которое можно применять на практике:

При параллельном соединении резисторов общее сопротивление цепи будет меньше наименьшего сопротивления, входящего в эту цепь.

Что ещё нужно учитывать при соединении резисторов?

Во-первых, обязательно учитывается их номинальная мощность. Например, нам нужно подобрать замену резистору на 100 Ом и мощностью 1 Вт. Возьмём два резистора по 50 Ом каждый и соединим их последовательно. На какую мощность рассеяния должны быть рассчитаны эти два резистора?

Поскольку через последовательно соединённые резисторы течёт один и тот же постоянный ток (допустим 0,1 А), а сопротивление каждого из них равно 50 Ом, тогда мощность рассеивания каждого из них должна быть не менее 0,5 Вт. В результате на каждом из них выделится по 0,5 Вт мощности. В сумме это и будет тот самый 1 Вт.

Данный пример достаточно грубоват. Поэтому, если есть сомнения, стоит брать резисторы с запасом по мощности.

Подробнее о мощности рассеивания резистора читайте тут.

Во-вторых, при соединении стоит использовать однотипные резисторы, например, серии МЛТ. Конечно, нет ничего плохого в том, чтобы брать разные. Это лишь рекомендация.

Главная » Радиоэлектроника для начинающих » Текущая страница

Также Вам будет интересно узнать:

Источник: https://go-radio.ru/connection-of-resistors.html

Параллельное соединение резисторов

Господа, в прошлый раз мы с вами говорили про последовательное сопротивление резисторов. Сегодня я бы хотел вам рассказать про другой возможный вид соединения – параллельное.

Чем различается последовательное и параллельное соединение я уже писал в предыдущей статье.  Но все-таки вытащу сюда картинку из той прошлой статьи, я ж знаю, что вам будет лень ходить по ссылкам .

  • А) – Последовательное соединение
  • В) – Параллельное соединение
  • Рисунок 1 – Последовательное и параллельное соединение
  • Как мы видим из рисунка 1, параллельное соединение – это такое соединение, при котором одни концы всех резисторов соединены в один узел, а другие концы – в другой узел.

Сейчас наша задача будет разобраться, как ведут себя токи, напряжения, сопротивления и мощности при таком подключении. Для этого прошу вас взглянуть на рисунок 2, где подробно разрисован расклад дел для параллельного соединения. Будем полагать, что мы знаем величины R1, R2 и R3, а также величину приложенного к схеме напряжения U. Про токи же мы ничего не знаем.

Рисунок 2 – Параллельное соединения

Что мы видим на рисунке 2? Ну, в первую очередь – два узла А и B. В узел А сходятся одни концы всех резисторов, а в узел В – другие концы. Пусть узел А имеет потенциал φ1, а узел В – потенциал φ2. Из рисунка 2 видно, что для всех резисторов R1, R2 и R3 у нас одна и та же разность потенциалов U.

Как следует из статьи про потенциалы, это означает, что напряжение на всех резисторах у нас одинаково и равно приложенному напряжению U. Это важный вывод, его следует хорошо запомнить.

С токами дело обстоит по-другому. Проанализируем рисунок 2 слева направо. Пусть у нас в цепи течет ток I. Течет он себе, течет, никого не трогает и тут вдруг натыкается на узел А. Что в этом случае говорит полюбившаяся вам статья про первый закон Кирхгофа? А то, что ток I в узле А разделится на три тока I1, I2, I3. При этом будет выполняться равенство

То есть через резистор R1 будет протекать ток I1, через резистор R2 – ток I2, а через резистор R3 – ток I3.

Итак, у нас в системе уже тихо-мирно текут себе три тока. И все хорошо, пока они не наткнуться на узел В. Тут снова вступает в силу первый закон Кирхгофа. Эти три тока I1, I2, I3 вновь соединятся в один ток I. Причем после узла В ток будет иметь такую же величину I, какой он был до узла А.

То есть если все вышесказанное воплотить в лаконичный язык наскальной живописи, положение дел можно представить себе вот так

Как же найти эти самые токи I1, I2, I3? Господа, полагаю, вы уже догадались, что на помощь нам придет горячо нами всеми любимый закон Ома. Действительно, мы знаем сопротивления резисторов и, кроме того, нам известно, что на всех них падает одно и тоже напряжение U. Поэтому легко находим токи

Отлично, мы разобрались с напряжениями и с токами в такой схеме. А помните в статье про последовательное сопротивление мы ловко преобразовали три резистора в один с эквивалентным им сопротивлением? Нельзя ли и здесь сделать что-то подобное? Оказывается, вполне себе можно. Как мы помним, токи в схеме распределены таким вот образом

  1. Обзовем эквивалентное сопротивление буковкой R. И подставим в это выражение только что найденные нами токи I1, I2, I3
  2. Видим, что здесь без проблем можно сократить левую и правую части на U. Получаем
  3. Господа, важный вывод: при параллельном соединении резисторов обратное эквивалентное сопротивление равно сумме обратных сопротивлений отдельных резисторов.
  4. То есть для упрощения различных расчетов электрических схем такую вот цепочку параллельно соединенных резисторов можно заменить одним резистором с соответствующим сопротивлением, как показано на рисунке 3.
  5. Рисунок 3 – Преобразование параллельного соединение

Весьма частый случай на практике, когда соединены параллельно не много резисторов, а всего два. Поэтому полезно знать наизусть итоговое сопротивление такой схемы. Давайте посмотрим, чему оно равно:

То есть, если у вас два сопротивления соединены параллельно, то по этой формуле вы легко высчитаете общее сопротивление. Рассмотрим пример. Пусть у нас параллельно соединены два резистора 10 кОм и 15 кОм. Чему равно их общее сопротивление?

Заметьте, господа, итоговое сопротивление у нас получилось 6 кОм, что меньше 10 кОм и 15 кОм. То есть при параллельном соединении общее сопротивление меньше любого из составляющих.

Это всегда верно для любого количества резисторов, а не только для двух. Итоговое сопротивление всегда уменьшается (в отличии от последовательного сопротивления, где итоговое сопротивление всегда растет).

Этот факт полезно запомнить.

Еще один часто встречающийся на практике случай – когда параллельно соединены несколько резисторов с одинаковым сопротивлением. Допустим, каждый из них обладает сопротивлением R1 и всего их N штук. Тогда по нашей общей формуле для эквивалентного сопротивления

  • То есть при параллельном соединении N одинаковых резисторов с сопротивлением R1 итоговое сопротивление будет в N раз меньше этого самого сопротивления R1.
  • Так-с, с током разобрались, с напряжением разобрались, с эквивалентным сопротивлением вроде тоже…осталась мощность. Для этого воспользуемся вот этим выражением, которое мы писали чуть выше в статье
  • Умножим левую и правую части на напряжение U.
  • Как мы помним из статьи про мощность произведение тока на напряжение есть мощность. То есть мы можем записать
  • где Р – мощность, выдаваемая источником;
  • P1 – мощность, рассеиваемая на резисторе R1;
  • P2 – мощность, рассеиваемая на резисторе R2;
  • P3 – мощность, рассеиваемая на резисторе R3.

Заметьте, господа, формула в точности такая же, как и для случая последовательного соединения резисторов. И там и там мощность, выдаваемая источником, равна сумме мощностей, рассеиваемых на резисторах цепи.

Итак, господа, мы рассмотрели основные соотношения при параллельном соединении резисторов. Теперь осталось поговорить, где это параллельное соединение можно использовать и для чего.

1) Ну, во-первых, параллельное соединение применяют во всех случаях, когда хотят запитать несколько нагрузок от одного источника напряжения. При этом пользуются тем свойством, что при параллельном соединении напряжения на всех нагрузках одинаково.

То есть, допустим, вы берете источник напряжения, выставляете на нем напряжение 5 В и цепляете к этому источнику сразу несколько своих устройств. Узлами А и В в этом случае будут клеммы источника. На каждое из устройств в этом случае придет напряжение 5 В.

Да и все устройства в вашей квартире (лампочки, компьютеры, телевизоры и все прочее) соединены между собой параллельно.

2) Второе возможное применение встречается не так часто, но, думаю, о нем тоже следует рассказать. Допустим, вы делаете какую-то схему, где необходим очень точный подгон сопротивления. Скажем, надо получить сопротивление 6 кОм. Такое сопротивление найти нелегко, их просто не продают. Зато у вас есть два сопротивления 10 кОм и 15 кОм.

Вы их соединяете параллельно и получаете требуемые 6 кОм. Как показывает практика, 3 параллельных резисторов достаточно для получения итогового результирующего сопротивления требуемого номинала с весьма хорошей точностью. Конечно, таких вещей лучше избегать и, если есть возможность, всегда стараться применять стандартные сопротивления.

Но бывают случаи, когда это невозможно, и тогда приходит на помощь этот метод.

3) Третий пункт будет немного похож на первый. Его суть заключается в следующим. Допустим, нам надо снять с источника питания 10 Вт мощности. А у нас в наличии только резисторы, которые позволяют рассеивать на себе 1 Вт. Что делать? Можно соединить 10 резисторов параллельно и с каждого снимать по 1 Вт. Мы же помним нашу формулу

Конечно, лучше брать не 10 резисторов, а хотя бы 15 и рассеивать на них меньше, чем 1 Вт. Работать на пределе никогда не следует.

Кстати, тут очень вовремя к моменту написания статьи пришли платы с производства! Господа, прошу вас взглянуть на рисунок 4.

Рисунок 4 – Плата нагревателя

На нем изображена плата нагревателя (флешка для масштаба). В чем суть? Имеется весьма сложное устройство, предназначенное для работы в арктических условиях.

Найти же компоненты, которые надежно функционировать при температурах минус 55 градусов и при этом стоят адекватных денег и обладают адекватными размерами бывает непросто. Обычно элементная база в лучшем случае рассчитана на минус 40 градусов.

И было принято решение разработать вот такой вот нагреватель для прогрева чувствительных к холоду аналоговых узлов устройства. Он управляется с микроконтроллера и автоматически включается при температурах меньше минус 40 градусов.

Как вы можете видеть из рисунка 4, этот нагреватель представляет собой 30 параллельно соединенных резисторов с сопротивлениями 150 Ом. Каждый резистор, согласно документации, способен рассеивать до 1 Вт мощности. Используя изученные формулки, мы можем посчитать, что в сумме такая система обладает сопротивлением

  1. и теоретически может рассеивать мощность

Ну, с сопротивлением вопросов нет, оно действительно равно 5 Ом. Ну, плюс-минус 5 % на допуск резисторов, что в данном случае вообще не критично. А вот с мощностью тут не так все однозначно. Помните про закон Джоуля-Ленца, который мы рассматривали? Резисторы будут греться, причем не слабо.

Как показывает практика, если нагружать резисторы по полной, то есть рассеивать на каждом по 1 Вт, то в течении нескольких секунд их температура улетит за 150 градусов. Такая высокая температура критична для резистора и может привести к его разрушению.

Я был готов к такому развитию событий, поэтому заложил для платы нагревателя максимальное напряжение 9 вольт. Это значит, что на каждом резисторе будет выделяться

  • что почти в два раза меньше максимально допустимой мощности в 1 Вт. В сумме на всей плате выделялось, соответственно

Эксперимент показал, что резисторы достигли температуры с комнатных 25 градусов до критичных 120 градусов приблизительно за 10 секунд работы и температура продолжала уверенно расти.

Очевидно, если оставить на длительное время включенным такой нагреватель при комнатной температуре, он неминуемо выйдет из строя.

Возможно, при работе на минус 55 градусах перегрев бы не был столь критичным, однако хотелось исключить вариант спалить плату на столе, поэтому я понизил напряжение, подаваемое на плату на 3 вольта: стал подавать 6 вольт. Теперь на каждом резисторе рассеивалось

  1. а на всей плате

Теперь температура поднималась до 100-110 градусов примерно за 30-40 секунд работы и оставалась на этом уровне (выходила в точку термодинамического равновесия). Эта температура вполне подходит для нагревателя.

Однако пока это были лишь эксперименты на столе при комнатной температуре, главный эксперимент – в термокамере на минус 55 градусах – впереди. Возможно, по его результатам потребуется чуть увеличить рассеиваемую мощность.

А может все останется как есть и этой мощности будет достаточно для вывода девайса на режим за адекватное время, время покажет .

На сегодня все, господа. Удачи вам и до новых встреч!

Источник: http://myelectronix.ru/postoyannyy-tok/40-parallelnoe-soedinenie-rezistorov

Как отличается параллельное и последовательное соединение резисторов?

Большое разнообразие схем основано на двух видах соединений – последовательное параллельное. Для каждого типа существуют свои собственные законы и принципы. Именно это и позволяет создавать устройства с самыми различными техническими параметрами, в том числе и резисторы. Что же такое резистор?

Резистор – радиодеталь, созданная для контроля напряжения и тока в цепи, увеличивая либо понижая его. Резисторы могут быть двух видов – постоянные и переменные. Так, например, светодиоды требуют для себя совсем небольшого тока. Для этого в электрическую цепочку перед светодиодом устанавливается резистор, который обеспечивает необходимое напряжение для работы последнего.

В статье подробны рассмотрены все аспекты последовательного и параллельного подключения резисторов. Бонусом к статье являются видеоролик и детальная информационная статья на рассматриваемую тему.

Последовательное подключение

Начнем с последовательного соединения. По этой схеме каждый резистор подключается с другим только в одной точке, их может быть в цепи 2, 3 и больше. Обозначим сопротивления: R1, R2, R3 и напряжение источника в цепи Uц. При подключении источника питания в ней начнет протекать ток Iц. В цепи с последовательным соединением ток протекает по всем резисторам один за другим.

Поскольку ток течет через все резисторы их сопротивления и ток суммируется, Iц = I1+I2+I3, Rц = R1 +R2 + R3, чем больше отдельно взятое сопротивление, тем тяжелее электронам преодолевать участок цепи. Мощность резисторов при последовательном и параллельном соединении рассчитывается по разным формулам. В последовательных цепях — складываем, в параллельных — это обратно пропорциональная величина.

Последовательное соединение характеризуется тем, что элементы идут друг за другом. Конец одного подключается к началу другого. При подключении полученной цепочки к источнику тока получается кольцо.

Теоретическая часть

Последовательное соединение характерно тем, что через все элементы протекает ток одинаковой силы. То есть, если цепочка состоит из двух резисторов R1 и R2 (как на рисунке ниже), то ток протекающий через каждое из них и любую другую часть цепи будет одинаковой (I = I1 = I2).

Суммарное сопротивление всей цепи последовательно соединенных резисторов считается как сумма сопротивлений всех ее элементов. То есть, номиналы складывают. R = R1 + R2 — это и есть формула расчета сопротивления при последовательном соединении резисторов. Если элементов больше двух, будет просто больше слагаемых.

Еще одно свойство последовательного соединения — на каждом элементе напряжение отличается. Ток в цепи одинаковый, а напряжение на резисторе зависит от его номинала.

Последовательное подключение.

Примеры расчета

Давайте рассмотрим пример. Цепь представлена на рисунке выше. Есть источник тока и два сопротивления. Пусть R1=1,2 кОм, R2= 800 Ом, а ток в цепи 2 А. По закону Ома U = I * R. Подставляем наши значения:

  • U1 = R1 * I = 1200 Ом * 2 А = 2400 В;
  • U2 = R2 * I = 800 Ом * 2А = 1600 В.

Будет интересно➡  SMD резисторы: что это такое и для чего используются?

Общее напряжение цепи считается как сумма напряжений на резисторах: U = U1 + U2 = 2400 В + 1600 В = 4000 В. Полученную цифру можно проверить. Для этого найдем суммарное сопротивление цепи и умножим его на ток.   R = R1 + R2 = 1200 Ом + 800 Ом = 2000 Ом.

Если подставить в формулу напряжения при последовательном соединении сопротивлений, получаем: U = R * I = 2000 Ом * 2  А = 4000 В. Получаем, что общее напряжение данной цепи 4000 В.

А теперь посмотрите на схему. На первом вольтметре (возле резистора R1) показания будут 2400 В, на втором  — 1600 В.  При этом напряжение источника питания — 4000 В. Последовательное соединение – это соединение двух или более резисторов в форме цепи, в которой каждый отдельный резистор соединяется с другим отдельным резистором только в одной точке.

Материал по теме: Как проверить варистор мультиметром.

Общее сопротивление Rобщ

При таком соединении, через все резисторы проходит один и тот же электрический ток. Чем больше элементов на данном участке электрической цепи, тем «труднее» току протекать через него. Следовательно, при последовательном соединении резисторов их общее сопротивление увеличивается, и оно равно сумме всех сопротивлений.

Параллельное соединение резисторов

Параллельное соединение – это соединение, при котором резисторы соединяются между собой обоими контактами. В результате к одной точке (электрическому узлу) может быть присоединено несколько резисторов.

Параллельное подключение резисторов.

Общее сопротивление Rобщ

При таком соединении, через каждый резистор потечет отдельный ток. Сила данного тока будет обратно пропорциональна сопротивлению резистора. В результате общая проводимость такого участка электрической цепи увеличивается, а общее сопротивление в свою очередь уменьшается.

Таким образом, при параллельном подсоединении резисторов с разным сопротивлением, общее сопротивление будет всегда меньше значения самого маленького отдельного резистора.

Если посмотреть на изображение параллельного соединения, заметно, что ко всем элементам прилагается одинаковое напряжение.

То есть, при параллельном соединении резисторов, на каждом из них будет одинаковое напряжение U = U1 = U2 = U3. Получается, что ток разделяется на несколько «ручейков».

То есть, при параллельном соединении резисторов сила тока, протекающего через каждый из элементов, отличается. I = I1+I2+I3. И зависит сила тока (согласно тому же закону Ома) от сопротивления каждого участка цепи.

В случае с параллельным соединением резисторов — от их номинала.

Предлагаем также почитать интересный материал про малоизвестные факты о двигателях постоянного тока в другой нашей статье.

Схема параллельного соединения

Общее сопротивление участка цепи при таком соединении становится ниже. Его высчитывают по формуле: 1/R = 1/R1 + 1/R + 1/R3+. Такая форма хоть и понятна, но неудобна.

Формула расчета сопротивления параллельно подключенных резисторов получается тем сложнее, чем больше элементов соединены параллельно.

Но больше двух-трех редко кто объединяет, так что на практике достаточно знать только две формулы приведенные ниже.

Если подставить значения в эти формулы, то заметим, что результат будет меньше, чем сопротивление резистора с наименьшим номиналом. Это стоит запомнить: результирующее сопротивление включенных параллельно резисторов будет ниже самого маленького номинала. Давайте сначала рассчитаем параллельное соединение двух резисторов разного номинала и посмотрим что получится.

Соединили параллельно 150 Ом и 100 Ом. Считаем результирующее: 150*100 / (150+100) = 15000/250 = 60 Ом. Если соединить 150 Ом и 50 Ом, получим: 150*50 / (150+50) = 7500 / 200 = 37,5 Ом.

 Как видим, в обоих случаях результат оказывается меньше чем самый низкий номинал соединенных деталей. Этим и пользуются, если в наличии нет сопротивления небольшого номинала.

Проблема только в том, что подбирать сложновато: надо каждый раз считать используя калькулятор.

Как высчитывать сопротивление составных резисторов

Возможно, вам будет проще, если знать, что соединив два одинаковых резистора параллельно, получим результат в два раза меньше. Например, соединив параллельно два резистора по 100 Ом получим составное сопротивление 50 Ом. Проверим? Считаем: 100*100 / (100+100) = 10000 / 200 = 50 Ом. При соединении параллельно трех резисторов, считать приходится больше, так как формула сложнее.

Если подключить параллельно 150 Ом, 100 Ом и 50 Ом, результирующее будет 27,3 Ом. Попробуем с более низкими номиналами. Если параллельно включены 20 Ом, 15 Ом и 10 Ом. Получим результирующее сопротивление 4,61 Ом. Вот вам подтверждение правила. Суммарное сопротивление параллельно соединенных резисторов меньше чем самый низкий номинал.

Параллельное соединение резисторов

Параллельное соединение резисторов это соединение, в котором начала всех резисторов соединены в одну общую точку (А), а концы в другую общую точку. При этом по каждому резистору течет свой ток. При параллельном соединении при протекании тока из точки А в точку Б, он имеет несколько путей.

 Таким образом, увеличение числа параллельно соединенных резисторов ведет к увеличению путей протекания тока, то есть к уменьшению противодействия протеканию тока. А это значит, чем большее количество резисторов соединить параллельно, тем меньше станет значение общего сопротивления такого участка цепи.

Общее сопротивление параллельно соединенных резисторов определяется следующим отношением: 1/Rобщ= 1/R1+1/R2+1/R3+…+1/Rn.

Формулы расчета параллельного и последовательного подключения.

Следует отметить, что здесь действует правило «меньше – меньшего». Это означает, что общее сопротивление всегда будет меньше сопротивления любого параллельно включенного резистора. Общее сопротивление для двух параллельно соединенных резисторов рассчитывается по следующей формуле: Rобщ= R1*R2/R1+R2.

Если имеет место два параллельно соединенных резистора с одинаковыми сопротивлениями, то их общее сопротивление будет равно половине сопротивления одного из них. Параллельное соединение резисторов. При параллельном соединении резисторов нескольких приемников они включаются между двумя точками электрической цепи, образуя параллельные ветви.

Заменяя лампы резисторами с сопротивлениями R1, R2, R3, При параллельном соединении ко всем резисторам приложено одинаковое напряжение U. Поэтому согласно закону Ома: I1=U/R1; I2=U/R2; I3=U/R3.

Источник: https://ElectroInfo.net/radiodetali/rezistory/kak-otlichaetsja-parallelnoe-i-posledovatelnoe-soedinenie-rezistorov.html

Последовательное и параллельное соединение проводников — урок. Физика, 8 класс.

В быту и в промышленности в электрическую цепь соединяются сразу несколько потребителей электрической энергии. Различают три вида соединения сопротивлений (резисторов):

  1. последовательное соединение проводников
  2. параллельное соединение проводников
  3. смешанное соединение проводников

Последовательное соединение проводников

Схема соединения выглядит следующим образом:

 

 

Обрати внимание!

При последовательном соединении все входящие в него проводники соединяются друг за другом, т.е. конец первого проводника соединяется с началом второго.

 

 

Опыт показывает:

Сила тока в любых частях цепи одна и та же (об этом свидетельствуют показания амперметров): I=I1=I2.

Если выкрутить одну лампу, то цепь разомкнётся, а другая лампа тоже погаснет.

 

Опыт показывает следующее: 

При последовательном соединении сопротивлений результирующее напряжение равно сумме напряжений на участках: U=U1&plus;U2.

 

 

Результирующее сопротивление последовательно соединённых потребителей равно сумме сопротивлений потребителей: R=R1&plus;R2.

Для проверки данного утверждения можно использовать омметр. При подключении омметра ключ должен быть разомкнут!

 

Омметр подключают по очереди к каждому потребителю, а потом к обоим одновременно.

 

Сопротивление цепи \(R\), состоящей из \(n\) одинаковых ламп, сопротивлением R1 каждая, в \(n\) раз больше сопротивления одной лампы: \(R\) = R1* \(n\).

 

Параллельное соединение проводников

Схема соединения выглядит следующим образом:

 

 

Обрати внимание!

При параллельном соединении все входящие в него проводники одним своим концом присоединяются к одной точке цепи А, а вторым концом — к другой точке В.

 

 

Опыт доказывает:

Сила тока в неразветвлённой части цепи равна сумме сил тока в отдельных параллельно соединённых проводниках.

Об этом свидетельствуют показания амперметров: I=I1+I2.

 

 

Если выкрутить одну лампу, то другая лампа продолжает гореть. Это свойство используют для подключения бытовых приборов в помещении.

 

Опыт свидетельствует, что:

Напряжение на участке цепи АВ и на концах всех параллельно соединённых проводников одно и то же.

Об этом свидетельствуют показания вольтметров:

U=U1=U2.

 

 

Общее сопротивление цепи при параллельном соединении проводников определяется по формуле:

 

1R=1R1&plus;1R2.

 

Обратное значение общего сопротивления равно сумме обратных значений сопротивлений отдельных проводников.

Для проверки формулы можно использовать омметр. При подключении омметра ключ должен быть разомкнут!

 

Сопротивление цепи \(R\), состоящей из \(n\) одинаковых ламп, сопротивлением R1 каждая, в \(n\) раз меньше сопротивления одной лампы: \(R\) = R1/ \(n\).

Источники:

http://files.school-collection.edu.ru/dlrstore/669ba073-e921-11dc-95ff-0800200c9a66/3_17.swf
http://files.school-collection.edu.ru/dlrstore/669ba074-e921-11dc-95ff-0800200c9a66/3_18.swf

http://class-fizika.narod.ru/8_33.htm

Формула сопротивления тока при параллельном соединении

Сопротивление проводников. Параллельное и последовательное соединение проводников.

Электри́ческое сопротивле́ние — физическая величина, характеризующая свойства проводника препятствовать прохождению электрического тока и равная отношениюнапряжения на концах проводника к силе тока, протекающего по нему [1] . Сопротивление для цепей переменного тока и для переменных электромагнитных полей описывается понятиями импеданса и волнового сопротивления. Сопротивлением (резистором) также называют радиодеталь, предназначенную для введения в электрические цепи активного сопротивления.

Сопротивление (часто обозначается буквой R или r) считается, в определённых пределах, постоянной величиной для данного проводника; её можно рассчитать как

U — разность электрических потенциалов (напряжение) на концах проводника;

I — сила тока, протекающего между концами проводника под действием разности потенциалов.

При последовательном соединении проводников (рис. 1.9.1) сила тока во всех проводниках одинакова:

Последовательное соединение проводников

По закону Ома, напряжения U1 и U2 на проводниках равны

Общее напряжение U на обоих проводниках равно сумме напряжений U1 и U2:

где R – электрическое сопротивление всей цепи. Отсюда следует:

При последовательном соединении полное сопротивление цепи равно сумме сопротивлений отдельных проводников.

Этот результат справедлив для любого числа последовательно соединенных проводников.

При параллельном соединении (рис. 1.9.2) напряжения U1 и U2 на обоих проводниках одинаковы:

Сумма токов I1 + I2, протекающих по обоим проводникам, равна току в неразветвленной цепи:

Этот результат следует из того, что в точках разветвления токов (узлы A и B) в цепи постоянного тока не могут накапливаться заряды. Например, к узлу A за время Δt подтекает заряд IΔt, а утекает от узла за то же время заряд I1Δt + I2Δt. Следовательно,I = I1 + I2.

Параллельное соединение проводников

Записывая на основании закона Ома

где R – электрическое сопротивление всей цепи, получим

При параллельном соединении проводников величина, обратная общему сопротивлению цепи, равна сумме величин, обратных сопротивлениям параллельно включенных проводников.

Этот результат справедлив для любого числа параллельно включенных проводников.

Формулы для последовательного и параллельного соединения проводников позволяют во многих случаях рассчитывать сопротивление сложной цепи, состоящей из многих резисторов. На рис. 1.9.3 приведен пример такой сложной цепи и указана последовательность вычислений.

Расчет сопротивления сложной цепи. Сопротивления всех проводников указаны вомах (Ом)

Следует отметить, что далеко не все сложные цепи, состоящие из проводников с различными сопротивлениями, могут быть рассчитаны с помощью формул для последовательного и параллельного соединения. На рис. 1.9.4 приведен пример электрической цепи, которую нельзя рассчитать указанным выше методом.

Пример электрической цепи, которая не сводится к комбинации последовательно и параллельно соединенных проводников

Проверим справедливость показанных здесь формул на простом эксперименте.

Возьмём два резистора МЛТ-2 на 3 и 47 Ом и соединим их последовательно. Затем измерим общее сопротивление получившейся цепи цифровым мультиметром. Как видим оно равно сумме сопротивлений резисторов, входящих в эту цепочку.

Замер общего сопротивления при последовательном соединении

Теперь соединим наши резисторы параллельно и замерим их общее сопротивление.

Измерение сопротивления при параллельном соединении

Как видим, результирующее сопротивление (2,9 Ом) меньше самого меньшего (3 Ом), входящего в цепочку. Отсюда вытекает ещё одно известное правило, которое можно применять на практике:

При параллельном соединении резисторов общее сопротивление цепи будет меньше наименьшего сопротивления, входящего в эту цепь.

Что ещё нужно учитывать при соединении резисторов?

Во-первых, обязательно учитывается их номинальная мощность. Например, нам нужно подобрать замену резистору на 100 Ом и мощностью 1 Вт . Возьмём два резистора по 50 Ом каждый и соединим их последовательно. На какую мощность рассеяния должны быть рассчитаны эти два резистора?

Поскольку через последовательно соединённые резисторы течёт один и тот же постоянный ток (допустим 0,1 А ), а сопротивление каждого из них равно 50 Ом , тогда мощность рассеивания каждого из них должна быть не менее 0,5 Вт . В результате на каждом из них выделится по 0,5 Вт мощности. В сумме это и будет тот самый 1 Вт .

Данный пример достаточно грубоват. Поэтому, если есть сомнения, стоит брать резисторы с запасом по мощности.

Подробнее о мощности рассеивания резистора читайте .

Во-вторых, при соединении стоит использовать однотипные резисторы, например, серии МЛТ. Конечно, нет ничего плохого в том, чтобы брать разные. Это лишь рекомендация.

Параллельное соединение резисторов — одно из двух видов электрических соединений, когда оба вывода одного резистора соединены с соответствующими выводами другого резистора или резисторов. Зачастую или параллельно для того, чтобы создать более сложные электронные схемы.

Схема параллельного соединения показан на рисунке ниже. При параллельном соединении резисторов, напряжение на всех резисторах будет одинаковым, а протекающий через них ток будет пропорционален их сопротивлению:

Формула параллельного соединения резисторов

Общее сопротивление нескольких резисторов соединенных параллельно определяется по следующей формуле:

Ток, протекающий через отдельно взятый резистор, согласно , можно найти по формуле:

Параллельное соединение резисторов — расчет

Пример №1

При разработке устройства, возникла необходимость установить резистор с сопротивлением 8 Ом. Если мы просмотрим весь номинальный ряд стандартных значений резисторов, то мы увидим, что резистора с сопротивлением в 8 Ом в нем нет.

Выходом из данной ситуации будет использование двух параллельно соединенных резисторов. Эквивалентное значение сопротивления для двух резисторов соединенных параллельно рассчитывается следующим образом:

Данное уравнение показывает, что если R1 равен R2, то сопротивление R составляет половину сопротивления одного из двух резисторов. При R = 8 Ом, R1 и R2 должны, следовательно, иметь значение 2 × 8 = 16 Ом.
Теперь проведем проверку, рассчитав общее сопротивление двух резисторов:

Таким образом, мы получили необходимое сопротивление 8 Ом, соединив параллельно два резистора по 16 Ом.

Пример расчета №2

Найти общее сопротивление R из трех параллельно соединенных резисторов:

Общее сопротивление R рассчитывается по формуле:

Этот метод расчета может быть использованы для расчета любого количества отдельных сопротивлений соединенных параллельно.

Один важный момент, который необходимо запомнить при расчете параллельно соединенных резисторов – это то, что общее сопротивление всегда будет меньше, чем значение наименьшего сопротивления в этой комбинации.

Как рассчитать сложные схемы соединения резисторов

Более сложные соединения резисторов могут быть рассчитаны путем систематической группировки резисторов. На рисунке ниже необходимо посчитать общее сопротивление цепи, состоящей из трех резисторов:

Для простоты расчета, сначала сгруппируем резисторы по параллельному и последовательному типу соединения.
Резисторы R2 и R3 соединены последовательно (группа 2). Они в свою очередь соединены параллельно с резистором R1 (группа 1).

Последовательное соединение резисторов группы 2 вычисляется как сумма сопротивлений R2 и R3:

В результате мы упрощаем схему в виде двух параллельных резисторов. Теперь общее сопротивление всей схемы можно посчитать следующим образом:

Расчет более сложных соединений резисторов можно выполнить используя законы Кирхгофа.

Ток, протекающий в цепи параллельно соединенных резисторах

Общий ток I протекающий в цепи параллельных резисторов равняется сумме отдельных токов, протекающих во всех параллельных ветвях, причем ток в отдельно взятой ветви не обязательно должен быть равен току в соседних ветвях.

Несмотря на параллельное соединение, к каждому резистору приложено одно и то же напряжение. А поскольку величина сопротивлений в параллельной цепи может быть разной, то и величина протекающего тока через каждый резистор тоже будет отличаться (по определению закона Ома).

Рассмотрим это на примере двух параллельно соединенных резисторов. Ток, который течет через каждый из резисторов (I1 и I2) будет отличаться друг от друга поскольку сопротивления резисторов R1 и R2 не равны.
Однако мы знаем, что ток, который поступает в цепь в точке «А» должен выйти из цепи в точке «B» .

Первое правило Кирхгофа гласит: «Общий ток, выходящий из цепи равен току входящий в цепь».

Таким образом, протекающий общий ток в цепи можно определить как:

Затем с помощью закона Ома можно вычислить ток, который протекает через каждый резистор:

Ток, протекающий в R1 = U ÷ R1 = 12 ÷ 22 кОм = 0,545 мА

Ток, протекающий в R 2 = U ÷ R2 = 12 ÷ 47 кОм = 0,255 мА

Таким образом, общий ток будет равен:

I = 0,545 мА + 0,255 мА = 0,8 мА

Это также можно проверить, используя закон Ома:

I = U ÷ R = 12 В ÷ 15 кОм = 0,8 мА (то же самое)

где 15кОм — это общее сопротивление двух параллельно соединенных резисторов (22 кОм и 47 кОм)

И в завершении хочется отметить, что большинство современных резисторов маркируются цветными полосками и назначение ее можно узнать .

Параллельное соединение резисторов — онлайн калькулятор

Чтобы быстро вычислить общее сопротивление двух и более резисторов, соединенных параллельно, вы можете воспользоваться следующим онлайн калькулятором:

Подведем итог

Когда два или более резистора соединены так, что оба вывода одного резистора соединены с соответствующими выводами другого резистора или резисторов, то говорят, что они соединены между собой параллельно. Напряжение на каждом резисторе внутри параллельной комбинации одинаковое, но токи, протекающие через них, могут отличаться друг от друга, в зависимости от величины сопротивлений каждого резистора.

Эквивалентное или полное сопротивление параллельной комбинации всегда будет меньше минимального сопротивления резистора входящего в параллельное соединение.

На практике нередко встречается задача нахождения сопротивления проводников и резисторов при различных способах соединения. В статье рассмотрено, как рассчитывается сопротивление при и некоторые другие технические вопросы.

Сопротивление проводника

Все проводники имеют свойство препятствовать течению электрического тока, его принято называть электрическим сопротивлением R, оно измеряется в омах. Это основное свойство проводниковых материалов.

Для ведения электротехнических расчётов применяется удельное сопротивление – ρ Ом·м/мм 2 . Все металлы – хорошие проводники, наибольшее применение получили медь и алюминий, гораздо реже применяется железо. Лучший проводник – серебро, оно применяется в электротехнической и электронной промышленности. Широко распространены сплавы с высоким значением сопротивления.

При расчёте сопротивления используется известная из школьного курса физики формула:

R = ρ · l/S, S – площадь сечения; l – длина.

Если взять два проводника, то их сопротивление при параллельном соединении станет меньше из-за увеличения общего сечения.

и нагрев проводника

Для практических расчётов режимов работы проводников применяется понятие плотности тока – δ А/мм 2 , она вычисляется по формуле:

δ = I/S, I – ток, S – сечение.

Ток, проходя по проводнику, нагревает его. Чем больше δ, тем сильнее нагревается проводник. Для проводов и кабелей разработаны нормы допустимой плотности, которые приводятся в Для проводников нагревательных устройств существуют свои нормы плотности тока.

Если плотность δ выше допустимой, может произойти разрушение проводника, например, при перегреве кабеля у него разрушается изоляция.

Правилами регламентируется производить расчёт проводников на нагрев.

Способы соединения проводников

Любой проводник гораздо удобнее изображать на схемах как электрическое сопротивление R, тогда их легко читать и анализировать. Существует всего три способа соединения сопротивлений. Первый способ самый простой – последовательное соединение.

На фото видно, что полное сопротивление равно: R = R 1 + R 2 + R 3 .

Второй способ более сложный – параллельное соединение. Расчёт сопротивления при параллельном соединении выполняется поэтапно. Рассчитывается полная проводимость G = 1/R, а затем полное сопротивление R = 1/G.

Можно поступить и по-другому, прежде рассчитать общее сопротивление при R1 и R2, после этого повторить операцию и найти R.

Третий способ соединения наиболее сложный – смешанное соединение, то есть присутствуют все рассмотренные варианты. Схема приведена на фото.

Для расчёта этой схемы её следует упростить, для этого заменяют резисторы R2 и R3 одним R2,3. Получается несложная схема.

R2,3,4 = R2,3 · R4/(R2,3 + R4).

Схема становится ещё проще, в ней остаются резисторы, имеющие последовательное соединение. В более сложных ситуациях используется этот же метод преобразования.

Виды проводников

В электронной технике, при производстве проводники представляют собою тонкие полоски медной фольги. Ввиду малой длины сопротивление у них незначительно, им во многих случаях можно пренебречь. Для этих проводников сопротивление при параллельном соединении уменьшается вследствие увеличения сечения.

Большой раздел проводников представляют обмоточные провода. Они выпускаются разных диаметров – от 0,02 до 5,6 миллиметра. Для мощных трансформаторов и электродвигателей выпускаются медные шинки прямоугольного сечения. Иногда при ремонте заменяют провод большого диаметра на несколько параллельно соединённых меньшего размера.

Особый раздел проводников представляют провода и кабели, промышленность предоставляет широчайший выбор марок для самых различных нужд. Нередко приходится заменять один кабель на несколько, меньшего сечения. Причины этого бывают самые различные, например, кабель сечением 240 мм 2 очень трудно прокладывать по трассе с крутыми изгибами. Его заменяют на 2×120 мм 2 , и проблема решена.

Расчёт проводов на нагрев

Проводник нагревается протекающим током, если его температура превысит допустимую, наступает разрушение изоляции. ПУЭ предусматривает расчёт проводников на нагрев, исходными данными для него являются сила тока и условия внешней среды, в которой проложен проводник. По этим данным из таблиц в ПУЭ выбирается рекомендуемое проводника или кабеля).

На практике встречаются ситуации, когда нагрузка на действующий кабель сильно возросла. Существует два выхода ‒ заменить кабель на другой, это бывает дорого, или параллельно ему проложить ещё один, чтобы разгрузить основной кабель. В этом случае сопротивление проводника при параллельном соединении уменьшается, следовательно падает выделение тепла.

Чтобы правильно выбрать сечение второго кабеля, пользуются таблицами ПУЭ, важно при этом не ошибиться с определением его рабочего тока. В этой ситуации охлаждение кабелей будет даже лучше, чем у одного. Рекомендуется рассчитать сопротивление при параллельном соединении двух кабелей, чтобы точнее определить их тепловыделение.

Расчёт проводников на потерю напряжения

При расположении потребителя R н на большом расстоянии L от источника энергии U 1 возникает довольно большое на проводах линии. К потребителю R н поступает напряжение U 2 значительно ниже начального U 1 . Практически в качестве нагрузки выступает различное электрооборудование, подключаемое к линии параллельно.

Для решения проблемы производят расчет сопротивления при параллельном соединении всего оборудования, так находится сопротивление нагрузки R н. Далее следует определить сопротивление проводов линии.

Здесь S – сечение провода линии, мм 2 .

Каждый в этой жизни сталкивался с резисторами. Люди с гуманитарными профессиями, как и все, изучали в школе на уроках физики проводники электрического тока и закон Ома.

С резисторами также имеют дело студенты технических университетов и инженеры различных производственных предприятий. Перед всеми этими людьми, так или иначе, вставала задача расчёта электрической цепи при различных видах соединения резисторов. В данной статье речь пойдёт о расчёте физических параметров, характеризующих цепь.

Виды соединений

Резистор – пассивный элемент , присутствующий в каждой электрической цепи. Он предназначен для того, чтобы сопротивляться электрическому току. Существует два вида резисторов:

Зачем же спаивать проводники друг с другом? Например, если для какой-то электрической цепи нужно определённое сопротивление. А среди номинальных показателей нужного нет. В таком случае необходимо подобрать элементы схемы с определёнными значениями сопротивления и соединить их. В зависимости от вида соединения и сопротивлений пассивных элементов мы получим какое-то определённое сопротивление цепи. Оно называется эквивалентным. Его значение зависит от вида спайки проводников. Существует три вида соединения проводников:

Значение эквивалентного сопротивления в цепи считается достаточно легко. Однако, если резисторов в схеме очень много, то лучше воспользоваться специальным калькулятором, который считает это значение. При ведении расчёта вручную, чтобы не допускать ошибок, необходимо проверять, ту ли формулу вы взяли.

Последовательное соединение проводников

В последовательной спайке резисторы идут как бы друг за другом. Значение эквивалентного сопротивления цепи равно сумме сопротивлений всех резисторов. Особенность схем с такой спайкой заключается в том, что значение тока постоянно . Согласно закону Ома, напряжение в цепи равно произведению тока и сопротивления. Так как ток постоянен, то для вычисления напряжения на каждом резисторе, достаточно перемножить значения. После этого необходимо сложить напряжения всех резисторов, и тогда мы получим значение напряжения во всей цепи.

Расчёт очень простой. Так как с ним имеют дело в основном инженеры-разработчики, то для них не составит труда сосчитать всё вручную. Но если резисторов очень много, то проще воспользоваться специальным калькулятором.

Примером последовательного соединения проводников в быту является ёлочная гирлянда.

Параллельное соединение резисторов

При параллельном соединении проводников эквивалентное сопротивление в цепи считается по-другому. Немного сложнее, чем при последовательном.

Его значение в таких цепях равняется произведению сопротивлений всех резисторов, делённому на их сумму. А также есть и другие варианты этой формулы. Параллельное соединение резисторов всегда снижает эквивалентное сопротивление цепи. То есть, его значение всегда будет меньше, чем наибольшее значение какого-то из проводников.

В таких схемах значение напряжения постоянно . То есть значение напряжения во всей цепи равно значениям напряжений каждого из проводников. Оно задаётся источником напряжения.

Сила тока в цепи равна сумме всех токов, протекающих через все проводники. Значение силы тока, протекающего через проводник. равно отношению напряжения источника к сопротивлению этого проводника.

Примеры параллельного соединения проводников:

  1. Освещение.
  2. Розетки в квартире.
  3. Производственное оборудование.

Для расчёта схем с параллельным соединением проводников лучше пользоваться специальным калькулятором. Если в схеме много резисторов, спаянных параллельно, то гораздо быстрее вы посчитаете эквивалентное сопротивление с помощью этого калькулятора.

Смешанное соединение проводников

Этот вид соединения состоит из каскадов резисторов . Например, у нас есть каскад из 10 проводников, соединённых последовательно, и после него идёт каскад из 10 проводников, соединённых параллельно. Эквивалентное сопротивление этой схемы будет равно сумме эквивалентных сопротивлений этих каскадов. То есть, по сути, здесь последовательное соединение двух каскадов проводников.

Многие инженеры занимаются оптимизацией различных схем. Её целью является уменьшение количества элементов в схеме за счёт подбора других, с подходящими значениями сопротивлений. Сложные схемы разбиваются на несколько небольших каскадов, ведь так гораздо проще вести расчёты.

Сейчас, в двадцать первом веке, инженерам стало гораздо проще работать. Ведь несколько десятилетий назад все расчёты производились вручную. А сейчас программисты разработали специальный калькулятор для расчёта эквивалентного сопротивления цепи. В нём запрограммированы формулы, по которым ведутся расчёты.

В этом калькуляторе можно выбрать вид соединения, и потом ввести в специальные поля значения сопротивлений. Через несколько секунд вы уже увидите это значение.

Как я и обещал в статье про переменные резисторы (ссылка), сегодня речь пойдет о возможных способах соединения резисторов, в частности о последовательном соединении и о параллельном.

Последовательное соединение резисторов.

Давайте начнем с рассмотрения цепей, элементы которой соединены последовательно. И хоть мы и будем рассматривать только резисторы в качестве элементов цепи в данной статье, но правила, касающиеся напряжений и токов при разных соединениях будут справедливы и для других элементов. Итак, первая цепь, которую мы будем разбирать выглядит следующим образом:

Здесь у нас классический случай последовательного соединения – два последовательно включенных резистора. Но не будем забегать вперед и рассчитывать общее сопротивление цепи, а для начала рассмотрим все напряжения и токи. Итак, первое правило заключается в том, что протекающие по всем проводникам токи при последовательном соединении равны между собой:

А для определения общего напряжения при последовательном соединении, напряжения на отдельных элементах необходимо просуммировать:

В то же время, по закону Ома для напряжений, сопротивлений и токов в данной цепи справедливы следующие соотношения:

Тогда для вычисления общего напряжения можно будет использовать следующее выражение:

Но для общего напряжение также справедлив закон Ома:

Здесь – это общее сопротивление цепи, которое исходя из двух формул для общего напряжения равно:

Таким образом, при последовательном соединении резисторов общее сопротивление цепи будет равно сумме сопротивлений всех проводников.

Например для следующей цепи:

Общее сопротивление будет равно:

Количество элементов значения не имеет, правило, по которому мы определяем общее сопротивление будем работать в любом случае &#128578; А если при последовательном соединении все сопротивления равны (), то общее сопротивление цепи составит:

в данной формуле равно количеству элементов цепи.

С последовательным соединением резисторов мы разобрались, давайте перейдем к параллельному.

Параллельное соединение резисторов.

При параллельном соединении напряжения на проводниках равны:

А для токов справедливо следующее выражение:

То есть общий ток разветвляется на две составляющие, а его значение равно сумме всех составляющих. По закону Ома:

Подставим эти выражения в формулу общего тока:

А по закону Ома ток:

Приравниваем эти выражения и получаем формулу для общего сопротивления цепи:

Данную формулу можно записать и несколько иначе:

Таким образом, при параллельном соединении проводников величина, обратная общему сопротивлению цепи, равна сумме величин, обратных сопротивлениям параллельно включенных проводников.

Аналогичная ситуация будет наблюдаться и при большем количестве проводников, соединенных параллельно:

Смешанное соединение резисторов.

Помимо параллельного и последовательного соединений резисторов существует еще смешанное соединение. Из названия уже понятно, что при таком соединении в цепи присутствуют резисторы, соединенные как параллельно, так и последовательно. Вот пример такой цепи:

Давайте рассчитаем общее сопротивление цепи. Начнем с резисторов и – они соединены параллельно. Мы можем рассчитать общее сопротивление для этих резисторов и заменить их в схеме одним единственным резистором :

Теперь у нас образовались две группы последовательно соединенных резисторов:

Заменим эти две группы двумя резисторами, сопротивление которых равно:

Как видите, схема стала уже совсем простой ) Заменим группу параллельно соединенных резисторов и одним резистором :

И в итоге у нас на схеме осталось только два резистора соединенных последовательно:

Общее сопротивление цепи получилось равным:

Таким вот образом достаточно большая схема свелась к простейшему последовательному соединению двух резисторов &#128521;

Тут стоит отметить, что некоторые схемы невозможно так просто преобразовать и определить общее сопротивление – для таких схем нужно использовать правила Кирхгофа, о которых мы обязательно поговорим в будущих статьях. А сегодняшняя статья на этом подошла к концу, до скорых встреч на нашем сайте!

Параллельное соединение сопротивлений | Электрикам

Параллельным соединением резисторов (или приемников энергии, ветвей, сопротивлений) называется такое, при котором к одним и тем же двум узлам электрической цепи (рисунок 1) присоединены несколько резисторов (ветвей).

Рис. 1 Изображение параллельного соединения трех резисторов

Проводимость при параллельном соединении

Сопротивление при параллельном соединении:

Для трёх параллельно соединенных сопротивлений

Для двух параллельно соединенных сопротивлений

Для ветвей с одинаковым сопротивлением где n количество ветвей

.

Ток при параллельном соединении

Мощность при параллельном соединении

Доказательство

Так как резисторы присоединены к одним и тем же узлам, то каждый из них находится под одинаковым напряжением U. Согласно закону Ома токи в сопртивлениях определяются по формулам

Из этих формул следует, что токи в параллельных ветвях с сопротивлениями распределяются прямо пропорционально проводимостям ветвей или обратно пропорционально их сопротивлениям. Ряд параллельно соединенных резисторов можно заменить эквивалентным с сопротивлением R, значение которого должно быть таким, чтобы при том же напряжении на выводах ток в эквивалентном резисторе был равен сумме токов в отдельных ветвях:

Эквивалентная проводимость

(1)

т. е. эквивалентная проводимость параллельного соединения резисторов равна сумме проводимостей всех параллельных ветвей. Следовательно, эквивалентное сопротивление будет меньше самого малого из параллельно соединенных резисторов.
Формула (1) дает возможность определить и эквивалентное сопротивление параллельного соединения резисторов. Например, при трех ветвях эквивалентная проводимость

и эквивалентное сопротивление

.

Для двух резисторов

Если сопротивление ветвей одинаково R1 = R2 = R3, то можно воспользоваться формулой

   

в общем случае при соединении n резисторов с одинаковым сопротивлением R1 эквивалентное сопротивление равно

  .

Мощности параллельно соединенных резисторов равна сумме мощностей всех резисторов

  

Физическая формула расчета (определения) эквивалентного сопротивления в цепи

Если электрическая цепь содержит несколько резисторов, то для подсчёта её основных параметров (силы тока, напряжения, мощности) удобно все резистивные устройства заменить на одно эквивалентное сопротивление цепи. Только для него должно выполняться следующее требование: его сопротивление должно быть равным суммарному значению сопротивлений всех элементов, то есть показания амперметра и вольтметра в обычной схеме и в преобразованной не должны измениться. Такой подход к решению задач называется методом свёртывания цепи.

Метод свёртывания цепи

Внимание! Расчёт эквивалентного (общего или суммарного) сопротивления в случае последовательного или параллельного подключения выполняется по разным формулам.

Последовательное соединение элементов

В случае последовательного подключения все приборы соединяются последовательно друг с другом, а собранная цепь не имеет разветвлений.

При таком подключении сила тока, проходящая через каждый резистор, будет одинаковая, а общее падение напряжения складывается из суммарных падений напряжения на каждом из приборов.

Последовательное подключение приборов

Чтобы определить суммарное значение в этом случае, воспользуемся законом Ома, который записывается следующим образом:

I = U/R.

Из вышестоящего выражения получаем значение R:

R = U/I (1).

Поскольку при последовательном соединении:

  • I = I1 = I2 =…= IN (2),
  • U = U1 + U2 +…+ UN (3),

формула для расчёта эквивалентного сопротивления (Rобщ или Rэкв) из (1) – (3) будет иметь вид:

  • Rэкв = (U1 + U2 + …+ UN)/I,
  • Rэкв = R1 + R2 + … + RN (4).

Таким образом, если имеется N последовательно соединённых одинаковых элементов, то их можно заменить на одно устройство, у которого:

Rобщ = N·R (5).

Параллельное соединение

При таком подключении входы от всех устройств соединены в одной точке, выходы – в другой точке. Эти точки в физике и электротехнике называются узлами. На электрических схемах узлы представляют собой места разветвления проводников и обозначаются точками.

Параллельное соединение

Расчет эквивалентного сопротивления также выполняем с помощью закона Ома.

В этом случае общее значение силы тока складывается из суммы сил токов, протекающих по каждой ветви, а величина падения напряжения для каждого устройства и общее напряжение одинаковые.

Если имеются N резистивных устройств, подключенных таким образом, то:

I = I1 + I2  + … + IN (6),

U = U1 = U2 = … = UN (7).

Из выражений (1), (6) и (7) имеем:

  • Rобщ = U/(I1 + I2 + …+ IN),
  • 1/Rэкв = 1/R1 + 1/R2 +…+ 1/RN (8).

Если имеется N одинаковых резисторов, имеющих подключение данного типа, то формула (8) преобразуется следующим образом:

Rобщ = R · R / N·R = R / N (9).

Если соединены несколько катушек индуктивности, то их суммарное индуктивное сопротивление рассчитывается так же, как и для резисторов.

Расчёт при смешанном соединении устройств

В случае смешанного подключения присутствуют участки с последовательным и параллельным подключениями элементов.

При решении задачи используют метод сворачивания цепи (метод эквивалентных преобразований). Его используют для вычисления параметров в том случае, если есть один источник энергии.

Предположим, задана следующая задача. Электрическая схема (см. рис. ниже) состоит из 7 резисторов. Рассчитайте токи на всех резисторах, если имеются следующие исходные данные:

  • R1 = 1Ом,
  • R2 = 2Ом,
  • R3 = 3Ом,
  • R4 = 6Ом,
  • R5 = 9Ом,
  • R6 = 18Ом,
  • R7 = 2,8Ом,
  • U = 32В.

Электрическая схема

Из закона Ома имеем: 

I = U/R,

где R – суммарное сопротивление всех приборов.

Его будем находить, воспользовавшись методом сворачивания цепи.

Элементы R2 и R3 подключены параллельно, поэтому их можно заменить на R2,3, величину которого можно рассчитать по формуле:

R2,3= R2·R3 / (R2+R3).

R4, R5 и R6 также включены параллельно, и их можно заменить на R4,5,6, которое вычисляется следующим образом:

1/R4,5,6 = 1/R4+1/R5+1/R6.

Таким образом, схему, изображённую на картинке выше, можно заменить на эквивалентную, в которой вместо резисторов R2, R3 и R4, R5, R6 используются R2,3 и R4,5,6.

Эквивалентная схема

Согласно картинке выше, в результате преобразований получаем последовательное соединение резисторов R1, R2,3, R4,5,6 и R7.

Rобщ может быть найдено по формуле:

Rобщ = R1 + R2,3 + R4,5,6 + R7.

Подставляем числовые значения и рассчитываем R для определённых участков:

  • R2.3 = 2Ом·3Ом / (2Ом + 3Ом) = 1,2Ом,
  • 1/R4,5,6 = 1/6Ом + 1/9Ом + 1/18Ом = 1/3Ом,
  • R4,5,6 = 3Ом,
  • Rэкв = 1Ом + 1,2Ом + 3Ом + 2,8Ом= 8Ом.

Теперь, после того, как нашли Rэкв, можно вычислять значение I:

I = 32В / 8Ом = 4А.

После того, как мы получили величину общего тока, можно вычислить силу тока, протекающую на каждом участке.

Поскольку R1, R2,3, R4,5,6 и R7 соединены последовательно, то:

I1 = I2,3 = I4,5,6 = I7 = I = 4А.

На участке R2,3 напряжение находим по формуле:

  • U2,3 = I2,3·R2,3,
  • U2,3 = 4А·1,2Ом = 4,8В.

Поскольку R2 и R3 подключены параллельно, то U2,3 = U2 = U3, следовательно:

  • I2 = U2 / R2,
  • I2 = 4,8В / 2Ом = 2,4А,
  • I3 = U3 / R3,
  • I3 = 4,8В / 3Ом = 1,6А.

Проверяем правильность решения:

  • I2,3 = I2 + I3,
  • I2,3 = 2,4А + 1,6А = 4А.

На участке R4,5,б напряжение также находим, исходя из закона Ома:

  • U4,5,6 = I4,5,6·R4,5,6,
  • U4,5,6 = 4А·3Ом = 12В.

Так как R4, R5, Rб подключены параллельно друг к другу, то:

U4,5,6 = U4 = U5 = U6 = 12В.

Вычисляем I4, I5, I6:

  • I4 = U4 / R4,
  • I4 = 12В / 6Ом = 2А,
  • I5 = U5 / R5,
  • I5 = 12В / 9Ом » 1,3А,
  • I6 = U6 / R6,
  • I5 = 12В / 18Ом » 0,7А.

Проверяем правильность решения:

I4,5,6 = 2А + 1,3А + 0,7А = 4А.

Чтобы автоматизировать выполнение расчётов эквивалентных значений для различных участков цепи, можно воспользоваться сервисами сети Интернет, которые предлагают на их сайтах выполнить онлайн вычисления нужных электрических характеристик. Сервис обычно имеет встроенную специальную программу – калькулятор, которая помогает быстро выполнить расчет сопротивления цепи любой сложности.

Таким образом, использование метода эквивалентных преобразований при расчёте смешанных соединений различных устройств позволяет упростить и ускорить выполнение вычислений основных электрических параметров.

Видео

Оцените статью:Калькулятор параллельного сопротивления

— Инструменты для электротехники и электроники

Как рассчитать полное сопротивление резисторов, включенных параллельно

Расчет эквивалентного сопротивления (R EQ ) параллельно подключенных резисторов вручную может быть утомительным. Этот инструмент был разработан, чтобы помочь вам быстро рассчитать эквивалентное сопротивление, независимо от того, подключены ли у вас два или десять резисторов параллельно. Чтобы использовать его, просто укажите количество параллельных резисторов и значение сопротивления для каждого из них.

Вы можете легко вычислить эквивалентное сопротивление, если у вас есть два идентичных резистора, подключенных параллельно: это половина отдельного сопротивления. Это удобно, когда вам нужно определенное значение сопротивления, а подходящей детали нет в наличии. Например, если вы знаете, что вам нужно около 500 Ом, чтобы получить желаемую яркость светодиодной цепи, вы можете использовать два резистора 1 кОм параллельно.

Имейте в виду, что ток через отдельный резистор не изменяется, когда вы добавляете резисторы параллельно, потому что добавление резисторов параллельно не влияет на напряжение на выводах резисторов.Изменяется общий ток, подаваемый источником питания, а не ток через один конкретный резистор.

Уравнения

$$ \ frac {1} {R_ {EQ}} = \ frac {1} {R_ {1}} + \ frac {1} {R_ {2}} + \ frac {1} {R_ {3}} + … + \ frac {1} {R_ {N}} $$

Когда у вас есть только два параллельно подключенных резистора: $$ R_ {EQ} = \ frac {R_1 \ times R_2} {R_1 + R_2} $$

Приложения

последовательно подключенных резисторов эквивалентны одному резистору, сопротивление которого является суммой каждого отдельного резистора.С другой стороны, параллельное соединение резисторов дает эквивалентное сопротивление, которое всегда ниже, чем у каждого отдельного резистора. Если подумать, это имеет смысл: если вы подаете напряжение на резистор, протекает определенное количество тока. Если вы добавите еще один резистор параллельно первому, вы, по сути, откроете новый канал, по которому может течь больше тока. Независимо от того, насколько велик второй резистор, общий ток, протекающий от источника питания, будет, по крайней мере, немного выше, чем ток через единственный резистор.А если общий ток выше, общее сопротивление должно быть ниже.

Дополнительная литература

Формулы и калькулятор »Электроника

Формулы, расчеты и калькулятор для определения общего сопротивления резисторов, установленных последовательно и параллельно.


Resistance Tutorial:
Что такое сопротивление
Закон Ома
Омические и неомические проводники
Сопротивление лампы накаливания
Удельное сопротивление
Таблица удельного сопротивления для распространенных материалов
Температурный коэффициент сопротивления
Электрическая проводимость
Последовательные и параллельные резисторы
Таблица параллельных резисторов


Резисторы могут быть размещены во многих конфигурациях в электрической или электронной схеме — иногда последовательно, иногда параллельно.

Когда они размещаются в этих конфигурациях, важно иметь возможность рассчитать общее сопротивление. Этого можно довольно легко достичь, если использовать правильные формулы — есть простые формулы как для последовательных, так и для параллельных резисторов.

При проектировании электронной схемы или по другой причине возможность вычисления сопротивления комбинации резисторов может быть очень полезной.

В электронных схемах комбинации резисторов могут быть сведены к последовательным элементам и параллельным элементам, хотя при использовании других электронных компонентов комбинации могут быть более сложными.Однако во многих случаях расчет значений последовательного и параллельного сопротивления имеет большое значение.

Резисторы в серии

Самая простая конфигурация электронной схемы — это резисторы, включенные последовательно. Это может произойти, если несколько этих электронных компонентов соединены последовательно или необходимо добавить сопротивление кабеля к сопротивлению резистора и т. Д.

Если резисторы соединены последовательно, то общее сопротивление является просто суммой отдельных резисторов.

Последовательные резисторы

Величину резисторов или сопротивлений, включенных последовательно, можно математически выразить следующим образом:

Пример расчета последовательных резисторов:
В качестве примера, если три резистора с номиналами 1 кОм, 2 кОм и 3 кОм соединены последовательно, то общее сопротивление составит 1 + 2 + 3 кОм = 6 кОм.

В реальных жизненных ситуациях и аспектах проектирования электрических и электронных схем будет много областей, где есть электронные компоненты, такие как резисторы или другие элементы, вносящие сопротивление, где количество последовательно соединенных сопротивлений, которые необходимо суммировать.

Сопротивления параллельно

Есть также много случаев, когда электронные компоненты, такие как резисторы, а также другие элементы, вызывающие сопротивление, появляются в электрической или электронной цепи параллельно.

Если резисторы размещены параллельно, они разделяют ток, и ситуацию немного сложнее рассчитать, но все же довольно легко.

1Rtotal = 1R1 + 1R2 + 1R3 + ……

Пример расчета сопротивления резисторов, включенных параллельно:
Чтобы дать пример, если есть три резистора, подключенных параллельно со значениями 1 кОм, 2 кОм и Омега и 3 кОм, то можно вычислить общее значение комбинации:

1 / R Итого = 1/1000 + 1/2000 + 1/3000

1 / R Итого = 1/1000 + 1/2000 + 1/3000

1 / R Итого = 6/6000 + 3/6000 + 2/6000

1 / R Итого = 11/6000

R Всего = 6000/11 Ом или 545 Ом

Корпус только двух резисторов, включенных параллельно

Во многих конструкциях электронных схем наиболее распространенный экземпляр резисторов, включенных параллельно, состоит только из двух электронных компонентов.

Часто бывает так, что один резистор подключается параллельно другому. Или другой случай может быть, когда резистор помещается на клеммы для цепи или сети, которая имеет определенное сопротивление. В этом случае необходимо только рассчитать общее сопротивление для двух параллельно включенных резисторов.

Если необходимо рассчитать общее значение для двух параллельных резисторов, уравнением можно манипулировать и значительно упростить его, как показано ниже:

Эта формула значительно упрощает вычисление номинала двух параллельных резисторов, так как требует только одного умножения, одного сложения и одного деления.Часто это можно сделать мысленно или на клочке бумаги. В качестве альтернативы можно использовать наш простой калькулятор для двух параллельно включенных резисторов, приведенный ниже.

Вычислитель для двух параллельно включенных резисторов

Этот калькулятор параллельного сопротивления обеспечивает простой метод расчета общего сопротивления для двух резисторов, соединенных параллельно.

Хотя вычисление номиналов параллельных резисторов для двух резисторов упрощается до простой формулы, иногда гораздо проще и быстрее использовать калькулятор.

Чтобы использовать калькулятор параллельных резисторов, просто введите значения параллельных резисторов в Ом, Ом или кОм и т. Д. В два поля ввода, но обратите внимание, что все значения должны быть в одних и тех же единицах, то есть оба в Ом кОм МОм и т. Д. Затем вычислитель параллельных резисторов предоставит общее сопротивление двух резисторов в тех же единицах, что и вход.

Введите два значения для резисторов, R1 и R2, в поля, представленные в калькуляторе ниже, нажмите «Рассчитать», и будет предоставлено общее сопротивление.

Калькулятор параллельного сопротивления

Калькулятор параллельных резисторов позволяет легко рассчитать сопротивление двух резисторов, включенных параллельно, экономя записывать все и прибегая к ручке и бумаге или калькулятору в той или иной форме.

Знание того, как рассчитать значения резисторов, включенных последовательно и параллельно, является ключом к пониманию того, как работают электрические и электронные схемы. Эти концепции используются как вторая натура при проектировании электрических и электронных схем.

Другие основные концепции электроники:
Напряжение
Текущий
Мощность
Сопротивление
Емкость
Индуктивность
Трансформеры
Децибел, дБ
Законы Кирхгофа
Q, добротность
РЧ шум

Вернуться в меню «Основные понятия электроники». . .

Последовательные и параллельные резисторы

Резисторы

серии

Общее сопротивление в цепи с последовательно включенными резисторами равно сумме отдельных сопротивлений.

Цели обучения

Рассчитайте общее сопротивление в цепи с последовательно включенными резисторами

Ключевые выводы

Ключевые моменты
  • Одинаковый ток течет последовательно через каждый резистор.
  • Отдельные последовательно включенные резисторы не получают полное напряжение источника, а делят его.
  • Общее сопротивление в последовательной цепи равно сумме отдельных сопротивлений: [латекс] \ text {RN} (\ text {series}) = \ text {R} _1 + \ text {R} _2 + \ text {R} _3 +… + \ text {R} _ \ text {N} [/ latex].
Ключевые термины
  • серия : ряд элементов, которые следуют одно за другим или связаны друг за другом.
  • сопротивление : Противодействие прохождению электрического тока через этот элемент.

Обзор

Большинство схем имеет более одного компонента, называемого резистором, который ограничивает поток заряда в цепи. Мера этого предела для потока заряда называется сопротивлением. Самыми простыми комбинациями резисторов являются последовательное и параллельное соединение.Общее сопротивление комбинации резисторов зависит как от их индивидуальных значений, так и от способа их подключения.

Последовательные цепи : Краткое введение в анализ последовательных и последовательных цепей, включая закон Кирхгофа по току (KCL) и закон Кирхгофа по напряжению (KVL).

Резисторы серии

Резисторы включены последовательно, когда заряд или ток должны проходить через компоненты последовательно.

Резисторы в серии : Эти четыре резистора соединены последовательно, потому что, если бы ток подавался на один конец, он бы протекал через каждый резистор последовательно до конца.

показывает резисторы, последовательно подключенные к источнику напряжения. Общее сопротивление в цепи равно сумме отдельных сопротивлений, поскольку ток должен проходить через каждый резистор последовательно через цепь.

Резисторы, подключенные последовательно : три резистора, подключенные последовательно к батарее (слева), и эквивалентное одиночное или последовательное сопротивление (справа).

Использование закона Ома для расчета изменений напряжения в резисторах серии

Согласно закону Ома падение напряжения V на резисторе при протекании через него тока рассчитывается по формуле V = IR, где I — ток в амперах (A), а R — сопротивление в омах (Ω). .

Таким образом, падение напряжения на R 1 равно В 1 = IR 1 , на R 2 равно V 2 = IR 2 , а на R 3 равно V 3 = IR 3 .Сумма напряжений будет равна: V = V 1 + V 2 + V 3 , исходя из сохранения энергии и заряда. Если подставить значения отдельных напряжений, получим:

[латекс] \ text {V} = \ text {IR} _1 + \ text {IR} _2 + \ text {IR} _3 [/ latex]

или

[латекс] \ text {V} = \ text {I} (\ text {R} _1 + \ text {R} _2 + \ text {R} _3) [/ латекс]

Это означает, что полное сопротивление в серии равно сумме отдельных сопротивлений. Следовательно, для каждой цепи с Н количество резисторов, включенных последовательно:

[латекс] \ text {RN} (\ text {series}) = \ text {R} _1 + \ text {R} _2 + \ text {R} _3 +… + \ text {R} _ \ text {N }.[/ латекс]

Поскольку весь ток должен проходить через каждый резистор, он испытывает сопротивление каждого, а последовательно соединенные сопротивления просто складываются.

Поскольку напряжение и сопротивление имеют обратную зависимость, отдельные последовательно включенные резисторы не получают полное напряжение источника, а делят его. Об этом свидетельствует пример, когда две лампочки соединены в последовательную цепь с аккумулятором. В простой схеме, состоящей из одной батареи 1,5 В и одной лампочки, падение напряжения на лампе будет равно 1.5V через него. Однако, если бы две лампочки были соединены последовательно с одной и той же батареей, на каждой из них было бы падение напряжения 1,5 В / 2 или 0,75 В. Это будет очевидно по яркости света: каждая из двух последовательно соединенных лампочек будет в два раза слабее, чем одиночная лампочка. Следовательно, резисторы, соединенные последовательно, потребляют такое же количество энергии, как и один резистор, но эта энергия распределяется между резисторами в зависимости от их сопротивлений.

Параллельные резисторы

Общее сопротивление в параллельной цепи равно сумме обратных сопротивлений каждого отдельного сопротивления.

Цели обучения

Рассчитайте общее сопротивление в цепи с параллельно включенными резисторами

Ключевые выводы

Ключевые моменты
  • Общее сопротивление в параллельной цепи меньше наименьшего из отдельных сопротивлений.
  • Каждый резистор, включенный параллельно, имеет то же напряжение, что и приложенный к нему источник (напряжение в параллельной цепи постоянно).
  • Параллельные резисторы не получают суммарный ток каждый; они делят его (ток зависит от номинала каждого резистора и общего количества резисторов в цепи).
Ключевые термины
  • сопротивление : Противодействие прохождению электрического тока через этот элемент.
  • параллельно : Расположение электрических компонентов, при котором ток течет по двум или более путям.

Обзор

Резисторы в цепи могут быть включены последовательно или параллельно. Общее сопротивление комбинации резисторов зависит как от их индивидуальных значений, так и от способа их подключения.

Parallel Circuits : Краткий обзор анализа параллельных цепей с использованием таблиц VIRP для студентов-физиков средней школы.

Параллельные резисторы

Резисторы включены параллельно, когда каждый резистор подключен непосредственно к источнику напряжения путем соединения проводов с незначительным сопротивлением. Таким образом, к каждому резистору приложено полное напряжение источника.

Параллельные резисторы : Параллельное соединение резисторов.

Каждый резистор потребляет такой же ток, как если бы он был единственным резистором, подключенным к источнику напряжения. Это верно для схем в доме или квартире. Каждая розетка, подключенная к устройству («резистор»), может работать независимо, и ток не должен проходить через каждое устройство последовательно.

Закон

Ом и параллельные резисторы

Каждый резистор в цепи имеет полное напряжение. Согласно закону Ома токи, протекающие через отдельные резисторы, равны [латекс] \ text {I} _1 = \ frac {\ text {V}} {\ text {R} _1} [/ latex], [latex] \ text {I} _2 = \ frac {\ text {V}} {\ text {R} _2} [/ latex] и [latex] \ text {I} _3 = \ frac {\ text {V}} {\ text {R} _3} [/ латекс].Сохранение заряда подразумевает, что полный ток равен сумме этих токов:

Параллельные резисторы : три резистора, подключенные параллельно батарее, и эквивалентное одиночное или параллельное сопротивление.

[латекс] \ text {I} = \ text {I} _1 + \ text {I} _2 + \ text {I} _3. [/ Latex]

Подстановка выражений для отдельных токов дает:

[латекс] \ text {I} = \ frac {\ text {V}} {\ text {R} _1} + \ frac {\ text {V}} {\ text {R} _2} + \ frac {\ текст {V}} {\ text {R} _3} [/ latex]

или

[латекс] \ text {I} = \ text {V} (\ frac {1} {\ text {R} _1} + \ frac {1} {\ text {R} _2} + \ frac {1} { \ text {R} _3}) [/ latex]

Это означает, что полное сопротивление в параллельной цепи равно сумме обратных сопротивлений каждого отдельного сопротивления.Следовательно, для каждой цепи с числом [latex] \ text {n} [/ latex] или параллельно подключенных резисторов —

[латекс] \ text {R} _ {\ text {n} \; (\ text {parallel})} = \ frac {1} {\ text {R} _1} + \ frac {1} {\ text { R} _2} + \ frac {1} {\ text {R} _3}… + \ frac {1} {\ text {R} _ \ text {n}}. [/ Latex]

Это соотношение приводит к общему сопротивлению, которое меньше наименьшего из отдельных сопротивлений. Когда резисторы подключены параллельно, от источника течет больше тока, чем протекает для любого из них по отдельности, поэтому общее сопротивление ниже.

Каждый резистор, включенный параллельно, имеет такое же полное напряжение источника, как на него, но делит общий ток между ними. Примером может служить соединение двух лампочек в параллельную цепь с аккумулятором на 1,5 В. В последовательной цепи две лампочки будут вдвое менее тусклыми при подключении к одному источнику батареи. Однако, если бы две лампочки были подключены параллельно, они были бы столь же яркими, как если бы они были подключены к батарее по отдельности. Поскольку к обеим лампочкам подается одинаковое полное напряжение, батарея также разряжается быстрее, поскольку она по существу обеспечивает полную энергию обеими лампочками.В последовательной цепи батарея будет работать столько же, сколько и с одной лампочкой, только тогда яркость будет разделена между лампочками.

Комбинированные схемы

Комбинированная цепь может быть разбита на аналогичные части, которые работают последовательно или параллельно.

Цели обучения

Описать расположение резисторов в комбинированной цепи и его практическое значение

Ключевые выводы

Ключевые моменты
  • Более сложные соединения резисторов иногда представляют собой просто комбинации последовательного и параллельного.
  • Различные части комбинированной схемы могут быть идентифицированы как последовательные или параллельные, уменьшены до их эквивалентов, а затем уменьшены до тех пор, пока не останется единственное сопротивление.
  • Сопротивление в проводах снижает ток и мощность, подаваемые на резистор. Если сопротивление в проводах относительно велико, как в изношенном (или очень длинном) удлинителе, то эти потери могут быть значительными и повлиять на выходную мощность в бытовые приборы.
Ключевые термины
  • серия : ряд элементов, которые следуют одно за другим или связаны друг за другом.
  • параллельно : Расположение электрических компонентов, при котором ток течет по двум или более путям.
  • комбинированная схема : электрическая цепь, содержащая несколько резисторов, которые соединены комбинацией как последовательного, так и параллельного соединения.

Комбинированные схемы

Более сложные соединения резисторов иногда представляют собой просто комбинации последовательного и параллельного. Это часто встречается, особенно если учитывать сопротивление проводов.В этом случае сопротивление провода включено последовательно с другими сопротивлениями, включенными параллельно.

Комбинированная цепь может быть разбита на аналогичные части, которые являются последовательными или параллельными, как показано на схеме. На рисунке общее сопротивление может быть рассчитано путем соединения трех резисторов друг с другом последовательно или параллельно. R 1 и R 2 соединены параллельно друг другу, поэтому мы знаем, что для этого подмножества сопротивление, обратное сопротивлению, будет равно:

Сеть резисторов : В этой комбинированной схеме цепь может быть разбита на последовательный компонент и параллельный компонент.

Комбинированные схемы : Два параллельно включенных резистора с одним резистором.

[латекс] \ frac {1} {\ text {R} _1} + \ frac {1} {\ text {R} _2} [/ latex] или [латекс] \ frac {\ text {R} _1 \ text {R} _2} {\ text {R} _1 + \ text {R} _2} [/ latex]

R 3 подключен последовательно к и R 1 и R 2 , поэтому сопротивление будет рассчитываться как:

[латекс] \ text {R} = \ frac {\ text {R} _1 \ text {R} _2} {\ text {R} _1 + \ text {R} _2} + \ text {R} _3 [/ латекс ]

Сложные комбинированные схемы

Для более сложных комбинированных схем различные части могут быть идентифицированы как последовательные или параллельные, уменьшены до их эквивалентов, а затем уменьшены до тех пор, пока не останется единственное сопротивление, как показано на.На этом рисунке комбинация из семи резисторов была идентифицирована как включенные последовательно или параллельно. На исходном изображении две обведенные кружком секции показывают резисторы, включенные параллельно.

Сокращение комбинированной схемы : Эта комбинация из семи резисторов имеет как последовательные, так и параллельные части. Каждый из них идентифицируется и приводится к эквивалентному сопротивлению, а затем уменьшается до тех пор, пока не будет достигнуто одно эквивалентное сопротивление.

Уменьшение этих параллельных резисторов до одного значения R позволяет нам визуализировать схему в более упрощенном виде.На верхнем правом изображении мы видим, что обведенная кружком часть содержит два последовательно соединенных резистора. Мы можем дополнительно уменьшить это до другого значения R, добавив их. Следующий шаг показывает, что два обведенных резистора включены параллельно. Уменьшение тех бликов, что последние два соединены последовательно и, таким образом, могут быть уменьшены до одного значения сопротивления для всей цепи.

Практическое применение комбинированной схемы состоит в том, что сопротивление в проводах снижает ток и мощность, подаваемую на резистор.Комбинированная цепь может быть преобразована в последовательную цепь на основе понимания эквивалентного сопротивления параллельных ветвей комбинированной цепи. Последовательная цепь может использоваться для определения общего сопротивления цепи. По сути, сопротивление провода является последовательным с резистором. Таким образом, увеличивается общее сопротивление и уменьшается ток. Если сопротивление провода относительно велико, как в изношенном (или очень длинном) удлинителе, то эти потери могут быть значительными. Если потребляется большой ток, падение ИК-излучения в проводах также может быть значительным.

Зарядка аккумулятора: последовательные и параллельные ЭДС

При последовательном включении источников напряжения их ЭДС и внутренние сопротивления складываются; параллельно они остаются прежними.

Цели обучения

Сравнить сопротивления и электродвижущие силы для источников напряжения, подключенных с одинаковой и противоположной полярностью, последовательно и параллельно

Ключевые выводы

Ключевые моменты
  • ЭДС, соединенные последовательно с одинаковой полярностью, являются аддитивными и приводят к более высокой полной ЭДС.
  • Две ЭДС, соединенные последовательно с противоположной полярностью, имеют общую ЭДС, равную разнице между ними, и могут использоваться для зарядки источника более низкого напряжения.
  • Два источника напряжения с идентичными ЭДС, соединенные параллельно, имеют чистую ЭДС, эквивалентную одному источнику ЭДС, однако чистое внутреннее сопротивление меньше и, следовательно, дает более высокий ток.
Ключевые термины
  • параллельно : расположение электрических компонентов, при котором ток течет по двум или более путям.
  • электродвижущая сила : (ЭДС) — напряжение, генерируемое батареей или магнитной силой в соответствии с законом Фарадея. Она измеряется в вольтах, а не в ньютонах, и поэтому на самом деле не является силой.
  • серия : ряд вещей, которые следуют одно за другим или связаны друг за другом.

Когда используется более одного источника напряжения, они могут быть подключены последовательно или параллельно, аналогично резисторам в цепи.Когда источники напряжения включены последовательно в одном направлении, их внутренние сопротивления складываются, а их электродвижущая сила или ЭДС складываются алгебраически. Эти типы источников напряжения распространены в фонариках, игрушках и других приборах. Обычно ячейки включены последовательно, чтобы обеспечить большую суммарную ЭДС.

Фонарик и лампочка : Последовательное соединение двух источников напряжения в одном направлении. Эта схема представляет собой фонарик с двумя последовательно включенными ячейками (источниками напряжения) и одной лампочкой (сопротивление нагрузки).

Батарея — это соединение нескольких гальванических элементов. Однако недостатком такого последовательного соединения ячеек является то, что их внутреннее сопротивление увеличивается. Иногда это может быть проблематично. Например, если вы поместите в машину две батареи на 6 В вместо обычной батареи на 12 В, вы добавите как ЭДС, так и внутреннее сопротивление каждой батареи. Таким образом, вы получите ту же ЭДС 12 В, хотя внутреннее сопротивление тогда будет удвоено, что вызовет у вас проблемы, когда вы захотите запустить двигатель.

Но, если ячейки противостоят друг другу — например, когда одна вставляется в прибор задом наперед, — общая ЭДС меньше, так как это алгебраическая сумма отдельных ЭДС. Когда он перевернут, он создает ЭДС, которая противодействует другой, и приводит к разнице между двумя источниками напряжения.

Зарядное устройство : представляет два источника напряжения, соединенных последовательно с противоположными ЭДС. Ток течет в направлении большей ЭДС и ограничивается суммой внутренних сопротивлений.(Обратите внимание, что каждая ЭДС представлена ​​на рисунке буквой E.) Зарядное устройство, подключенное к аккумулятору, является примером такого подключения. Зарядное устройство должно иметь большую ЭДС, чем батарея, чтобы через него протекал обратный ток.

Когда два источника напряжения с идентичными ЭДС соединены параллельно и также подключены к сопротивлению нагрузки, общая ЭДС равна индивидуальной ЭДС. Но общее внутреннее сопротивление уменьшается, поскольку внутренние сопротивления параллельны. Таким образом, параллельное соединение может производить больший ток.

Две идентичные ЭДС : Два источника напряжения с одинаковыми ЭДС (каждый отмечен буквой E), подключенные параллельно, создают одинаковую ЭДС, но имеют меньшее общее внутреннее сопротивление, чем отдельные источники. Параллельные комбинации часто используются для подачи большего тока.

ЭДС и напряжение на клеммах

Выходное напряжение или напряжение на клеммах источника напряжения, такого как аккумулятор, зависит от его электродвижущей силы и внутреннего сопротивления.

Цели обучения

Выразите взаимосвязь между электродвижущей силой и напряжением на клеммах в форме уравнения

Ключевые выводы

Ключевые моменты
  • Электродвижущая сила (ЭДС) — это разность потенциалов источника при отсутствии тока.
  • Напряжение на клеммах — это выходное напряжение устройства, измеренное на его клеммах.
  • Напряжение на клеммах рассчитывается по формуле V = ЭДС — Ir.
Ключевые термины
  • электродвижущая сила : (ЭДС) — напряжение, генерируемое батареей или магнитной силой в соответствии с законом Фарадея. Она измеряется в вольтах, а не в ньютонах, и поэтому на самом деле не является силой.
  • напряжение на клеммах : Выходное напряжение устройства, измеренное на его клеммах.
  • разность потенциалов : разница в потенциальной энергии между двумя точками в электрическом поле; разница в заряде между двумя точками в электрической цепи; Напряжение.

Когда вы забываете выключить автомобильные фары, они постепенно тускнеют по мере разрядки аккумулятора. Почему они просто не мигают, когда батарея разряжена? Их постепенное затемнение означает, что выходное напряжение батареи уменьшается по мере разряда батареи. Причина снижения выходного напряжения для разряженных или перегруженных батарей заключается в том, что все источники напряжения состоят из двух основных частей — источника электрической энергии и внутреннего сопротивления.

Электродвижущая сила

Все источники напряжения создают разность потенциалов и могут подавать ток, если подключены к сопротивлению. В небольшом масштабе разность потенциалов создает электрическое поле, которое воздействует на заряды, вызывая ток. Мы называем эту разность потенциалов электродвижущей силой (сокращенно ЭДС). ЭДС — это вообще не сила; это особый тип разности потенциалов источника при отсутствии тока. Единицы измерения ЭДС — вольты.

Электродвижущая сила напрямую связана с источником разности потенциалов, например с конкретной комбинацией химических веществ в батарее.Однако при протекании тока ЭДС отличается от выходного напряжения устройства. Напряжение на выводах батареи, например, меньше, чем ЭДС, когда батарея подает ток, и оно падает дальше, когда батарея разряжается или разряжается. Однако, если выходное напряжение устройства можно измерить без потребления тока, то выходное напряжение будет равно ЭДС (даже для сильно разряженной батареи).

Напряжение на клеммах

представляет схематическое изображение источника напряжения.Выходное напряжение устройства измеряется на его клеммах и называется напряжением на клеммах В . Напряжение на клеммах определяется уравнением:

Схематическое изображение источника напряжения : Любой источник напряжения (в данном случае углеродно-цинковый сухой элемент) имеет ЭДС, связанную с источником разности потенциалов, и внутреннее сопротивление r, связанное с его конструкцией. (Обратите внимание, что сценарий E означает ЭДС.) Также показаны выходные клеммы, на которых измеряется напряжение на клеммах V.Поскольку V = ЭДС-Ir, напряжение на клеммах равно ЭДС, только если ток не течет.

[латекс] \ text {V} = \ text {emf} — \ text {Ir} [/ latex],

, где r — внутреннее сопротивление, а I — ток, протекающий во время измерения.

I является положительным, если ток течет от положительного вывода. Чем больше ток, тем меньше напряжение на клеммах. Точно так же верно, что чем больше внутреннее сопротивление, тем меньше напряжение на клеммах.

резисторов последовательно и параллельно

Цели обучения

К концу этого раздела вы сможете:

  • Нарисуйте цепь с резисторами, включенными параллельно и последовательно.
  • Рассчитайте падение напряжения тока на резисторе, используя закон Ома.
  • Сравните способ расчета общего сопротивления для резисторов, включенных последовательно и параллельно.
  • Объясните, почему полное сопротивление параллельной цепи меньше наименьшего сопротивления любого из резисторов в этой цепи.
  • Рассчитайте общее сопротивление цепи, которая содержит смесь резисторов, подключенных последовательно и параллельно.

Большинство схем имеет более одного компонента, называемого резистором , который ограничивает поток заряда в цепи.Мера этого предела расхода заряда называется сопротивлением . Простейшие комбинации резисторов — это последовательное и параллельное соединение, показанное на рисунке 1. Общее сопротивление комбинации резисторов зависит как от их индивидуальных значений, так и от способа их подключения.

Рис. 1. (a) Последовательное соединение резисторов. (б) Параллельное соединение резисторов.

Когда резисторы в серии ? Резисторы включены последовательно всякий раз, когда поток заряда, называемый током , должен проходить через устройства последовательно.Например, если ток течет через человека, держащего отвертку, в землю, тогда R 1 на рисунке 1 (a) может быть сопротивлением вала отвертки, R 2 сопротивлением ее ручки , R 3 сопротивление тела человека и R 4 сопротивление его обуви. На рисунке 2 показаны резисторы, последовательно подключенные к источнику напряжения . Кажется разумным, что полное сопротивление является суммой отдельных сопротивлений, учитывая, что ток должен проходить через каждый резистор последовательно.(Этот факт был бы преимуществом для человека, желающего избежать поражения электрическим током, который мог бы уменьшить ток, надев обувь с резиновыми подошвами с высоким сопротивлением. прибор, уменьшающий рабочий ток.) ​​

Рис. 2. Три резистора, подключенных последовательно к батарее (слева), и эквивалентное одиночное или последовательное сопротивление (справа).

Чтобы убедиться, что последовательно соединенные сопротивления действительно складываются, давайте рассмотрим потерю электроэнергии, называемую падением напряжения , в каждом резисторе на Рисунке 2.Согласно закону Ома падение напряжения В на резисторе при протекании через него тока рассчитывается по формуле В = IR , где I равно току в амперах (A) и R — сопротивление в Ом (Ом). Другой способ представить это: В, — это напряжение, необходимое для протекания тока I через сопротивление R . Таким образом, падение напряжения на R 1 составляет В 1 = IR 1 , что на R 2 составляет В 2 = IR 2 и что по R 3 составляет V 3 = IR 3 .Сумма этих напряжений равна выходному напряжению источника; то есть

В = В 1 + В 2 + В 3 .

Это уравнение основано на сохранении энергии и сохранении заряда. Электрическая потенциальная энергия может быть описана уравнением PE = qV , где q — электрический заряд, а V — напряжение. Таким образом, энергия, поставляемая источником, составляет кв.кв. , а энергия, рассеиваемая резисторами, равна

.

qV 1 + qV 2 + qV 3 .

Установление связей: законы сохранения

Вывод выражений для последовательного и параллельного сопротивления основан на законах сохранения энергии и сохранения заряда, которые гласят, что общий заряд и полная энергия постоянны в любом процессе. Эти два закона непосредственно участвуют во всех электрических явлениях и будут многократно использоваться для объяснения как конкретных эффектов, так и общего поведения электричества.

Эти энергии должны быть равны, потому что в цепи нет другого источника и другого назначения для энергии.Таким образом, qV = qV 1 + qV 2 + qV 3 . Заряд q отменяется, давая V = V 1 + V 2 + V 3 , как указано. (Обратите внимание, что одинаковое количество заряда проходит через батарею и каждый резистор за заданный промежуток времени, поскольку нет емкости для хранения заряда, нет места для утечки заряда и заряд сохраняется.) Теперь подстановка значений для отдельных напряжений дает

V = IR 1 + IR 2 + IR 3 = I ( R 1 + R 2 + R ).

Обратите внимание, что для эквивалентного сопротивления одной серии R с , мы имеем

В = ИК с .

Это означает, что полное или эквивалентное последовательное сопротивление R с трех резисторов составляет R с = R 1 + R 2 + R 3 .Эта логика действительна в общем для любого количества резисторов, включенных последовательно; таким образом, полное сопротивление R с последовательного соединения равно

R с = R 1 + R 2 + R 3 +…,

, как предлагается. Поскольку весь ток должен проходить через каждый резистор, он испытывает сопротивление каждого, а последовательно соединенные сопротивления просто складываются.

Пример 1. Расчет сопротивления, тока, падения напряжения и рассеиваемой мощности: анализ последовательной цепи

Предположим, что выходное напряжение батареи на рисунке 2 равно 12.0 В, а сопротивления равны R 1 = 1,00 Ом, R 2 = 6,00 Ом и R 3 = 13,0 Ом. а) Каково полное сопротивление? (б) Найдите ток. (c) Рассчитайте падение напряжения на каждом резисторе и покажите, как они складываются, чтобы равняться выходному напряжению источника. (d) Рассчитайте мощность, рассеиваемую каждым резистором. (e) Найдите выходную мощность источника и покажите, что она равна общей мощности, рассеиваемой резисторами.

Стратегия и решение для (а)

Общее сопротивление — это просто сумма отдельных сопротивлений, определяемая следующим уравнением:

[латекс] \ begin {array} {lll} {R} _ {\ text {s}} & = & {R} _ {1} + {R} _ {2} + {R} _ {3} \ \ & = & 1.00 \ text {} \ Omega + 6.00 \ text {} \ Omega + 13.0 \ text {} \ Omega \\ & = & 20.0 \ text {} \ Omega \ end {array} \\ [/ latex].

Стратегия и решение для (b)

Ток определяется по закону Ома, В = IR . Ввод значения приложенного напряжения и общего сопротивления дает ток для цепи:

[латекс] I = \ frac {V} {{R} _ {\ text {s}}} = \ frac {12.0 \ text {V}} {20.0 \ text {} \ Omega} = 0.60 \ text {A }\\[/латекс].

Стратегия и решение для (c)

Напряжение — или падение IR — на резисторе определяется законом Ома.Ввод тока и значения первого сопротивления дает

.

В 1 = IR 1 = (0,600 A) (1,0 Ом) = 0,600 В.

Аналогично

В 2 = IR 2 = (0,600 A) (6,0 Ом) = 3,60 В

и

V3 = IR 3 = (0,600 A) (13,0 Ом) = 7,80 В.

Обсуждение для (c)

Три капли IR добавляют к 12.0 В, прогноз:

В 1 + В 2 + В 3 = (0,600 + 3,60 + 7,80) В = 12,0 В.

Стратегия и решение для (d)

Самый простой способ рассчитать мощность в ваттах (Вт), рассеиваемую резистором в цепи постоянного тока, — это использовать закон Джоуля , P = IV , где P — электрическая мощность. В этом случае через каждый резистор протекает одинаковый полный ток.Подставляя закон Ома V = IR в закон Джоуля, мы получаем мощность, рассеиваемую первым резистором, как

P 1 = I 2 R 1 = (0,600 A) 2 (1,00 Ом) = 0,360 Вт

Аналогично

P 2 = I 2 R 2 = (0,600 A) 2 (6,00 Ом) = 2,16 Вт

и

P 3 = I 2 R 3 = (0.{2}} {R} \\ [/ latex], где В, — падение напряжения на резисторе (а не полное напряжение источника). Будут получены те же значения.

Стратегия и решение для (e)

Самый простой способ рассчитать выходную мощность источника — использовать P = IV , где В, — напряжение источника. Это дает

P = (0,600 A) (12,0 В) = 7,20 Вт.

Обсуждение для (e)

Обратите внимание, что по совпадению общая мощность, рассеиваемая резисторами, также равна 7.20 Вт, столько же, сколько мощность, выдаваемая источником. То есть

P 1 + P 2 + P 3 = (0,360 + 2,16 + 4,68) W = 7,20 Вт

Мощность — это энергия в единицу времени (ватты), поэтому для сохранения энергии требуется, чтобы выходная мощность источника была равна общей мощности, рассеиваемой резисторами.

Основные характеристики резисторов серии

  1. Последовательные сопротивления добавить: R с = R 1 + R 2 + R 3 +….
  2. Одинаковый ток протекает последовательно через каждый резистор.
  3. Отдельные последовательно включенные резисторы не получают полное напряжение источника, а делят его.

На рисунке 3 показаны резисторы параллельно , подключенные к источнику напряжения. Резисторы включены параллельно, когда каждый резистор подключен непосредственно к источнику напряжения с помощью соединительных проводов с незначительным сопротивлением. Таким образом, к каждому резистору приложено полное напряжение источника. Каждый резистор потребляет такой же ток, как если бы он один был подключен к источнику напряжения (при условии, что источник напряжения не перегружен).Например, автомобильные фары, радио и т. Д. Подключены параллельно, так что они используют полное напряжение источника и могут работать полностью независимо. То же самое и в вашем доме, или в любом другом здании. (См. Рисунок 3 (b).)

Рис. 3. (a) Три резистора, подключенных параллельно батарее, и эквивалентное одиночное или параллельное сопротивление. (б) Электроснабжение в доме. (Источник: Dmitry G, Wikimedia Commons)

Чтобы найти выражение для эквивалентного параллельного сопротивления R p , давайте рассмотрим протекающие токи и их связь с сопротивлением.Поскольку каждый резистор в цепи имеет полное напряжение, токи, протекающие через отдельные резисторы, равны [латекс] {I} _ {1} = \ frac {V} {{R} _ {1}} \\ [/ latex] , [латекс] {I} _ {2} = \ frac {V} {{R} _ {2}} \\ [/ latex] и [латекс] {I} _ {3} = \ frac {V} {{R} _ {3}} \\ [/ латекс]. Сохранение заряда подразумевает, что полный ток I , производимый источником, является суммой этих токов:

I = I 1 + I 2 + I 3 .

Подстановка выражений для отдельных токов дает

[латекс] I = \ frac {V} {{R} _ {1}} + \ frac {V} {{R} _ {2}} + \ frac {V} {{R} _ {3}} = V \ left (\ frac {1} {{R} _ {1}} + \ frac {1} {{R} _ {2}} + \ frac {1} {{R} _ {3}} \ справа) \\ [/ латекс].

Обратите внимание, что закон Ома для эквивалентного одиночного сопротивления дает

[латекс] I = \ frac {V} {{R} _ {p}} = V \ left (\ frac {1} {{R} _ {p}} \ right) \\ [/ latex].

Члены в круглых скобках в последних двух уравнениях должны быть равны. Обобщая для любого количества резисторов, общее сопротивление R p параллельного соединения связано с отдельными сопротивлениями на

[латекс] \ frac {1} {{R} _ {p}} = \ frac {1} {{R} _ {1}} + \ frac {1} {{R} _ {2}} + \ гидроразрыв {1} {{R} _ {\ text {.} 3}} + \ text {.} \ Text {…} \\ [/ latex]

Это соотношение приводит к общему сопротивлению R p , которое меньше наименьшего из отдельных сопротивлений. (Это видно в следующем примере.) При параллельном подключении резисторов от источника течет больше тока, чем протекает по любому из них по отдельности, поэтому общее сопротивление ниже.

Пример 2. Расчет сопротивления, тока, рассеиваемой мощности и выходной мощности: анализ параллельной цепи

Пусть выходное напряжение батареи и сопротивления в параллельном соединении на Рисунке 3 будут такими же, как в ранее рассмотренном последовательном соединении: В = 12.0 В, R 1 = 1,00 Ом, R 2 = 6,00 Ом и R 3 = 13,0 Ом. а) Каково полное сопротивление? (б) Найдите полный ток. (c) Рассчитайте токи в каждом резисторе и покажите, как они складываются, чтобы равняться общему выходному току источника. (d) Рассчитайте мощность, рассеиваемую каждым резистором. (e) Найдите выходную мощность источника и покажите, что она равна общей мощности, рассеиваемой резисторами.

Стратегия и решение для (а)

Общее сопротивление для параллельной комбинации резисторов находится с помощью следующего уравнения.Ввод известных значений дает

[латекс] \ frac {1} {{R} _ {p}} = \ frac {1} {{R} _ {1}} + \ frac {1} {{R} _ {2}} + \ frac {1} {{R} _ {3}} = \ frac {1} {1 \ text {.} \ text {00} \ text {} \ Omega} + \ frac {1} {6 \ text {. } \ text {00} \ text {} \ Omega} + \ frac {1} {\ text {13} \ text {.} 0 \ text {} \ Omega} \\ [/ latex].

Таким образом,

[латекс] \ frac {1} {{R} _ {p}} = \ frac {1.00} {\ text {} \ Omega} + \ frac {0 \ text {.} \ Text {1667}} {\ текст {} \ Omega} + \ frac {0 \ text {.} \ text {07692}} {\ text {} \ Omega} = \ frac {1 \ text {.} \ text {2436}} {\ text { } \ Omega} \\ [/ латекс].

(Обратите внимание, что в этих вычислениях каждый промежуточный ответ отображается с дополнительной цифрой.) Мы должны перевернуть это, чтобы найти полное сопротивление R p . Это дает

[латекс] {R} _ {\ text {p}} = \ frac {1} {1 \ text {.} \ Text {2436}} \ text {} \ Omega = 0 \ text {.} \ Text { 8041} \ text {} \ Omega \\ [/ latex].

Общее сопротивление с правильным количеством значащих цифр составляет R p = 0,804 Ом

Обсуждение для (а)

R p , как и предполагалось, меньше наименьшего индивидуального сопротивления.

Стратегия и решение для (b)

Полный ток можно найти из закона Ома, заменив полное сопротивление R p . Это дает

[латекс] I = \ frac {V} {{R} _ {\ text {p}}} = \ frac {\ text {12.0 V}} {0.8041 \ text {} \ Omega} = \ text {14} \ text {.} \ text {92 A} \\ [/ latex].

Обсуждение для (б)

Ток I для каждого устройства намного больше, чем для тех же устройств, подключенных последовательно (см. Предыдущий пример).Схема с параллельным соединением имеет меньшее общее сопротивление, чем резисторы, включенные последовательно.

Стратегия и решение для (c)

Отдельные токи легко вычислить по закону Ома, поскольку каждый резистор получает полное напряжение. Таким образом,

[латекс] {I} _ {1} = \ frac {V} {{R} _ {1}} = \ frac {12.0 \ text {V}} {1.00 \ text {} \ Omega} = 12.0 \ text {A} \\ [/ латекс].

Аналогично

[латекс] {I} _ {2} = \ frac {V} {{R} _ {2}} = \ frac {12.0 \ text {V}} {6.00 \ text {} \ Omega} = 2 \ text {.} \ text {00} \ text {A} \\ [/ latex]

и

[латекс] {I} _ {3} = \ frac {V} {{R} _ {3}} = \ frac {\ text {12} \ text {.} 0 \ text {V}} {\ text {13} \ text {.} \ Text {0} \ text {} \ Omega} = 0 \ text {.} \ Text {92} \ text {A} \\ [/ latex].

Обсуждение для (c)

Общий ток складывается из отдельных токов:

I 1 + I 2 + I 3 = 14,92 A.

Это соответствует сохранению заряда.{2}} {13.0 \ text {} \ Omega} = 11.1 \ text {W} \\ [/ latex].

Обсуждение для (d)

Мощность, рассеиваемая каждым резистором при параллельном подключении, значительно выше, чем при последовательном подключении к тому же источнику напряжения.

Стратегия и решение для (e)

Общую мощность также можно рассчитать несколькими способами. Выбрав P = IV и введя полный ток, получим

P = IV = (14,92 A) (12,0 В) = 179 Вт.

Обсуждение для (e)

Общая мощность, рассеиваемая резисторами, также 179 Вт:

P 1 + P 2 + P 3 = 144 Вт + 24,0 Вт + 11,1 Вт = 179 Вт

Это соответствует закону сохранения энергии.

Общее обсуждение

Обратите внимание, что как токи, так и мощность при параллельном подключении больше, чем для тех же устройств, подключенных последовательно.

Основные характеристики параллельных резисторов

  1. Параллельное сопротивление определяется из [latex] \ frac {1} {{R} _ {\ text {p}}} = \ frac {1} {{R} _ {1}} + \ frac {1} { {R} _ {2}} + \ frac {1} {{R} _ {3}} + \ text {…} \\ [/ latex], и оно меньше любого отдельного сопротивления в комбинации.
  2. На каждый параллельно включенный резистор подается такое же полное напряжение источника. (В системах распределения электроэнергии чаще всего используются параллельные соединения для питания бесчисленных устройств, обслуживаемых одним и тем же напряжением, и для того, чтобы они могли работать независимо.)
  3. Параллельные резисторы не получают суммарный ток каждый; они делят это.

Сочетания последовательного и параллельного

Более сложные соединения резисторов иногда представляют собой просто комбинации последовательного и параллельного. Они часто встречаются, особенно если учесть сопротивление провода. В этом случае сопротивление провода включено последовательно с другими сопротивлениями, включенными параллельно. Комбинации последовательного и параллельного подключения можно свести к одному эквивалентному сопротивлению, используя методику, показанную на рисунке 4.Различные части идентифицируются как последовательные или параллельные, уменьшаются до их эквивалентов и далее уменьшаются до тех пор, пока не останется единственное сопротивление. Процесс занимает больше времени, чем труден.

Рис. 4. Эта комбинация из семи резисторов имеет как последовательные, так и параллельные части. Каждый из них идентифицируется и приводится к эквивалентному сопротивлению, а затем уменьшается до тех пор, пока не будет достигнуто одно эквивалентное сопротивление.

Самая простая комбинация последовательного и параллельного сопротивления, показанная на рисунке 4, также является наиболее поучительной, поскольку она используется во многих приложениях.Например, R 1 может быть сопротивлением проводов от автомобильного аккумулятора к его электрическим устройствам, которые подключены параллельно. R 2 и R 3 могут быть стартером и светом салона. Ранее мы предполагали, что сопротивление провода незначительно, но, когда это не так, оно имеет важные последствия, как показывает следующий пример.

Пример 3. Расчет сопротивления,

IR Падение, ток и рассеиваемая мощность: объединение последовательных и параллельных цепей

На рис. 5 показаны резисторы из двух предыдущих примеров, подключенные другим способом — сочетание последовательного и параллельного.Мы можем считать R 1 сопротивлением проводов, ведущих к R 2 и R 3 . (а) Найдите полное сопротивление. (b) Каково падение IR в R 1 ? (c) Найдите текущие значения от I 2 до R 2 . (d) Какую мощность рассеивает R 2 ?

Рис. 5. Эти три резистора подключены к источнику напряжения, так что R 2 и R 3 параллельны друг другу, и эта комбинация последовательно с R 1 .

Стратегия и решение для (а)

Чтобы найти полное сопротивление, отметим, что R 2 и R 3 подключены параллельно, а их комбинация R p последовательно с R 1 . Таким образом, полное (эквивалентное) сопротивление этой комбинации составляет

.

R до = R 1 + R p .

Сначала находим R p , используя уравнение для параллельных резисторов и вводя известные значения:

[латекс] \ frac {1} {{R} _ {\ text {p}}} = \ frac {1} {{R} _ {2}} + \ frac {1} {{R} _ {3 }} = \ frac {1} {6 \ text {.} \ text {00} \ text {} \ Omega} + \ frac {1} {\ text {13} \ text {.} 0 \ text {} \ Omega} = \ frac {0.2436} {\ text {} \ Омега} \\ [/ латекс].

Инвертирование дает

[латекс] {R} _ {\ text {p}} = \ frac {1} {0,2436} \ text {} \ Omega = 4.11 \ text {} \ Omega \\ [/ latex].

Таким образом, общее сопротивление равно

.

R до = R 1 + R p = 1,00 Ом + 4,11 Ом = 5,11 Ом.

Обсуждение для (а)

Общее сопротивление этой комбинации является промежуточным между значениями чистой серии и чистой параллели (20.0 Ом и 0,804 Ом соответственно), найденные для тех же резисторов в двух предыдущих примерах.

Стратегия и решение для (b)

Чтобы найти падение IR в R 1 , отметим, что полный ток I протекает через R 1 . Таким образом, его IR падение составляет

В 1 = ИК 1

Мы должны найти I , прежде чем сможем вычислить V 1 .Полный ток I находится с помощью закона Ома для цепи. То есть

[латекс] I = \ frac {V} {{R} _ {\ text {tot}}} = \ frac {\ text {12.0} \ text {V}} {5.11 \ text {} \ Omega} = 2.35 \ text {A} \\ [/ latex].

Вводя это в выражение выше, мы получаем

В 1 = IR 1 = (2,35 A) (1,00 Ом) = 2,35 В.

Обсуждение для (б)

Напряжение, приложенное к R 2 и R 3 , меньше общего напряжения на величину В 1 .Когда сопротивление провода велико, это может существенно повлиять на работу устройств, представленных R 2 и R 3 .

Стратегия и решение для (c)

Чтобы найти ток через R 2 , мы должны сначала найти приложенное к нему напряжение. Мы называем это напряжение В p , потому что оно приложено к параллельной комбинации резисторов. Напряжение, приложенное как к R 2 , так и к R 3 , уменьшается на величину В 1 , и поэтому оно составляет

V p = V V 1 = 12.0 В — 2,35 В = 9,65 В.

Теперь ток I 2 через сопротивление R 2 находится по закону Ома:

[латекс] {I} _ {2} = \ frac {{V} _ {\ text {p}}} {{R} _ {2}} = \ frac {9.65 \ text {V}} {6.00 \ текст {} \ Omega} = 1,61 \ text {A} \\ [/ latex].

Обсуждение для (c)

Ток меньше 2,00 А, которые протекали через R 2 , когда он был подключен параллельно к батарее в предыдущем примере параллельной цепи.

Стратегия и решение для (d)

Мощность, рассеиваемая на R 2 определяется на

P 2 = ( I 2 ) 2 R 2 = (1,61 A) 2 (6,00 Ом) = 15,5 Вт

Обсуждение для (d)

Мощность меньше 24,0 Вт, рассеиваемых этим резистором при параллельном подключении к источнику 12,0 В.

Одним из следствий этого последнего примера является то, что сопротивление в проводах снижает ток и мощность, подаваемую на резистор.Если сопротивление провода относительно велико, как в изношенном (или очень длинном) удлинителе, то эти потери могут быть значительными. Если потребляется большой ток, падение IR в проводах также может быть значительным.

Например, когда вы роетесь в холодильнике и включается мотор, свет холодильника на мгновение гаснет. Точно так же вы можете увидеть тусклый свет в салоне, когда вы запускаете двигатель вашего автомобиля (хотя это может быть связано с сопротивлением внутри самой батареи).

Что происходит в этих сильноточных ситуациях, показано на рисунке 6. Устройство, обозначенное номером R 3 , имеет очень низкое сопротивление, поэтому при его включении протекает большой ток. Этот повышенный ток вызывает большее падение IR в проводах, представленных R 1 , уменьшая напряжение на лампочке (которое составляет R 2 ), которое затем заметно гаснет.

Рис. 6. Почему гаснет свет, когда включен большой прибор? Ответ заключается в том, что большой ток, потребляемый двигателем прибора, вызывает значительное падение напряжения в проводах и снижает напряжение на свету.

Проверьте свое понимание

Можно ли любую произвольную комбинацию резисторов разбить на последовательную и параллельную? Посмотрите, сможете ли вы нарисовать принципиальную схему резисторов, которые нельзя разбить на комбинации последовательно и параллельно.

Раствор

Нет, есть много способов подключения резисторов, которые не являются комбинациями последовательного и параллельного, включая петли и переходы. В таких случаях правила Кирхгофа, которые будут включены в Правила Кирхгофа, позволят вам проанализировать схему.

Стратегии решения проблем для последовательных и параллельных резисторов

  1. Нарисуйте четкую принципиальную схему, пометив все резисторы и источники напряжения. Этот шаг включает в себя список известных проблем, поскольку они отмечены на вашей принципиальной схеме.
  2. Определите, что именно необходимо определить в проблеме (определите неизвестные). Письменный список полезен.
  3. Определите, включены ли резисторы последовательно, параллельно или в комбинации последовательно и параллельно.Изучите принципиальную схему, чтобы сделать эту оценку. Резисторы включены последовательно, если через них должен последовательно проходить один и тот же ток.
  4. Используйте соответствующий список основных функций для последовательных или параллельных подключений, чтобы найти неизвестные. Есть один список для серий, а другой — для параллелей. Если ваша проблема представляет собой комбинацию последовательного и параллельного соединения, уменьшайте ее поэтапно, рассматривая отдельные группы последовательных или параллельных соединений, как это сделано в этом модуле и примерах. Особое примечание: при обнаружении R необходимо соблюдать осторожность.
  5. Проверьте, являются ли ответы разумными и последовательными. Единицы и числовые результаты должны быть разумными. Общее последовательное сопротивление должно быть больше, а общее параллельное сопротивление, например, должно быть меньше. Мощность должна быть больше для одних и тех же устройств, подключенных параллельно, по сравнению с последовательными и т. Д.

Сводка раздела

Концептуальные вопросы

1. Переключатель имеет переменное сопротивление, близкое к нулю в замкнутом состоянии и очень большое в разомкнутом, и он включен последовательно с устройством, которым он управляет.Объясните влияние переключателя на рис. 7 на ток в разомкнутом и замкнутом состоянии.

Рис. 7. Переключатель обычно включается последовательно с источником сопротивления и напряжения. В идеале переключатель имеет почти нулевое сопротивление в замкнутом состоянии, но имеет чрезвычайно большое сопротивление в разомкнутом состоянии. (Обратите внимание, что на этой диаграмме скрипт E представляет напряжение (или электродвижущую силу) батареи.)

2. Какое напряжение на разомкнутом переключателе на Рисунке 7?

3. На разомкнутом переключателе есть напряжение, как на Рисунке 7.Почему же тогда мощность, рассеиваемая разомкнутым переключателем, мала?

4. Почему мощность, рассеиваемая замкнутым переключателем, как на Рисунке 7, мала?

5. Студент в физической лаборатории по ошибке подключил электрическую лампочку, батарею и выключатель, как показано на рисунке 8. Объясните, почему лампочка горит, когда выключатель разомкнут, и гаснет, когда выключатель замкнут. (Не пытайтесь — батарея сильно разряжается!)

Рис. 8. Ошибка подключения. Включите этот переключатель параллельно устройству, обозначенному [латекс] R [/ латекс].(Обратите внимание, что на этой диаграмме скрипт E представляет напряжение (или электродвижущую силу) батареи.)

6. Зная, что сила электрического шока зависит от величины тока, протекающего через ваше тело, вы бы предпочли, чтобы он был включен последовательно или параллельно с сопротивлением, таким как нагревательный элемент тостера, если он шокирован им? Объяснять.

7. Были бы ваши фары тусклыми при запуске двигателя автомобиля, если бы провода в вашем автомобиле были сверхпроводниками? (Не пренебрегайте внутренним сопротивлением батареи.) Объяснять.

8. Некоторые гирлянды праздничных огней соединены последовательно для экономии затрат на проводку. В старой версии использовались лампочки, которые при перегорании разрывают электрическое соединение, как выключатель. Если одна такая лампочка перегорит, что случится с остальными? Если такая цепочка работает от 120 В и имеет 40 одинаковых лампочек, каково нормальное рабочее напряжение каждой? В более новых версиях используются лампы, которые при перегорании замыкаются накоротко, как замкнутый выключатель. Если одна такая лампочка перегорит, что случится с остальными? Если такая цепочка работает от 120 В и в ней осталось 39 идентичных лампочек, каково тогда рабочее напряжение каждой?

9.Если две бытовые лампочки мощностью 60 и 100 Вт подключить последовательно к бытовой электросети, какая из них будет ярче? Объяснять.

10. Предположим, вы проводите физическую лабораторию, в которой вас просят вставить резистор в цепь, но все прилагаемые резисторы имеют большее сопротивление, чем запрошенное значение. Как бы вы соединили доступные сопротивления, чтобы попытаться получить меньшее запрошенное значение?

11. Перед Второй мировой войной некоторые радиостанции получали питание через «шнур сопротивления», который имел значительное сопротивление.Такой резистивный шнур снижает напряжение до желаемого уровня для ламп радиоприемника и т.п., и это экономит расходы на трансформатор. Объясните, почему шнуры сопротивления нагреваются и тратят энергию при включенном радио.

12. У некоторых лампочек есть три уровня мощности (не включая ноль), получаемые от нескольких нитей накала, которые индивидуально переключаются и соединяются параллельно. Какое минимальное количество нитей нити необходимо для трех режимов мощности?

Задачи и упражнения

Примечание. Можно считать, что данные, взятые из цифр, имеют точность до трех значащих цифр.

1. (а) Каково сопротивление десяти последовательно соединенных резисторов сопротивлением 275 Ом? (б) Параллельно?

2. (a) Каково сопротивление последовательно соединенных резисторов 1,00 × 10 2 Ом, 2,50 кОм и 4,00 кОм? (б) Параллельно?

3. Какое наибольшее и наименьшее сопротивление можно получить, соединив резисторы на 36,0 Ом, 50,0 Ом и 700 Ом?

4. Тостер на 1800 Вт, электрическая сковорода на 1400 Вт и лампа на 75 Вт подключены к одной розетке в цепи 15 А, 120 В.(Три устройства работают параллельно, если подключены к одной розетке.) а) Какой ток потребляет каждое устройство? (b) Перегорит ли эта комбинация предохранитель на 15 А?

5. Фара мощностью 30,0 Вт и стартер мощностью 2,40 кВт обычно подключаются параллельно в систему на 12,0 В. Какую мощность потребляли бы одна фара и стартер при последовательном подключении к батарее 12,0 В? (Не обращайте внимания на любое другое сопротивление в цепи и любое изменение сопротивления в двух устройствах.)

6.(a) Для батареи 48,0 В и резисторов 24,0 Ом и 96,0 Ом найдите для каждого из них ток и мощность при последовательном соединении. (b) Повторите, когда сопротивления включены параллельно.

7. Ссылаясь на пример комбинирования последовательных и параллельных цепей и рисунок 5, вычислите I 3 двумя следующими способами: (a) по известным значениям I и I 2 ; (б) используя закон Ома для R 3 . В обеих частях явно показано, как вы следуете шагам, описанным в описании стратегии решения проблем для последовательных и параллельных резисторов выше.

Рис. 5. Эти три резистора подключены к источнику напряжения, так что R 2 и R 3 параллельны друг другу, и эта комбинация последовательно с R 1 .

8. Ссылаясь на рисунок 5: (a) Вычислите P 3 и обратите внимание на его сравнение с P 3 , найденным в первых двух примерах задач в этом модуле. (b) Найдите полную мощность, отдаваемую источником, и сравните ее с суммой мощностей, рассеиваемых резисторами.

9. См. Рисунок 6 и обсуждение затемнения света при включении тяжелого прибора. (а) Учитывая, что источник напряжения составляет 120 В, сопротивление провода составляет 0,400 Ом, а номинальная мощность лампы составляет 75,0 Вт, какая мощность будет рассеиваться лампой, если при включении двигателя через провода пройдет в общей сложности 15,0 А? Предположите незначительное изменение сопротивления лампы. б) Какая мощность потребляет двигатель?

Рис. 6. Почему гаснет свет, когда включен большой прибор? Ответ заключается в том, что большой ток, потребляемый двигателем прибора, вызывает значительное падение напряжения в проводах и снижает напряжение на свету.

10. Линия электропередачи 240 кВ, имеющая 5,00 × 10 2 , подвешена к заземленным металлическим опорам с помощью керамических изоляторов, каждый из которых имеет сопротивление 1,00 × 10 9 Ом (рис. 9 (а)). Какое сопротивление на землю у 100 изоляторов? (b) Рассчитайте мощность, рассеиваемую 100 из них. (c) Какая доля мощности, переносимой линией, составляет это? Ясно покажите, как вы следуете шагам, описанным в описании стратегии решения проблем для последовательных и параллельных резисторов выше.

Рис. 9. Высоковольтная (240 кВ) линия электропередачи 5,00 × 10 2 подвешена к заземленной металлической опоре электропередачи. Ряд керамических изоляторов обеспечивает сопротивление 1,00 × 10 9 Ом каждый.

11. Покажите, что если два резистора R 1 и R 2 объединены, и один из них намного больше другого ( R 1 >> R 2 ): (a ) Их последовательное сопротивление почти равно большему сопротивлению R 1 .(b) Их параллельное сопротивление почти равно меньшему сопротивлению R 2 .

12. Необоснованные результаты Два резистора, один из которых имеет сопротивление 145 Ом, подключены параллельно, чтобы получить общее сопротивление 150 Ом. а) Каково значение второго сопротивления? б) Что неразумного в этом результате? (c) Какие предположения необоснованны или непоследовательны?

13. Необоснованные результаты Два резистора, один из которых имеет сопротивление 900 кОм, соединены последовательно, чтобы получить общее сопротивление 0.500 МОм. а) Каково значение второго сопротивления? б) Что неразумного в этом результате? (c) Какие предположения необоснованны или непоследовательны?

Глоссарий

серия:
последовательность резисторов или других компонентов, включенных в цепь один за другим
резистор:
компонент, обеспечивающий сопротивление току, протекающему через электрическую цепь
сопротивление:
, вызывая потерю электроэнергии в цепи
Закон Ома:
соотношение между током, напряжением и сопротивлением в электрической цепи: В = IR
напряжение:
электрическая потенциальная энергия на единицу заряда; электрическое давление, создаваемое источником питания, например аккумулятором
падение напряжения:
потеря электроэнергии при прохождении тока через резистор, провод или другой компонент
ток:
поток заряда через электрическую цепь мимо заданной точки измерения
Закон Джоуля:
соотношение между потенциальной электрической мощностью, напряжением и сопротивлением в электрической цепи, определяемое следующим образом: [latex] {P} _ {e} = \ text {IV} [/ latex]
параллельно:
подключение резисторов или других компонентов в электрической цепи таким образом, что каждый компонент получает одинаковое напряжение от источника питания; часто изображается на диаграмме в виде лестницы, где каждый компонент находится на ступеньке лестницы

Избранные решения проблем и упражнения

1.(а) 2,75 кОм (б) 27,5 Ом

3. (а) 786 Ом (б) 20,3 Ом

5. 29,6 Вт

7. (а) 0,74 А (б) 0,742 А

9. (а) 60,8 Вт (б) 3,18 кВт

11. (a) [латекс] \ begin {array} {} {R} _ {\ text {s}} = {R} _ {1} + {R} _ {2} \\ \ Rightarrow {R} _ {\ text {s}} \ приблизительно {R} _ {1} \ left ({R} _ {1} \ text {>>} {R} _ {2} \ right) \ end {array} \\ [/ латекс]

(b) [латекс] \ frac {1} {{R} _ {p}} = \ frac {1} {{R} _ {1}} + \ frac {1} {{R} _ {2} } = \ frac {{R} _ {1} + {R} _ {2}} {{R} _ {1} {R} _ {2}} \\ [/ latex],

, так что

[латекс] \ begin {array} {} {R} _ {p} = \ frac {{R} _ {1} {R} _ {2}} {{R} _ {1} + {R} _ {2}} \ приблизительно \ frac {{R} _ {1} {R} _ {2}} {{R} _ {1}} = {R} _ {2} \ left ({R} _ {1 } \ text {>>} {R} _ {2} \ right) \ text {.} \ end {array} \\ [/ latex]

13. (a) –400 кОм (b) Сопротивление не может быть отрицательным. (c) Считается, что последовательное сопротивление меньше, чем у одного из резисторов, но должно быть больше, чем у любого из резисторов.

6.2 Последовательные и параллельные резисторы — Введение в электричество, магнетизм и схемы

ЦЕЛИ ОБУЧЕНИЯ

По окончании раздела вы сможете:

  • Определите термин эквивалентное сопротивление
  • Рассчитайте эквивалентное сопротивление резисторов, включенных последовательно
  • Рассчитайте эквивалентное сопротивление резисторов, включенных параллельно

В статье «Ток и сопротивление» мы описали термин «сопротивление» и объяснили основную конструкцию резистора.По сути, резистор ограничивает поток заряда в цепи и представляет собой омическое устройство, где. В большинстве схем имеется более одного резистора. Если несколько резисторов соединены вместе и подключены к батарее, ток, подаваемый батареей, зависит от эквивалентного сопротивления цепи.

Эквивалентное сопротивление комбинации резисторов зависит как от их индивидуальных значений, так и от способа их подключения. Самыми простыми комбинациями резисторов являются последовательное и параллельное соединение (рисунок 6.2.1). В последовательной цепи выходной ток первого резистора течет на вход второго резистора; следовательно, ток в каждом резисторе одинаков. В параллельной схеме все выводы резистора на одной стороне резисторов соединены вместе, а все выводы на другой стороне соединены вместе. В случае параллельной конфигурации каждый резистор имеет одинаковое падение потенциала на нем, и токи через каждый резистор могут быть разными, в зависимости от резистора.Сумма отдельных токов равна току, протекающему по параллельным соединениям.

(рисунок 6.2.1)

Рисунок 6.2.1 (a) При последовательном соединении резисторов ток одинаков в каждом резисторе. (b) При параллельном соединении резисторов напряжение на каждом резисторе одинаковое.

Резисторы серии

Считается, что резисторы

включены последовательно, когда ток течет через резисторы последовательно. Рассмотрим рисунок 6.2.2, на котором показаны три последовательно включенных резистора с приложенным напряжением, равным.Поскольку заряды проходят только по одному пути, ток через каждый резистор одинаков. Эквивалентное сопротивление набора резисторов при последовательном соединении равно алгебраической сумме отдельных сопротивлений.

(рисунок 6.2.2)

Рисунок 6.2.2 (a) Три резистора, подключенные последовательно к источнику напряжения. (b) Исходная схема сокращается до эквивалентного сопротивления и источника напряжения.

На рисунке 6.2.2 ток, исходящий от источника напряжения, протекает через каждый резистор, поэтому ток через каждый резистор одинаков.Ток в цепи зависит от напряжения, подаваемого источником напряжения, и сопротивления резисторов. Для каждого резистора происходит падение потенциала, равное потере электрической потенциальной энергии при прохождении тока через каждый резистор. Согласно закону Ома падение потенциала на резисторе при протекании через него тока рассчитывается по формуле, где — ток в амперах (), а — сопротивление в омах (). Поскольку энергия сохраняется, а напряжение равно потенциальной энергии на заряд, сумма напряжения, приложенного к цепи источником, и падения потенциала на отдельных резисторах вокруг контура должны быть равны нулю:

Это уравнение часто называют законом петли Кирхгофа, который мы рассмотрим более подробно позже в этой главе.На рисунке 6.2.2 сумма падения потенциала каждого резистора и напряжения, подаваемого источником напряжения, должна равняться нулю:

Поскольку ток через каждый компонент одинаков, равенство можно упростить до эквивалентного сопротивления, которое представляет собой просто сумму сопротивлений отдельных резисторов.

Любое количество резисторов может быть подключено последовательно. Если резисторы соединены последовательно, эквивалентное сопротивление составляет

Ом.

(6.2.1)

Одним из результатов подключения компонентов в последовательную цепь является то, что если что-то происходит с одним компонентом, это влияет на все остальные компоненты. Например, если несколько ламп подключены последовательно и одна лампа перегорела, все остальные лампы погаснут.

ПРИМЕР 6.2.1


Эквивалентное сопротивление, ток и мощность в последовательной цепи

Батарея с напряжением на клеммах подключена к цепи, состоящей из четырех и одного последовательно соединенных резисторов (рисунок 6.2.3). Предположим, что батарея имеет незначительное внутреннее сопротивление. (а) Рассчитайте эквивалентное сопротивление цепи. (b) Рассчитайте ток через каждый резистор. (c) Рассчитайте падение потенциала на каждом резисторе. (d) Определите общую мощность, рассеиваемую резисторами, и мощность, потребляемую батареей.

(рисунок 6.2.3)

Рисунок 6.2.3 Простая последовательная схема с пятью резисторами.

Стратегия

В последовательной цепи эквивалентное сопротивление представляет собой алгебраическую сумму сопротивлений.Ток в цепи можно найти из закона Ома и равен напряжению, деленному на эквивалентное сопротивление. Падение потенциала на каждом резисторе можно найти с помощью закона Ома. Мощность, рассеиваемая каждым резистором, может быть найдена с помощью, а общая мощность, рассеиваемая резисторами, равна сумме мощности, рассеиваемой каждым резистором. Мощность, подаваемую аккумулятором, можно найти с помощью.

Решение

а. Эквивалентное сопротивление — это алгебраическая сумма сопротивлений:

г.Ток в цепи одинаков для каждого резистора в последовательной цепи и равен приложенному напряжению, деленному на эквивалентное сопротивление:

г. Падение потенциала на каждом резисторе можно найти с помощью закона Ома:

.

Обратите внимание, что сумма падений потенциала на каждом резисторе равна напряжению, подаваемому батареей.

г. Мощность, рассеиваемая резистором, равна, а мощность, отдаваемая батареей, равна:

Значение

Есть несколько причин, по которым мы использовали бы несколько резисторов вместо одного резистора с сопротивлением, равным эквивалентному сопротивлению цепи.Возможно, резистора необходимого размера нет в наличии, или нам нужно отводить выделяемое тепло, или мы хотим минимизировать стоимость резисторов. Каждый резистор может стоить от нескольких центов до нескольких долларов, но при умножении на тысячи единиц экономия затрат может быть значительной.

ПРОВЕРЬТЕ ПОНИМАНИЕ 6.2

Некоторые гирлянды миниатюрных праздничных огней закорачиваются при перегорании лампочки. Устройство, вызывающее короткое замыкание, называется шунтом, который позволяет току течь по разомкнутой цепи.«Короткое замыкание» похоже на протягивание куска проволоки через компонент. Луковицы обычно сгруппированы в серии по девять луковиц. Если перегорает слишком много лампочек, в конце концов открываются шунты. Что вызывает это?

Кратко обозначим основные характеристики последовательно соединенных резисторов:

    Сопротивления серии

  1. суммируются, чтобы получить эквивалентное сопротивление:

  2. Одинаковый ток протекает последовательно через каждый резистор.
  3. Отдельные последовательно включенные резисторы не получают полное напряжение источника, а делят его.Общее падение потенциала на последовательной конфигурации резисторов равно сумме падений потенциала на каждом резисторе.

Параллельные резисторы

На рисунке 6.2.4 показаны резисторы, включенные параллельно, подключенные к источнику напряжения. Резисторы включены параллельно, когда один конец всех резисторов соединен непрерывным проводом с незначительным сопротивлением, а другой конец всех резисторов также соединен друг с другом непрерывным проводом с незначительным сопротивлением.Падение потенциала на каждом резисторе одинаковое. Ток через каждый резистор можно найти с помощью закона Ома, где напряжение на каждом резисторе постоянно. Например, автомобильные фары, радио и другие системы подключены параллельно, так что каждая подсистема использует полное напряжение источника и может работать полностью независимо. То же самое и с электропроводкой в ​​вашем доме или любом здании.

(рисунок 6.2.4)

Рисунок 6.2.4 (a) Два резистора, подключенных параллельно источнику напряжения.(b) Исходная схема сокращается до эквивалентного сопротивления и источника напряжения.

Ток, протекающий от источника напряжения на рисунке 6.2.4, зависит от напряжения, подаваемого источником напряжения, и эквивалентного сопротивления цепи. В этом случае ток течет от источника напряжения и попадает в переход или узел, где цепь разделяется, протекая через резисторы и. По мере того, как заряды проходят от батареи, некоторые проходят через резистор, а некоторые — через резистор. Сумма токов, текущих в переход, должна быть равна сумме токов, текущих из перехода:

Это уравнение называется правилом соединения Кирхгофа и будет подробно обсуждено в следующем разделе.На рисунке 6.2.4 показано правило соединения. В этой схеме есть две петли, которые приводят к уравнениям и Обратите внимание, что напряжение на резисторах, включенных параллельно, одинаковое (), а ток является аддитивным:

Обобщая для любого количества резисторов, эквивалентное сопротивление параллельного соединения связано с отдельными сопротивлениями на

(6.2.2)

Это соотношение приводит к эквивалентному сопротивлению, которое меньше наименьшего из отдельных сопротивлений.Когда резисторы подключены параллельно, от источника течет больше тока, чем протекает для любого из них по отдельности, поэтому общее сопротивление ниже.

ПРИМЕР 6.2.2


Анализ параллельной цепи

Три резистора, и соединены параллельно. Параллельное соединение подключается к источнику напряжения. а) Какое эквивалентное сопротивление? (б) Найдите ток, подаваемый источником в параллельную цепь. (c) Рассчитайте токи в каждом резисторе и покажите, что в сумме они равны выходному току источника.(d) Рассчитайте мощность, рассеиваемую каждым резистором. (e) Найдите выходную мощность источника и покажите, что она равна общей мощности, рассеиваемой резисторами.

Стратегия

(a) Общее сопротивление для параллельной комбинации резисторов определяется с помощью.
(Обратите внимание, что в этих расчетах каждый промежуточный ответ отображается с дополнительной цифрой.)

(b) Ток, подаваемый источником, можно найти из закона Ома, заменив полное сопротивление.

(c) Отдельные токи легко вычислить по закону Ома, поскольку каждый резистор получает полное напряжение.Полный ток — это сумма отдельных токов:.

(d) Мощность, рассеиваемую каждым резистором, можно найти с помощью любого из уравнений, связывающих мощность с током, напряжением и сопротивлением, поскольку все три известны. Давайте использовать, так как каждый резистор получает полное напряжение.

(e) Полная мощность также может быть рассчитана несколькими способами, используйте.

Решение

а. Общее сопротивление для параллельной комбинации резисторов находится с помощью уравнения 6.2.2.Ввод известных значений дает

Общее сопротивление с правильным количеством значащих цифр составляет. Как и предполагалось, меньше минимального индивидуального сопротивления.

г. Полный ток можно найти из закона Ома, заменив полное сопротивление. Это дает

Ток для каждого устройства намного больше, чем для тех же устройств, подключенных последовательно (см. Предыдущий пример). Схема с параллельным соединением имеет меньшее общее сопротивление, чем резисторы, включенные последовательно.

г. Отдельные токи легко вычислить по закону Ома, поскольку на каждый резистор подается полное напряжение. Таким образом,

Аналогично

и

Общий ток складывается из отдельных токов:

г. Мощность, рассеиваемую каждым резистором, можно найти с помощью любого из уравнений, связывающих мощность с током, напряжением и сопротивлением, поскольку все три известны.Давайте использовать, так как каждый резистор получает полное напряжение. Таким образом,

Аналогично

и

e. Суммарную мощность также можно рассчитать несколькими способами. Выбор и ввод общей текущей доходности

Значение

Общая мощность, рассеиваемая резисторами, также:

Обратите внимание, что общая мощность, рассеиваемая резисторами, равна мощности, подаваемой источником.

ПРОВЕРЬТЕ ПОНИМАНИЕ 6.3


Рассмотрим одинаковую разность потенциалов, приложенную к одним и тем же трем последовательно включенным резисторам. Будет ли эквивалентное сопротивление последовательной цепи больше, меньше или равно трем резисторам, включенным параллельно? Будет ли ток в последовательной цепи выше, ниже или равен току, обеспечиваемому тем же напряжением, приложенным к параллельной цепи? Как мощность, рассеиваемая последовательно подключенными резисторами, будет сравниваться с мощностью, рассеиваемой параллельно резисторами?

ПРОВЕРЬТЕ ПОНИМАНИЕ 6.4


Как бы вы использовали реку и два водопада, чтобы смоделировать параллельную конфигурацию двух резисторов? Как разрушается эта аналогия?

Суммируем основные характеристики резисторов параллельно:

  1. Эквивалентное сопротивление найдено из

    и меньше любого отдельного сопротивления в комбинации.

  2. Падение потенциала на каждом параллельном резисторе одинаковое.
  3. Параллельные резисторы не получают суммарный ток каждый; они делят это.Ток, поступающий в параллельную комбинацию резисторов, равен сумме токов, протекающих через каждый резистор, включенный параллельно.

В этой главе мы представили эквивалентное сопротивление резисторов, соединенных последовательно, и резисторов, соединенных параллельно. Вы можете вспомнить, что в разделе «Емкость» мы ввели эквивалентную емкость конденсаторов, соединенных последовательно и параллельно. Цепи часто содержат как конденсаторы, так и резисторы. В таблице 6.2.1 приведены уравнения, используемые для эквивалентного сопротивления и эквивалентной емкости для последовательных и параллельных соединений.

(таблица 6.2.1)

Комбинация серий Параллельная комбинация
Эквивалентная емкость
Эквивалентное сопротивление

Таблица 10.1 Сводка по эквивалентному сопротивлению и емкости в последовательной и параллельной комбинациях

Сочетания последовательного и параллельного

Более сложные соединения резисторов часто представляют собой просто комбинации последовательного и параллельного соединения.Такие комбинации обычны, особенно если учесть сопротивление проводов. В этом случае сопротивление провода включено последовательно с другими сопротивлениями, включенными параллельно.

Последовательные и параллельные комбинации можно уменьшить до одного эквивалентного сопротивления, используя методику, показанную на Рисунке 6.2.5. Различные части могут быть идентифицированы как последовательные или параллельные соединения, уменьшенные до их эквивалентных сопротивлений, а затем уменьшенные до тех пор, пока не останется единственное эквивалентное сопротивление. Процесс занимает больше времени, чем труден.Здесь мы отмечаем эквивалентное сопротивление как.

(рисунок 6.2.5)

Обратите внимание, что резисторы и включены последовательно. Их можно объединить в одно эквивалентное сопротивление. Один из методов отслеживания процесса — включить резисторы в качестве индексов. Здесь эквивалентное сопротивление и равно

.

Теперь схема сокращается до трех резисторов, показанных на Рисунке 6.2.5 (c). Перерисовывая, мы теперь видим, что резисторы и составляют параллельную цепь.Эти два резистора можно уменьшить до эквивалентного сопротивления:

Этот шаг процесса сокращает схему до двух резисторов, показанных на Рисунке 6.2.5 (d). Здесь схема сводится к двум резисторам, которые в данном случае включены последовательно. Эти два резистора можно уменьшить до эквивалентного сопротивления, которое является эквивалентным сопротивлением цепи:

Основная цель этого анализа схемы достигнута, и теперь схема сводится к одному резистору и одному источнику напряжения.

Теперь мы можем проанализировать схему. Ток, обеспечиваемый источником напряжения, равен. Этот ток проходит через резистор и обозначен как. Падение потенциала можно найти с помощью закона Ома:

Глядя на рис. 6.2.5 (c), остается отбросить параллельную комбинацию и. Проходной ток можно найти с помощью закона Ома:

Резисторы и включены последовательно, поэтому токи и равны

.

Используя закон Ома, мы можем найти падение потенциала на двух последних резисторах.Потенциальные падения равны и. Окончательный анализ — это посмотреть на мощность, подаваемую источником напряжения, и мощность, рассеиваемую резисторами. Мощность, рассеиваемая резисторами

Общая энергия постоянна в любом процессе. Следовательно, мощность, подаваемая источником напряжения, равна. Анализ мощности, подаваемой в схему, и мощности, рассеиваемой резисторами, является хорошей проверкой достоверности анализа; они должны быть равны.

ПРОВЕРЬТЕ ПОНИМАНИЕ 6.5


Рассмотрите электрические цепи в вашем доме. Приведите хотя бы два примера схем, в которых для эффективной работы необходимо использовать комбинацию последовательных и параллельных схем.

Практическое применение

Одним из следствий этого последнего примера является то, что сопротивление в проводах снижает ток и мощность, подаваемую на резистор. Если сопротивление провода относительно велико, как в изношенном (или очень длинном) удлинителе, то эти потери могут быть значительными. Если протекает большой ток, провал в проводах также может быть значительным и проявляться в виде тепла, выделяемого в шнуре.

Например, когда вы роетесь в холодильнике и включается мотор, свет холодильника на мгновение гаснет. Точно так же вы можете увидеть тусклый свет в салоне, когда вы запускаете двигатель вашего автомобиля (хотя это может быть связано с сопротивлением внутри самой батареи).

Что происходит в этих сильноточных ситуациях, показано на Рисунке 6.2.7. Устройство, представленное значком, имеет очень низкое сопротивление, поэтому при его включении протекает большой ток.Этот увеличенный ток вызывает большее падение в проводах, представленных значком, уменьшая напряжение на лампочке (которая есть), которое затем заметно гаснет.

(рисунок 6.2.7)

Рисунок 6.2.7 Почему свет тускнеет, когда включен большой прибор? Ответ заключается в том, что большой ток, потребляемый двигателем прибора, вызывает значительное падение напряжения в проводах и снижает напряжение на свету.

Стратегия решения проблем: последовательные и параллельные резисторы


  1. Нарисуйте четкую принципиальную схему, пометив все резисторы и источники напряжения.Этот шаг включает список известных значений проблемы, поскольку они отмечены на вашей принципиальной схеме.
  2. Определите, что именно необходимо определить в проблеме (определите неизвестные). Письменный список полезен.
  3. Определите, включены ли резисторы последовательно, параллельно или в комбинации последовательно и параллельно. Изучите принципиальную схему, чтобы сделать эту оценку. Резисторы включены последовательно, если через них должен последовательно проходить один и тот же ток.
  4. Используйте соответствующий список основных функций для последовательных или параллельных подключений, чтобы найти неизвестные.Есть один список для серий, а другой — для параллелей.
  5. Проверьте, являются ли ответы разумными и последовательными.

ПРИМЕР 6.2.4


Объединение последовательных и параллельных цепей

Два резистора, соединенных последовательно, подключены к двум резисторам, включенным параллельно. Последовательно-параллельная комбинация подключается к батарее. Каждый резистор имеет сопротивление. Провода, соединяющие резисторы и аккумулятор, имеют незначительное сопротивление.Ток проходит через резистор. Какое напряжение подается от источника напряжения?

Стратегия

Используйте шаги предыдущей стратегии решения проблем, чтобы найти решение для этого примера.

Решение
  1. Нарисуйте четкую принципиальную схему (рисунок 6.2.8).

    (рисунок 6.2.8)

    Рисунок 6.2.8 Чтобы найти неизвестное напряжение, мы должны сначала найти эквивалентное сопротивление цепи.

  2. Неизвестно напряжение аккумулятора.Чтобы определить напряжение, подаваемое батареей, необходимо найти эквивалентное сопротивление.
  3. В этой схеме мы уже знаем, что резисторы и включены последовательно, а резисторы и включены параллельно. Эквивалентное сопротивление параллельной конфигурации резисторов и последовательно с последовательной конфигурацией резисторов и.
  4. Напряжение, подаваемое батареей, можно найти, умножив ток от батареи на эквивалентное сопротивление цепи.Ток от батареи равен току через него и равен. Нам нужно найти эквивалентное сопротивление, уменьшив схему. Чтобы уменьшить схему, сначала рассмотрите два резистора, включенных параллельно. Эквивалентное сопротивление составляет. Эта параллельная комбинация включена последовательно с двумя другими резисторами, поэтому эквивалентное сопротивление цепи равно. Таким образом, напряжение, подаваемое батареей, составляет.
  5. Один из способов проверить соответствие ваших результатов — это рассчитать мощность, подаваемую батареей, и мощность, рассеиваемую резисторами.Мощность, подаваемая аккумулятором, составляет

    Поскольку они включены последовательно, сквозной ток равен сквозному току. Т.к. ток через каждый будет. Мощность, рассеиваемая резисторами, равна сумме мощности, рассеиваемой каждым резистором:

    Поскольку мощность, рассеиваемая резисторами, равна мощности, выделяемой батареей, наше решение кажется последовательным.

Значение

Если проблема имеет комбинацию последовательного и параллельного соединения, как в этом примере, ее можно уменьшить поэтапно, используя предыдущую стратегию решения проблемы и рассматривая отдельные группы последовательных или параллельных соединений.При поиске параллельного подключения необходимо соблюдать осторожность. Кроме того, единицы и числовые результаты должны быть разумными. Эквивалентное последовательное сопротивление должно быть больше, а эквивалентное параллельное сопротивление, например, должно быть меньше. Мощность должна быть больше для одних и тех же устройств, подключенных параллельно, по сравнению с последовательными и т. Д.

Кандела Цитаты

Лицензионный контент CC, особая атрибуция

  • Загрузите бесплатно по адресу http: // cnx.org/contents/[email protected]. Получено с : http://cnx.org/contents/[email protected]. Лицензия : CC BY: Attribution

Расчет тока в последовательно-параллельных цепях

Расчет тока в последовательно-параллельных цепях


Рисунок 1. Последовательно-параллельные резисторы.

В цепи с резисторами, включенными как последовательно, так и
параллельно, рассматривать схему как комбинацию параллельных частей и последовательных
части.

Используйте формулы сопротивления для определения общего сопротивления
серийных и параллельных частей. Затем используйте закон Ома, чтобы вычислить напряжение
падает поперек и токи через каждую часть.

В схеме на Рисунке 1 сначала используйте параллельное сопротивление
формула для определения эквивалентного сопротивления R 123 .

Тогда формула последовательного сопротивления говорит нам R TOT = R 123 + R 4 .Итак, закон Ома дает полный ток цепи:

I TOT равняется как текущим I 4 хотя R 4 и текущему I 123 вход / выход из параллельной части. Используя закон Ома:

Итак, зная значение В 123 , снова применим закон Ома, чтобы найти ток I 1 через параллельный резистор R 1 :

и аналогично для I 2 и I 3 .

Пример

Для схемы на рисунке 1 предположим, что E = 9 В, R 1 = 500 Ом, R 2 = 1,0 кОм, R 3 = 1,5 кОм и R 4 = 220 Ом. Тогда R 123 = 273 Ом и R TOT = 493 Ом, поэтому

и, следовательно,

Аналогично для I 2 и I 3 .

Примечание: Важно осторожно обращаться с единицами измерения с метрическими префиксами. Выше мы отрегулировали десятичную точку и единицы измерения так, чтобы наша формула для тока давала вольт / Ом = ампер.

10.3: Последовательные и параллельные резисторы

Цели обучения

К концу раздела вы сможете:

  • Определите термин эквивалентное сопротивление
  • Рассчитайте эквивалентное сопротивление резисторов, включенных последовательно
  • Рассчитайте эквивалентное сопротивление резисторов, включенных параллельно

В статье «Ток и сопротивление» мы описали термин «сопротивление» и объяснили основную конструкцию резистора.По сути, резистор ограничивает поток заряда в цепи и представляет собой омическое устройство, где \ (V = IR \). В большинстве схем имеется более одного резистора. Если несколько резисторов соединены вместе и подключены к батарее, ток, подаваемый батареей, зависит от эквивалентного сопротивления цепи.

Эквивалентное сопротивление комбинации резисторов зависит как от их индивидуальных значений, так и от способа их подключения. Самыми простыми комбинациями резисторов являются последовательное и параллельное соединение (Рисунок \ (\ PageIndex {1} \)).В последовательной цепи выходной ток первого резистора течет на вход второго резистора; следовательно, ток в каждом резисторе одинаков. В параллельной схеме все выводы резистора на одной стороне резисторов соединены вместе, а все выводы на другой стороне соединены вместе. В случае параллельной конфигурации каждый резистор имеет одинаковое падение потенциала на нем, и токи через каждый резистор могут быть разными, в зависимости от резистора.Сумма отдельных токов равна току, протекающему по параллельным соединениям.

Рисунок \ (\ PageIndex {1} \): (a) При последовательном соединении резисторов ток одинаков в каждом резисторе. (b) При параллельном соединении резисторов напряжение на каждом резисторе одинаковое.

Резисторы серии

Считается, что резисторы

включены последовательно, когда ток течет через резисторы последовательно. Рассмотрим рисунок \ (\ PageIndex {2} \), на котором показаны три последовательно включенных резистора с приложенным напряжением, равным \ (V_ {ab} \).Поскольку заряды проходят только по одному пути, ток через каждый резистор одинаков. Эквивалентное сопротивление набора резисторов при последовательном соединении равно алгебраической сумме отдельных сопротивлений.

Рисунок \ (\ PageIndex {2} \): (a) Три резистора, подключенные последовательно к источнику напряжения. (b) Исходная схема сокращается до эквивалентного сопротивления и источника напряжения.

На рисунке \ (\ PageIndex {2} \) ток, идущий от источника напряжения, протекает через каждый резистор, поэтому ток через каждый резистор одинаков.Ток в цепи зависит от напряжения, подаваемого источником напряжения, и сопротивления резисторов. Для каждого резистора происходит падение потенциала, равное потере электрической потенциальной энергии при прохождении тока через каждый резистор. Согласно закону Ома, падение потенциала \ (V \) на резисторе при протекании через него тока рассчитывается по формуле \ (V = IR \), где \ (I \) — ток в амперах (\ (A \)), а \ (R \) — сопротивление в Ом \ ((\ Omega) \).N V_i = 0. \]

Это уравнение часто называют законом петли Кирхгофа, который мы рассмотрим более подробно позже в этой главе. На рисунке \ (\ PageIndex {2} \) сумма падения потенциала каждого резистора и напряжения, подаваемого источником напряжения, должна равняться нулю:

\ [\ begin {align *} V — V_1 — V_2 — V_3 & = 0, \\ [4pt] V & = V_1 + V_2 + V_3, \\ [4pt] & = IR_1 + IR_2 + IR_3, \ end { выровнять *} \]

Решение для \ (I \)

\ [\ begin {align *} I & = \ frac {V} {R_1 + R_2 + R_3} \\ [4pt] & = \ frac {V} {R_ {S}}.\ end {align *} \]

Поскольку ток через каждый компонент одинаков, равенство можно упростить до эквивалентного сопротивления (\ (R_ {S} \)), которое представляет собой просто сумму сопротивлений отдельных резисторов.

Эквивалентное сопротивление в последовательной цепи

Любое количество резисторов может быть подключено последовательно. Если \ (N \) резисторы соединены последовательно, эквивалентное сопротивление равно

. N R_i.\ label {серия эквивалентных сопротивлений} \]

Одним из результатов подключения компонентов в последовательную цепь является то, что если что-то происходит с одним компонентом, это влияет на все остальные компоненты. Например, если несколько ламп подключены последовательно и одна лампа перегорела, все остальные лампы погаснут.

Пример \ (\ PageIndex {1} \): эквивалентное сопротивление, ток и мощность в последовательной цепи

Батарея с напряжением на клеммах 9 В подключена к цепи, состоящей из четырех последовательно соединенных резисторов \ (20 \, \ Omega \) и одного \ (10 ​​\, \ Omega \) (Рисунок \ (\ PageIndex {3 } \)).Предположим, что батарея имеет незначительное внутреннее сопротивление.

  1. Рассчитайте эквивалентное сопротивление цепи.
  2. Рассчитайте ток через каждый резистор.
  3. Рассчитайте падение потенциала на каждом резисторе.
  4. Определите общую мощность, рассеиваемую резисторами, и мощность, потребляемую батареей.

Рисунок \ (\ PageIndex {3} \): Простая последовательная схема с пятью резисторами.

Стратегия

В последовательной цепи эквивалентное сопротивление представляет собой алгебраическую сумму сопротивлений.2R \), а общая мощность, рассеиваемая резисторами, равна сумме мощности, рассеиваемой каждым резистором. Мощность, подаваемая батареей, можно найти с помощью \ (P = I \ epsilon \).

Решение

  1. Эквивалентное сопротивление — это алгебраическая сумма сопротивлений (уравнение \ ref {серия эквивалентных сопротивлений}): \ [\ begin {align *} R_ {S} & = R_1 + R_2 + R_3 + R_4 + R_5 \\ [4pt ] & = 20 \, \ Омега + 20 \, \ Омега + 20 \, \ Омега + 20 \, \ Омега + 10 \, \ Омега = 90 \, \ Омега.2 (10 \, \ Omega) = 0,1 \, W, \ nonumber \] \ [P_ {рассеивается} = 0,2 \, W + 0,2 \, W + 0,2 \, W + 0,2 \, W + 0,1 \, W = 0,9 \, W, \ nonumber \] \ [P_ {источник} = I \ epsilon = (0,1 \, A) (9 \, V) = 0,9 \, W. \ nonumber \]

Значение

Есть несколько причин, по которым мы использовали бы несколько резисторов вместо одного резистора с сопротивлением, равным эквивалентному сопротивлению цепи. Возможно, резистора необходимого размера нет в наличии, или нам нужно отводить выделяемое тепло, или мы хотим минимизировать стоимость резисторов.Каждый резистор может стоить от нескольких центов до нескольких долларов, но при умножении на тысячи единиц экономия затрат может быть значительной.

Упражнение \ (\ PageIndex {1} \)

Некоторые гирлянды миниатюрных праздничных огней закорачиваются при перегорании лампочки. Устройство, вызывающее короткое замыкание, называется шунтом, который позволяет току течь по разомкнутой цепи. «Короткое замыкание» похоже на протягивание куска проволоки через компонент. Луковицы обычно сгруппированы в серии по девять луковиц.Если перегорает слишком много лампочек, в конце концов открываются шунты. Что вызывает это?

Ответ

Эквивалентное сопротивление девяти последовательно соединенных лампочек составляет 9 R . Ток равен \ (I = V / 9 \, R \). Если одна лампочка перегорит, эквивалентное сопротивление будет 8 R , и напряжение не изменится, но ток возрастет \ ((I = V / 8 \, R \). Чем больше лампочек перегорят, ток станет равным. В конце концов, ток становится слишком большим, что приводит к сгоранию шунта.№ Р_и. \]

  • Одинаковый ток протекает последовательно через каждый резистор.
  • Отдельные последовательно включенные резисторы не получают полное напряжение источника, а делят его. Общее падение потенциала на последовательной конфигурации резисторов равно сумме падений потенциала на каждом резисторе.
  • Параллельные резисторы

    На рисунке \ (\ PageIndex {4} \) показаны резисторы, включенные параллельно, подключенные к источнику напряжения. Резисторы включены параллельно, когда один конец всех резисторов соединен непрерывным проводом с незначительным сопротивлением, а другой конец всех резисторов также соединен друг с другом непрерывным проводом с незначительным сопротивлением.Падение потенциала на каждом резисторе одинаковое. Ток через каждый резистор можно найти с помощью закона Ома \ (I = V / R \), где напряжение на каждом резисторе постоянно. Например, автомобильные фары, радио и другие системы подключены параллельно, так что каждая подсистема использует полное напряжение источника и может работать полностью независимо. То же самое и с электропроводкой в ​​вашем доме или любом здании.

    Рисунок \ (\ PageIndex {4} \): Два резистора, подключенных параллельно источнику напряжения.(b) Исходная схема сокращается до эквивалентного сопротивления и источника напряжения.

    Ток, протекающий от источника напряжения на рисунке \ (\ PageIndex {4} \), зависит от напряжения, подаваемого источником напряжения, и эквивалентного сопротивления цепи. В этом случае ток течет от источника напряжения и попадает в переход или узел, где цепь разделяется, протекая через резисторы \ (R_1 \) и \ (R_2 \). По мере того, как заряды идут от аккумулятора, некоторые проходят через резистор \ (R_1 \), а некоторые — через резистор \ (R_2 \).Сумма токов, текущих в переход, должна быть равна сумме токов, текущих из перехода:

    \ [\ sum I_ {in} = \ sum I_ {out}. {- 1}.{-1}. \ label {10.3} \]

    Это соотношение приводит к эквивалентному сопротивлению \ (R_ {P} \), которое меньше наименьшего из отдельных сопротивлений. Когда резисторы подключены параллельно, от источника течет больше тока, чем протекает для любого из них по отдельности, поэтому общее сопротивление ниже.

    Пример \ (\ PageIndex {2} \): Анализ параллельной цепи

    Три резистора \ (R_1 = 1,00 \, \ Omega \), \ (R_2 = 2,00 \, \ Omega \) и \ (R_3 = 2,00 \, \ Omega \) подключены параллельно.Параллельное соединение подключается к источнику напряжения \ (V = 3,00 \, V \).

    1. Какое эквивалентное сопротивление?
    2. Найдите ток, подаваемый источником в параллельную цепь.
    3. Рассчитайте токи в каждом резисторе и покажите, что в сумме они равны выходному току источника.
    4. Рассчитайте мощность, рассеиваемую каждым резистором.
    5. Найдите выходную мощность источника и покажите, что она равна общей мощности, рассеиваемой резисторами.

    Стратегия

    (a) Общее сопротивление для параллельной комбинации резисторов определяется с помощью уравнения \ ref {10.3}. (Обратите внимание, что в этих расчетах каждый промежуточный ответ отображается с дополнительной цифрой.)

    (b) Ток, подаваемый источником, можно найти из закона Ома, заменив \ (R_ {P} \) на полное сопротивление \ (I = \ frac {V} {R_ {P}} \).

    (c) Отдельные токи легко вычислить по закону Ома \ (\ left (I_i = \ frac {V_i} {R_i} \ right) \), поскольку каждый резистор получает полное напряжение.{-1} = 0,50 \, \ Omega. \ Nonumber \] Общее сопротивление с правильным количеством значащих цифр равно \ (R_ {eq} = 0,50 \, \ Omega \). Как и предполагалось, \ (R_ {P} \) меньше наименьшего индивидуального сопротивления.

  • Полный ток можно найти из закона Ома, заменив полное сопротивление \ (R_ {P} \). Это дает \ [I = \ frac {V} {R_ {P}} = \ frac {3.00 \, V} {0.50 \, \ Omega} = 6.00 \, A. \ nonumber \] Текущий I для каждого устройства намного больше, чем для тех же устройств, подключенных последовательно (см. предыдущий пример).Схема с параллельным соединением имеет меньшее общее сопротивление, чем резисторы, включенные последовательно.
  • Отдельные токи легко вычислить по закону Ома, поскольку каждый резистор получает полное напряжение. Таким образом, \ [I_1 = \ frac {V} {R_1} = \ frac {3.00 \, V} {1.00 \, \ Omega} = 3.00 \, A. \ nonumber \] Аналогично, \ [I_2 = \ frac {V } {R_2} = \ frac {3.00 \, V} {2.00 \, \ Omega} = 1.50 \, A \ nonumber \] и \ [I_3 = \ frac {V} {R_3} = \ frac {3.00 \, V } {2.00 \, \ Omega} = 1.50 \, A. \ nonumber \] Полный ток — это сумма отдельных токов: \ [I_1 + I_2 + I_3 = 6.2} {2.00 \, \ Omega} = 4.50 \, W. \ nonumber \]
  • Общую мощность также можно рассчитать несколькими способами. Выбор \ (P = IV \) и ввод общей текущей доходности \ [P = IV = (6.00 \, A) (3.00 \, V) = 18.00 \, W. \ nonumber \]
  • Значение

    Общая мощность, рассеиваемая резисторами, также 18,00 Вт:

    \ [P_1 + P_2 + P_3 = 9,00 \, W + 4,50 \, W + 4,50 \, W = 18,00 \, W. \ nonumber \]

    Обратите внимание, что общая мощность, рассеиваемая резисторами, равна мощности, подаваемой источником.

    Упражнение \ (\ PageIndex {2A} \)

    Рассмотрим одну и ту же разность потенциалов \ ((V = 3,00 \, V) \), приложенную к одним и тем же трем последовательно включенным резисторам. Будет ли эквивалентное сопротивление последовательной цепи больше, меньше или равно трем резисторам, включенным параллельно? Будет ли ток в последовательной цепи выше, ниже или равен току, обеспечиваемому тем же напряжением, приложенным к параллельной цепи? Как мощность, рассеиваемая последовательно подключенными резисторами, будет сравниваться с мощностью, рассеиваемой параллельно резисторами?

    Решение

    Эквивалент последовательной схемы будет \ (R_ {eq} = 1.00 \, \ Omega + 2.00 \, \ Omega + 2.00 \, \ Omega = 5.00 \, \ Omega \), что выше эквивалентного сопротивления параллельной цепи \ (R_ {eq} = 0.50 \, \ Omega \ ). Эквивалентный резистор любого количества резисторов всегда выше, чем эквивалентное сопротивление тех же резисторов, соединенных параллельно. Ток через последовательную цепь будет \ (I = \ frac {3.00 \, V} {5.00 \, \ Omega} = 0.60 \, A \), что меньше суммы токов, проходящих через каждый резистор в параллельная цепь, \ (I = 6.00 \, А \). Это неудивительно, поскольку эквивалентное сопротивление последовательной цепи выше. Ток при последовательном соединении любого количества резисторов всегда будет ниже, чем ток при параллельном соединении тех же резисторов, поскольку эквивалентное сопротивление последовательной цепи будет выше, чем параллельной цепи. Мощность, рассеиваемая последовательно включенными резисторами, будет равна \ (P = 1,800 \, Вт \), что ниже мощности, рассеиваемой в параллельной цепи \ (P = 18.00 \, Вт \).

    Упражнение \ (\ PageIndex {2B} \)

    Как бы вы использовали реку и два водопада, чтобы смоделировать параллельную конфигурацию двух резисторов? Как разрушается эта аналогия?

    Решение

    Река, текущая горизонтально с постоянной скоростью, разделяется на две части и течет через два водопада. Молекулы воды аналогичны электронам в параллельных цепях. Количество молекул воды, которые текут в реке и падает, должно быть равно количеству молекул, которые текут над каждым водопадом, точно так же, как сумма тока через каждый резистор должна быть равна току, текущему в параллельном контуре.Молекулы воды в реке обладают энергией благодаря своему движению и высоте. Потенциальная энергия молекул воды в реке постоянна из-за их одинаковой высоты. Это аналогично постоянному изменению напряжения в параллельной цепи. Напряжение — это потенциальная энергия на каждом резисторе.

    При рассмотрении энергии аналогия быстро разрушается. В водопаде потенциальная энергия преобразуется в кинетическую энергию молекул воды. В случае прохождения электронов через резистор падение потенциала преобразуется в тепло и свет, а не в кинетическую энергию электронов.

    Суммируем основные характеристики резисторов параллельно:

    1. Эквивалентное сопротивление находится по формуле \ ref {10.3} и меньше любого отдельного сопротивления в комбинации.
    2. Падение потенциала на каждом параллельном резисторе одинаковое.
    3. Параллельные резисторы не получают суммарный ток каждый; они делят это. Ток, поступающий в параллельную комбинацию резисторов, равен сумме токов, протекающих через каждый резистор, включенный параллельно.

    В этой главе мы представили эквивалентное сопротивление резисторов, соединенных последовательно, и резисторов, соединенных параллельно. Как вы помните, в разделе «Емкость» мы ввели эквивалентную емкость конденсаторов, соединенных последовательно и параллельно. Цепи часто содержат как конденсаторы, так и резисторы. Таблица \ (\ PageIndex {1} \) суммирует уравнения, используемые для эквивалентного сопротивления и эквивалентной емкости для последовательных и параллельных соединений.

    Таблица \ (\ PageIndex {1} \): Сводка по эквивалентному сопротивлению и емкости в последовательной и параллельной комбинациях
    Комбинация серий Параллельная комбинация
    Эквивалентная емкость \ [\ frac {1} {C_ {S}} = \ frac {1} {C_1} + \ frac {1} {C_2} + \ frac {1} {C_3} +.N R_i \ nonumber \] \ [\ frac {1} {R_ {P}} = \ frac {1} {R_1} + \ frac {1} {R_2} + \ frac {1} {R_3} +. . . \ nonumber \]

    Сочетания последовательного и параллельного

    Более сложные соединения резисторов часто представляют собой просто комбинации последовательного и параллельного соединения. Такие комбинации обычны, особенно если учесть сопротивление проводов. В этом случае сопротивление провода включено последовательно с другими сопротивлениями, включенными параллельно.

    Комбинации последовательного и параллельного соединения могут быть уменьшены до одного эквивалентного сопротивления, используя метод, показанный на рисунке \ (\ PageIndex {5} \).Различные части могут быть идентифицированы как последовательные или параллельные соединения, уменьшенные до их эквивалентных сопротивлений, а затем уменьшенные до тех пор, пока не останется единственное эквивалентное сопротивление. Процесс занимает больше времени, чем труден. Здесь мы отмечаем эквивалентное сопротивление как \ (R_ {eq} \).

    Рисунок \ (\ PageIndex {5} \): (а) Исходная схема из четырех резисторов. (b) Шаг 1: резисторы \ (R_3 \) и \ (R_4 \) включены последовательно, и эквивалентное сопротивление равно \ (R_ {34} = 10 \, \ Omega \). (c) Шаг 2: сокращенная схема показывает, что резисторы \ (R_2 \) и \ (R_ {34} \) включены параллельно, с эквивалентным сопротивлением \ (R_ {234} = 5 \, \ Omega \).(d) Шаг 3: сокращенная схема показывает, что \ (R_1 \) и \ (R_ {234} \) включены последовательно с эквивалентным сопротивлением \ (R_ {1234} = 12 \, \ Omega \), которое является эквивалентное сопротивление \ (R_ {eq} \). (e) Уменьшенная схема с источником напряжения \ (V = 24 \, V \) с эквивалентным сопротивлением \ (R_ {eq} = 12 \, \ Omega \). Это приводит к току \ (I = 2 \, A \) от источника напряжения.

    Обратите внимание, что резисторы \ (R_3 \) и \ (R_4 \) включены последовательно. Их можно объединить в одно эквивалентное сопротивление. {- 1} = 5 \, \ Omega.\ nonumber \]

    Этот шаг процесса сокращает схему до двух резисторов, показанных на рисунке \ (\ PageIndex {5d} \). Здесь схема сводится к двум резисторам, которые в данном случае включены последовательно. Эти два резистора можно уменьшить до эквивалентного сопротивления, которое является эквивалентным сопротивлением цепи:

    \ [R_ {eq} = R_ {1234} = R_1 + R_ {234} = 7 \, \ Omega + 5 \ Omega = 12 \, \ Omega. \ nonumber \]

    Основная цель этого анализа схемы достигнута, и теперь схема сводится к одному резистору и одному источнику напряжения.

    Теперь мы можем проанализировать схему. Ток, обеспечиваемый источником напряжения, равен \ (I = \ frac {V} {R_ {eq}} = \ frac {24 \, V} {12 \, \ Omega} = 2 \, A \). Этот ток проходит через резистор \ (R_1 \) и обозначается как \ (I_1 \). Падение потенциала на \ (R_1 \) можно найти с помощью закона Ома:

    \ [V_1 = I_1R_1 = (2 \, A) (7 \, \ Omega) = 14 \, V. \ nonumber \]

    Глядя на рисунок \ (\ PageIndex {5c} \), это оставляет \ (24 \, V — 14 \, V = 10 \, V \) отбрасывать через параллельную комбинацию \ (R_2 \) и \ ( R_ {34} \).Ток через \ (R_2 \) можно найти по закону Ома:

    \ [I_2 = \ frac {V_2} {R_2} = \ frac {10 \, V} {10 \, \ Omega} = 1 \, A. \ nonumber \]

    Резисторы \ (R_3 \) и \ (R_4 \) включены последовательно, поэтому токи \ (I_3 \) и \ (I_4 \) равны

    .

    \ [I_3 = I_4 = I — I_2 = 2 \, A — 1 \, A = 1 \, A. \ nonumber \]

    Используя закон Ома, мы можем найти падение потенциала на двух последних резисторах. Потенциальные капли равны \ (V_3 = I_3R_3 = 6 \, V \) и \ (V_4 = I_4R_4 = 4 \, V \).2 (4 \, \ Omega) = 4 \, W, \\ [4pt] P_ {рассеивается} & = P_1 + P_2 + P_3 + P_4 = 48 \, W. \ end {align *} \]

    Общая энергия постоянна в любом процессе. Следовательно, мощность, подаваемая источником напряжения, составляет

    \ [\ begin {align *} P_s & = IV \\ [4pt] & = (2 \, A) (24 \, V) = 48 \, W \ end {align *} \]

    Анализ мощности, подаваемой в схему, и мощности, рассеиваемой резисторами, является хорошей проверкой достоверности анализа; они должны быть равны.

    Пример \ (\ PageIndex {3} \): объединение последовательных и параллельных цепей

    На рисунке \ (\ PageIndex {6} \) показаны резисторы, подключенные последовательно и параллельно.Мы можем считать \ (R_1 \) сопротивлением проводов, ведущих к \ (R_2 \) и \ (R_3 \).

    1. Найдите эквивалентное сопротивление цепи.
    2. Какое падение потенциала \ (V_1 \) на резисторе \ (R_1 \)?
    3. Найдите ток \ (I_2 \) через резистор \ (R_2 \).
    4. Какая мощность рассеивается \ (R_2 \)?

    Рисунок \ (\ PageIndex {6} \): Эти три резистора подключены к источнику напряжения так, чтобы \ (R_2 \) и \ (R_3 \) были параллельны друг другу, и эта комбинация была последовательно с \ (R_1 \).

    Стратегия

    (a) Чтобы найти эквивалентное сопротивление, сначала найдите эквивалентное сопротивление параллельного соединения \ (R_2 \) и \ (R_3 \). Затем используйте этот результат, чтобы найти эквивалентное сопротивление последовательного соединения с \ (R_1 \).

    (b) Ток через \ (R_1 \) можно найти с помощью закона Ома и приложенного напряжения. Ток через \ (R_1 \) равен току от батареи. Падение потенциала \ (V_1 \) на резисторе \ (R_1 \) (которое представляет собой сопротивление в соединительных проводах) можно найти с помощью закона Ома.{-1} = 5.10 \, \ Omega. \ Nonumber \] Общее сопротивление этой комбинации является промежуточным между значениями чистой серии и чисто параллельной (\ (20.0 \, \ Omega \) и \ (0.804 \, \ Omega \) ), соответственно).

  • Ток через \ (R_1 \) равен току, обеспечиваемому батареей: \ [I_1 = I = \ frac {V} {R_ {eq}} = \ frac {12.0 \, V} {5.10 \, \ Omega} = 2.35 \, A. \ nonumber \] Напряжение на \ (R_1 \) равно \ [V_1 = I_1R_1 = (2.35 \, A) (1 \, \ Omega) = 2.35 \, V. \ nonumber \] Напряжение, приложенное к \ (R_2 \) и \ (R_3 \), меньше напряжения, подаваемого батареей, на величину \ (V_1 \).Когда сопротивление провода велико, это может существенно повлиять на работу устройств, представленных \ (R_2 \) и \ (R_3 \).
  • Чтобы найти ток через \ (R_2 \), мы должны сначала найти приложенное к нему напряжение. Напряжение на двух параллельных резисторах одинаково: \ [V_2 = V_3 = V — V_1 = 12.0 \, V — 2.35 \, V = 9.65 \, V. \ nonumber \] Теперь мы можем найти ток \ (I_2 \) через сопротивление \ (R_2 \) по закону Ома: \ [I_2 = \ frac {V_2} {R_2} = \ frac {9.65 \, V} {6.00 \, \ Omega} = 1.2 (6.00 \, \ Omega) = 15.5 \, W. \ nonumber \]
  • Значение

    Анализ сложных схем часто можно упростить, сведя схему к источнику напряжения и эквивалентному сопротивлению. Даже если вся схема не может быть сведена к одному источнику напряжения и одному эквивалентному сопротивлению, части схемы могут быть уменьшены, что значительно упрощает анализ.

    Упражнение \ (\ PageIndex {3} \)

    Рассмотрите электрические цепи в вашем доме.Приведите хотя бы два примера схем, в которых для эффективной работы необходимо использовать комбинацию последовательных и параллельных схем.

    Решение

    Все цепи верхнего освещения параллельны и подключены к основному питанию, поэтому при перегорании одной лампочки все верхнее освещение не гаснет. У каждого верхнего света будет по крайней мере один переключатель, включенный последовательно с источником света, так что вы можете включать и выключать его.

    В холодильнике есть компрессор и лампа, которая загорается при открытии дверцы.Обычно у холодильника есть только один шнур для подключения к стене. Цепь, содержащая компрессор, и цепь, содержащая цепь освещения, параллельны, но есть переключатель, включенный последовательно со светом. Термостат управляет переключателем, который включен последовательно с компрессором, чтобы контролировать температуру холодильника.

    Практическое значение

    Одним из следствий этого последнего примера является то, что сопротивление в проводах снижает ток и мощность, подаваемую на резистор.Если сопротивление провода относительно велико, как в изношенном (или очень длинном) удлинителе, то эти потери могут быть значительными. Если потребляется большой ток, падение IR в проводах также может быть значительным и может проявляться из-за тепла, выделяемого в шнуре.

    Например, когда вы роетесь в холодильнике и включается мотор, свет холодильника на мгновение гаснет. Точно так же вы можете увидеть тусклый свет в салоне, когда вы запускаете двигатель вашего автомобиля (хотя это может быть связано с сопротивлением внутри самой батареи).

    Что происходит в этих сильноточных ситуациях, показано на рисунке \ (\ PageIndex {7} \). Устройство, обозначенное символом \ (R_3 \), имеет очень низкое сопротивление, поэтому при его включении протекает большой ток. Этот увеличенный ток вызывает большее падение IR в проводах, обозначенных \ (R_1 \), снижая напряжение на лампочке (которое равно \ (R_2 \)), которое затем заметно гаснет.

    Рисунок \ (\ PageIndex {7} \): Почему свет тускнеет, когда включен большой прибор? Ответ заключается в том, что большой ток, потребляемый двигателем прибора, вызывает значительное падение IR в проводах и снижает напряжение на свету.

    Стратегия решения проблем: последовательные и параллельные резисторы

    1. Нарисуйте четкую принципиальную схему, обозначив все резисторы и источники напряжения. Этот шаг включает список известных значений проблемы, поскольку они отмечены на вашей принципиальной схеме.
    2. Определите, что именно необходимо определить в проблеме (определите неизвестные). Письменный список полезен.
    3. Определите, включены ли резисторы последовательно, параллельно или в комбинации последовательно и параллельно.Изучите принципиальную схему, чтобы сделать эту оценку. Резисторы включены последовательно, если через них должен последовательно проходить один и тот же ток.
    4. Используйте соответствующий список основных функций для последовательных или параллельных подключений, чтобы найти неизвестные. Есть один список для серий, а другой — для параллелей.
    5. Проверьте, являются ли ответы разумными и последовательными.

    Пример \ (\ PageIndex {4} \): объединение последовательных и параллельных цепей

    Два резистора, соединенных последовательно \ ((R_1, \, R_2) \), соединены с двумя резисторами, включенными параллельно \ ((R_3, \, R_4) \).Последовательно-параллельная комбинация подключается к батарее. Каждый резистор имеет сопротивление 10,00 Ом. Провода, соединяющие резисторы и аккумулятор, имеют незначительное сопротивление. Через резистор \ (R_1 \) проходит ток 2,00 А. Какое напряжение подается от источника напряжения?

    Стратегия

    Используйте шаги предыдущей стратегии решения проблем, чтобы найти решение для этого примера.

    Решение

    Рисунок \ (\ PageIndex {8} \): Чтобы найти неизвестное напряжение, мы должны сначала найти эквивалентное сопротивление цепи.

    1. Нарисуйте четкую принципиальную схему (Рисунок \ (\ PageIndex {8} \)).
    2. Неизвестно напряжение аккумулятора. Чтобы определить напряжение, подаваемое батареей, необходимо найти эквивалентное сопротивление.
    3. В этой схеме мы уже знаем, что резисторы \ (R_1 \) и \ (R_2 \) включены последовательно, а резисторы \ (R_3 \) и \ (R_4 \) включены параллельно. Эквивалентное сопротивление параллельной конфигурации резисторов \ (R_3 \) и \ (R_4 \) последовательно с последовательной конфигурацией резисторов \ (R_1 \) и \ (R_2 \).{-1} = 5,00 \, \ Омега. \ nonumber \] Эта параллельная комбинация включена последовательно с двумя другими резисторами, поэтому эквивалентное сопротивление схемы равно \ (R_ {eq} = R_1 + R_2 + R_ {34} = (25.00 \, \ Omega \). поэтому напряжение, подаваемое батареей, равно \ (V = IR_ {eq} = 2.00 \, A (25.00 \, \ Omega) = 50.00 \, V \).
    4. Один из способов проверить соответствие ваших результатов — это рассчитать мощность, подаваемую батареей, и мощность, рассеиваемую резисторами. Мощность, обеспечиваемая аккумулятором, равна \ (P_ {batt} = IV = 100.2R_4 \\ [4pt] & = 40.00 \, W + 40.00 \, W + 10.00 \, W + 10.00 \, W = 100. \, W. \ end {align *} \]

      Поскольку мощность, рассеиваемая резисторами, равна мощности, выделяемой батареей, наше решение кажется последовательным.

      Значение

      Если проблема имеет комбинацию последовательного и параллельного соединения, как в этом примере, ее можно уменьшить поэтапно, используя предыдущую стратегию решения проблемы и рассматривая отдельные группы последовательных или параллельных соединений.При нахождении \ (R_ {eq} \) для параллельного соединения необходимо с осторожностью относиться к обратному. Кроме того, единицы и числовые результаты должны быть разумными. Эквивалентное последовательное сопротивление должно быть больше, а эквивалентное параллельное сопротивление, например, должно быть меньше. Мощность должна быть больше для одних и тех же устройств, подключенных параллельно, по сравнению с последовательными и т. Д.

      Авторы и ссылки

      • Сэмюэл Дж.

    Добавить комментарий

    Ваш адрес email не будет опубликован. Обязательные поля помечены *